Internal Medicine EOR

Ace your homework & exams now with Quizwiz!

Cholecystitis

INFLAMMATION OF THE GALLBLADDER

What are specific SE of INH? RIF? EMB?

INH - hepatitis, peripheral neuropathy RIF - hepatitis, flue syndrome, orange urine EMB - optic neuritis (red-green vision loss)

How do you manage a patient with active TB?

INH/RIF/PZA/Ethambutol (EMB) for 2 months followed by 4 months of additional multidrug treatment based on culture and sensitivity results. Patients require combination chemotherapy for 6-9 months

how do u treat latent tb

INHx9 months

hypertensive crises Rx

IV nitroprusside nitroglycerin labetalol (safe for pregnancy) esmolol fenoldopam hydralazine (safe for pregnancy) nicardipine (safe for pregnancy) clevidipine phentolamine enalaprilat PO captopril labetalol (safe for pregnancy) clonidine hydralazine (safe for pregnancy)

Ventricular Fibrillation Tx:

IV access, CPR, defibrillation

What is the treatment for hemodynamically stable VT? What about unstable?

IV amiodarone (or sotalol or procainamide) Cardioversion

What is the treatment for VTE post surgery?

IV heparin

Pericarditis work-up and Treatment

IV, monitor, pain control EKG CXR CBC, BMP, ESR, Trop(4-6 hours after onset of chest pain), +/- blood cultures Tx: Anti-inflammatory agent,Cardiology consult

Cholecystitis Treatment

IV, pain control, NPO DDx: Pancreatitis, hepatitis, PUD, renal lithiasis, AMI, PE CBC, CMP, lipase, UA Ultra Sound Outpatient surgical consult for most cholelithiasis Admission for cholecystitis- medicine vs surgery

Treatment of Guillian Barre Syndrome

IVIG or plasmapheresis, may need intubation. Hospitalized pt with close monitoring. Recovery is slow but approximately 60% make full recovery within 1 year.

Why may a patient who presents w inferior wall MI (sudden substernal, severe crushing pain, not relieved w rest, EKG abnormalities) also have bradycardia, diarrhea, and lightheadedness

IWMI comes off the RCA and MC is due to the posterior descending artery. It is located directly next to the left branch of the vagus nerve.

How do you establish a definitive diagnosis of TB?

Identification of M. tuberculosis from cultures (takes 6-8 weeks to grow) or DNA/RNA amplification (takes 1-2 days).

Guillian Barre Syndrome

Idiopathic polyneuropathy often following minor infections, immunizations or surgical procedures. Most times no cause is identified.

When do you evaluate a murmur further?

If >3/6, additional abnormal heart sounds, radiation, diastolic

Most common cause of post operative ileus? Causes of gastroparesis not related to surgery? Treatment for gastroparesis?

Ileus = decreased contractile activity of the gut Hypokalemia (also, magnesium depletion) small bowel Obstruction Bleed Medications (general anesthesia, opiates = constipation) Post surgical (cut the vagus nerve) 1. Diabetes (autonomic neruopathy) 2. Bulimia/Anorexia 1. Metoclopramide (dopamine antagonist) 2. Erythromycin

What is the treatment of HOCM?

Implantable defibrilator

What is a RBBB? What is a LBBB?

In a RBBB, the right fascicle is blocked, so the left ventricle in conducted first and then the impulse returns to the right With a LBBB, the right ventricle in conducted first, and the impulse returns to the right

Valvular Disease: Aortic Aneurysm Tx

In asymptomatic patients, monitoring is appropriate for lesions < 5 cm. ■ Surgical repair is indicated if the lesion is > 5.5 cm (abdominal), > 6 cm (thoracic), or smaller but rapidly enlarging. ■ Emergent surgery for symptomatic or ruptured aneurysms.

INH prophylaxis for 6-12 months is indicated in which patients?

In patients who tested negative in the past but converted to positive

PCI or CABG?

In the stable pt (ie, stable angina), angiography will tell whether we do PCI or CABG - single vessel: PCI - mult vessel or L main artery: CABG In the MI pt, angiography will tell us where to do PCI as problem is located to one artery - we can do CABG if PCI fails

In order, which medication classes are increased/added for persistent angina?

Increase dose of beta blocker. When maxed out, add Ca channel blocker.

Cardiogenic Pulmonary Edema pathophys:

Increase in pulmonary pressure (PCWP) secondary to ineffective filling/pumping of heart

Achalasia?

Increased LES pressure Diagnose with manometry **Solids and liquids**

PE findings with COPD?

Increased anteroposterior (AP) diameter Increased resonance on percussion Decreased breath sounds Early inspiratory crackles +/- wheezing Raspy, loud breathing with chronic bronchitis

Cardiac S3 is associated with what?

Increased filling pressures Mitral regurg Dilated cardiomyopathy CHF Pregnancy kids

What are CXR findings with chronic bronchitis?

Increased interstitial markings at bases Diaphragms are not flattened Non-specific peribronchial and perivascular markings

In systolic HF, back pressure results in increased pressure in the pulmonary vasculature which is known as this:

Increased pulmonary capillary wedge pressure (PCWP) >12; [the increased hydrostatic pressure causes extravasation of fluid - pulmonary edema]

Pneumonia

Infection of the bronchoalveolar tree can be caused by common nasopharyngeal bacteria (i.e., typical pneumonia) or bacteria, viruses, or fungi from the surrounding environment (i.e., atypical pneumonia); common causes vary by age group. (See the next cards for chart)

What is the etiology of solitary pulmonary nodules?

Infectious granulomas (from old/active TB, fungal infections, or FB reaction) Malignancy (40%) - carcinoma, hamartoma, mets, bronchial adenoma

What is acute bronchitis?

Inflammation of airways (trachea, bronchi, bronchioles). Characterized by cough.

Pericarditis

Inflammation of pericardium Cause: Idiopathic, viral, malignant, Dressler's S/Sx: Sharp, *Restrosternal, Improved with sitting up and leaning forward, Friction Rub* Common presentations: young, chest pain, no risk factors, after viral infection Dressler's common after procedure for MI

What causes a pleural effusion?

Inflammation of structures adjacent to the pleural space or lesions within the chest

What is pneumonia?

Inflammation of the alveoli or interstitium of the lung caused by microorganisms. It's the primary cause of mortality from infectious disease. Usually caused by a bacteria but can be caused by virus or fungus.

Cholangitis

Inflammation of the bile duct

Myocarditis

Inflammation of the myocardium, with many possible causes, including viruses (e.g. Coxsackie B), bacteria (e.g., group A streptococcus in rheumatic fever, Lyme disease, mycoplasma, and so on) , SLE, medications (e.g., sulfonamides); can also be idiopathic • May be asymptomatic, or may present with fatigue, fever, chest pain, pericarditis, CHF, or even death, SOB, arrhythmia, Fluid retention, myalgia • Look for elevations in cardiac enzyme levels and erythrocyte sedimentation rate. • Treatment is supportive. Treat underlying causes if possible, and treat any complications. Dx eval: EKG; endomyocardial, ECG = ST- and T-wave changes, conduction abnormalities

What is acute bronchitis?

Inflammation of trachea and bronchi caused by spread of URI or exposure to inhaled irritants.

What viruses cause CAP?

Influenza RSV Adenovirus Parainfluenza

Severe Acute asthma exacerbation treatment?

Inhaled B-agonist Anticholinergic medication by nebulizer IV steroids supplemental Oxygen

What meds are for long acting control of asthma?

Inhaled corticosteroids Long acting beta agonist (LABA) Cromolyn, Nedocromil Leukotriene modifier agonists Theophylline

Pathophysiology for dobutamine/digoxin

Inhibition of Na/K-ATPase on cardiac myocytes; preserves intracellular Na therefore slowing Na/Ca exchange and preserves intracellular Ca

What is the initial diagnostic test of choice for cor pulmonale? What test definitively diagnoses cor pulmonale

Initial - Doppler echo Definitive - cath

Clinical features of idiopathic pulmonary fibrosis?

Insidious dry cough Extertional dyspnea Constitutional symptoms (fatigue, malaise, etc) Clubbing of fingers Inspiratory crackles

Pathophysiology of cardiac tamponade?

Inspiration causes increased blood in the right ventricle --> interventricular septum has to bulge over into the left ventricle -> decreased LV preload --> decreased stroke volume

Cardiorenal syndrome Example?

Interchange between the kidney and the heart and how a disease in one organ can affect the other. CHF --> low cardiac output --> renal failure

In a CABG procedure, what vessels are used to bypass the blocked artery?

Internal thoracic artery Saphenous vein

What is Idiopathic pulmonary fibrosis (idiopathic fibrosing interstitial pneumonia)?

Interstitial lung disease. Three histopathologic types each with different natural histories and treatments

Aortic Dissection

Intimal tear with leaking blood into media and longitudinal cleavage from the adventitia Risk factors: *HTN*, connective tissue disease (Marfan's), pregnancy, smoking, aortic valve abnormalities M>F, 50-70 years old, 33% mortality if untreated

Causes of Postrenal Failure

Intrarenal: crystals, protein casts Ureteral: Calculi, tumor Urethral/bladder: BPH, neurogenic bladder

What are the 3 ways to cause esophageal stricture?

Intrinsic disease: narrows esophageal lumen by inflammation, fibrosis, neoplasia. Extrinsic disease: compromise esophageal lumen by direct invasion Other diseases that disrupt esophageal peristalsis: peptic, autoimmune, infectious, congenital, med induced, radiation induced, malignant, idiopathic

AFib

Irregularly Irregular Rate Control Rhythm Conversion Cardioversion if unstable

What is systolic dysfunction?

Is a ↓ ejection fraction and dilation of the heart. It is characterized by a ↓ ejection fraction (less than 45%). The strength of ventricular contraction is attenuated and inadequate for creating an adequate stroke volume, resulting in inadequate cardiac output. On the left side of the heart, the increased pressure is transmitted to the pulmonary vasculature, and the resultant hydrostatic pressure favors extravasation of fluid into the lung parenchyma, causing pulmonary edema.

What is diastolic dysfunction?

Is the ability of the heart to "relax" and receive blood. The failure of ventricular relaxation also results in elevated end-diastolic pressures, and the end result is identical to the case of systolic dysfunction (pulmonary edema in left heart failure, peripheral edema in right heart failure.)

A positive exercise stress test is defined as:

Ischemia - 2mm or more of ST depression Hypotension - drop of systolic BP or 10mmHg

CVA Signs and Symptoms

Ischemic: Abrupt, loss of function, Multiple syndromes based on anatomy, HTN Hemorrhagic: Abrupt, Loss of Function, Vomiting, Headache, Altered mental status (coma), HTN (>200 systolic often)

CVA Treatment

Ischemic: Activate stroke team, TPA vs Intervention, Modest BP control (no more than 25% decrease) <185/110 Hemorrhagic: Activate Stroke Team, Contact neurosurgeon, Modest BP control (variable practice, Nimodipine), Consider FFP, rVIIA, platelets, Reverse anti-coagulation

How do you manage a patient with LTBI?

Isoniazid (INH) for 9 months OR Rifampin (RIF) AND Pyrazinamide (PZA) for 2 months Note: Must rule out active TB

What is Myocardial infarction?

It happens when blood stops flowing properly to part of the heart and the heart muscle is injured due to not receiving enough oxygen.

the term"takotsubo" comes from...

Japanese name for octopus trap (shape is similar to systolic apical ballooning appearance of the left ventricle in the most common and typical form of this disorder

JONES criteria pneumonic

Joints Oh, no-carditis! Nodules Erythema marginatum Sydenham's chorea

rheumatic fever Dx (diagnosis based on ___ criteria)

Jones criteria -major criteria -minor criteria

hypertension Dx

K BUN Cr Ca glc Hct U/A lipids TSH urinary albumin: creatinine (if increased Cr, DM, or peripheral edema) renin? EKG (for LVH) CXR TTE (evaluate abnl valves, LVH)

Skin conditions associated with HIV?

Kaposi sarcoma (HHV-8) Bacillary angiomatosis (from the bacteria bartonella) Molluscum contagiosum

Herpes infections can cause what eye infection?

Keratoconjunctivitis (Dendrites with fluorescin dye)

Common radiographic finding in CHF that denotes interstitial edema:

Kerley B lines

ACEi protects the

Kidney. That way it's good for DM pt. Switch to ARBs if pt have cough symptoms.

What bug is most likely to cause pneumonia in a patient with ETOH abuse?

Klebsiella

EKG criteria for LBBB and RBBB?

LBBB: - QRS >120 msec - Broad monophasic R waves in (lateral leads) Lead 1, v5, v6 - Terminal S wave in lead V1 - Deep S waves in V1-V3 RBBB: - QRS >120 msec - RsR' (bunny ears) in lead v1

What serum marker helps with the diagnosis of PCP pneumonia?

LDH (elevated in PJP, especially in HIV pts. )

What vaccines are contraindicated in HIV patients?

LIVE vaccines Small pox Yellow fever Chicken pox (varicella) MMR Oral polio

DVT prophylaxis

LMWH (Lovenox)

Pneumonia Labs/DX

Labs = increased white blood cell count (WBC) (slight increase with viral cause, significant increase with bacterial or fungal cause) with left shift (more immature forms); positive sputum culture and possible positive blood culture with bacterial or fungal cause. Dx: CXR

What lab test can identify and monitor sepsis?

Lactate

Side effect of metformin?

Lactic acidosis especially in patients with *renal disease* (low HCO3 with anion gap in pt)

What conditions cause the following jugular venous waves? Large a wave Cannon a wave Absent a wave Large CV wave Slow y descent Rapid y descent Steep y descent

Large a wave - Tricuspid stenosis, Pulmonary stenosis, pulmonary HTN Cannon a wave - Ventricular tachycardia, AV block, PAC Absent a wave - Atrial fibrillation Large CV wave - Tricuspid regurgitation Slow y descent - Cardiac tamponade Rapid y descent - Constrictive pericarditis Steep y descent - Constricitve pericarditis

Arrhythmia Considerations

Large and heterogeneous group of conditions in which there is abnormal electrical activity in the heart. Benign vs. Malignant, Asymptomatic vs. Symptomatic (palpitations, dizziness, weakness, chest pain, dyspnea) Fast or slow, Stable or unstable Multifactorial

Dilated Cardiomyopathy

Left Ventricular dilation and ↓EF must be present for diagnosis.

HF is often classified as Left Heart Failure or Right Heart Failure, or both.

Left sided failure is more common, and leads to venous congestion of the pulmonary circulation, resulting in pulmonary edema.

What bug is most likely to cause pneumonia in a patient with exposure to aerosolized water?

Legionella

What are atypical bacteria that cause CAP?

Legionella Mycoplasma Chlamydia

___ is when a patient describes their chest pain by holding their clenched fist over their chest; considered to be a sign of ischemic chest pain

Levine's sign

PE findings of bronchiectasis?

Localized chest crackles and clubbing of fingers

#1 cause of aortic regurgitation

Longstanding HTN; also happens in pts w connective tissue disorders

1st thing to do when you find a pulmonary nodule on Chest X-ray? Benign vs. malignant nodules

Look at old chest Xray Benign: - Pt < 40 - < 3 cm - Concentric calfication Malignant: - Pt > 40 - > 3 cm - Eccentric calcification

How may the patient present with COPD?

Look for the classic *barrel chest*, use of accessory chest muscles, JVD, *SOB*, end-expiratory wheezing, and muffled breath sounds.

Shock Treatment

Look for underlying cause All bleeding stops....Eventually IV access (Central) -2 Large bore IVs Fluids first then blood Antibiotics early, source control Pressors after fluids and blood Ionotrops Epinephrine Consider Steroids (Stress dose steroids)

Dementia

Loss of mental capacity. Psychosocial level and cognitive abilities deteriorate and behavioral problems develop. Largest categories are Alzheimer dz and vascular dementia. Hallucinations, delusion, depression, repetitive behavior are common.

Wilson's disease serum findings

Low ceruloplasmin; High urinary Copper Hepatitis, Altered mental status, Corneal deposits

What is the clinical presentation of atypical PNA?

Low grade fever Dry cough Myalgia Fatigue Bullous myringitis (reddened tympanic membranes) <---pathognomonic of mycoplasma pneumonia

Appendicitis

Luminal obstruction-mucous accumulation-increased pressure-lymphatic obstruction- perforation Most common in 2nd and 3rd decades Classic: *Periumbilical pain, dull, migration to RLQ, sharp*, anorexia, vomiting, fever

Most common cause of clubbing? Other causes?

Lung cancer COPD

What is the leading cause of cancer deaths in men and women?

Lung cancer - overall survival rate is 15%

ITP is associated with

Lupus

What are PE findings with lung cancer?

Lymphadenopathy Hepatomegaly Clubbing of fingers

if cd4 count is less than 50, what are u using for ppx and which disease are you concerned about

MAC ppx is clarithromycin 5oo mg daily or azithro 1200mg weekly

Tx of CAP for healthy adults <60?

MACROLIDE such as erythromycin, clarithromycin, or azithromycin + supportive care (fluids, antipyretics, analgesics, relative rest) Alternative: Doxy

Inpatient MI complications:

MC complication: arrhythmia MC cause of sudden death: v fib

MEN 1 vs. MEN 2a vs. MEN 2B

MEN 1 - Pituitary, parathyroid, pancreas, zollinger ellison syndrome MEN 2a - medullary thyroid, Parathyroid, pheochromocytomas MEN 2b - Medullary thyroid, pheochromocytomas, Mucosal neuromas *Associated with marfinoid habitus* MEN 1 - men gene MEN 2a, 2b - ret gene

Pain is mostly substernal and associated w injury patterns on the electrocardiography and elevated cardiac biomarkers

MI

What does cocaine put you at risk for?

MI Stroke

MI Tx

MOAN BB *Morphine, Oxygen, Aspirin, Nitroglycerine, Beta-blocker*

What are characteristics of adenocarcinoma of lung?

MOST COMMON TYPE! Metastasizes to distant organs. Tumor arises from mucous glands and usually appears in periphery of lung. Not amenable to early detection through sputum examination.

Murmurs that worsen w valsalva

MVP, HOCM

What are typical manifestations of PNA due to Mycoplasma? Pneumocystis jiroveci? Legionella? Chlamydia? Strep pneumo? Klebsiella?

MYCOPLASMA - Low grade fever, cough, bullous myringitis, cold agglutinins PNEUMOCYSTIS JIROVECI - slow onset, immunosuppressed state (HIV+), increased LDH, profoundly hypoxemic, interstitial infiltrates LEGIONELLA - chronic cardiac or respiratory disease, hyponatremia, diarrhea CHLAMYDIA - longer prodrome, sore throat, hoarseness STREP PNEUMO - single rigor, rust-colored sputum KLEBSIELLA- currant jelly sputum, chronic illness (including alcohol abuse)

What antibiotic is appropriate for outpatient treatment of community acquired pneumonia (CAP)?

Macrolide (not erythromycin) or doxycycline

Treatment for Parkinson's Disease

Mainstay of tx are Levodopa and Carbidopa (when using together can reduce Levodopa to reduce AE)

Coronary vascular disease Major Risk factors

Male gender, ↑LDL , ↓HDL, DM, hypertension, a family history, smoking, and peripheral arterial disease.

Hiatal hernia History/PE

May be asymptomatic. Those with sliding hernias may present with GERD.

Treatment of Ulcerative colitis?

Mesalazine (5- ASA containing molecule) Sulfasalazine (5-ASA containing molecule) 5 - ASA are antiinflammatory drugs --> poorly absorbed by the intestine, that's why no systemic effect Severe disease - high dose IV steroids TNF inhibitors - infliximab (remicade), Humira (adalimumab)

What is the number one complication of asbestosis?

Mesothelioma

What is the medication of choice for treating symptomatic PVCs?

Metoprolol

The same drug is used to treat C. diff and Giardia, which is what?

Metronidazole

Exacerbations of COPD treatment?

Mild - short acting bronchodilators (B-agonists preferred) Moderate - short acting bronchodilators AND systemic corticosteroids, and/or Antibiotics Severe - hospital management (O2 therapy --> goal is arterial pO2 >60 mmHg or O2 sats >90%) *Consider ANTIBIOTICS for moderate/severe COPD exacerbation (Ceftriaxone/Macrolide or monotherapy with Fluoroquinolone) **O2 therapy is the only therapy proven to improve survival in COPD**

What are clinical features of chronic bronchitis?

Mild dyspnea Chronic productive cough Noisy lungs (wheezes and rhonchi) Peripheral edema Overweight and cyanotic

Renal Lithiasis

Mineral precipitation in collection system, Abrupt & sharp, Flank to groin pain, Writhing pain, N/V, diaphoresis DDx: Cholecystitis, splenic rupture, appendicitis, torsed gonad, shingles, trauma, diverticulitis, pyelonephritis IV, fluids, pain control (ketorolac) UA: expect hematuria, causion pyuria Non-contrast CT vs US +/- CBC, BMP Urology follow up, Tamsulosin, Opiate, Strainer

Diagnosis of Rheumatic Fever: Jones Criteria

Minor Criteria: ∙Fever ∙Arthralgia ∙Prolonged PR interval ∙Increased ESR or CRP ∙Leukocytosis

Rheumatic fever is most frequently associated with which of the following cardiac lesions?

Mitral Stenosis

Clue features: Transient pain, mid-systolic click, females

Mitral Valve Prolapse

Mid-systolic click w distinct hx/sx of chest pain, panic, palpitations

Mitral Valve Prolapse

Woman has chest pain, palpitations, and panic you should think of this murmur:

Mitral Valve Prolapse [worsens w valsalva, improves w squatting]

Systolic murmur w distinct hx/sx of prior MI or CAD, LVH, pronounced CHF

Mitral regurgitation

Systolic murmur appreciated best at the apex, S3 heart sound, wheezing, rales, rhonchi, JVD

Mitral regurgitation; tx w afterload reduction w nitrates and ACEi in non-emergent cases

Diastolic murmur w distinct hx/sx of systemic embolism, hoarseness, afib

Mitral stenosis

This murmur is classically heard better in a high volume state thus associated w pregnancy:

Mitral stenosis (blood flows through stenotic valve during diastole [mitral + tricuspid relaxation during diastole])

#1 etiology for murmurs

Mitral stenosis: rheumatic fever Mitral regurgitation: ischemia, infarcts Aortic stenosis: normal aging Aortic regurgitation: idiopathic MVP: congenital HOCM: congenital

Stereotypical pres cardiogenic pulmonary edema: Pt presents w worsening fatigue, dyspnea, and has a hx of untreated strep infxn and has had longstanding murmur

Mitral valve stenosis

Mallory-Weiss tear Tx

Most cases resolve spontaneously. If bleeding persists, injection of epinephrine can be used to stop the bleeding.

Post MI complications: MCC of death? New systolic murmur? Acute severe hypotension? VSD? Persistent ST elevation 1-2 months later - friction rub

Most common complications: 1. Arrhythmia 2. LV failure 3. Cardiogenic shock MCC death - V fib Papillary muscle rupture Free wall Cardiac tamponade Interventricular wall rupture Ventricular wall aneurysm Dressler's syndrome (autoimmune pericarditis)

Esophageal cancer signs and symptoms

Most common is dysphagia. 2nd most common is weight loss. Other: bleeding, epigastric pain, hoarse, cough

Myocarditis is common caused by

Most commonly caused by infection (e.g., Coxsackie virus, bacteria, rickettsiae, fungi, parasites) and occasionally caused by drug toxicity

Reciprocal ECG changes in MI

Most often reciprocal ECG changes occur in leads II, III, avF vs. leads I, aVL If there is an inferior STEMI, then you will see reciprocal ST depression in leads I, aVL and vice versa

Multiple competing pacemaker sites on atria which leads to tachycardia with at least 3 different morphologies of P waves w QRS complexes

Multifocal Atrial Tachycardia (MAT); cause is generally COPD or other respiratory disease

What is Sarcoidosis characterized by?

Multiorgan disease of idiopathic cause. Characterized by noncaseating granulomatous inflammation in affected organs (lungs, lymph nodes, eyes, skin, liver, spleen, salivary glands, heart, nervous system) 90% have lung involvement Incidence is highest in AA (especially women) and northern European whites

Heart Murmurs

Murmurs are abnormal (Harsh blowing sounds) heart sounds attributed to turbulent blood flow, between the chambers of the heart or from the heart into the pulmonary or aortic systems. Innocent - common in healthy children and young adults, often resolve with time

What pathogen causes tuberculosis (TB)? How is it spread?

Mycoplasma tuberculosis - spread by aerosol droplets expelled by a person's cough with active disease

What bug is most likely to cause pneumonia young adults?

Mycoplasma/chlamydia

What is the most common risk factor for the most common cause of death:

Myocardial Infarction

Clue features: chronic, vague and mild pain, fever

Myocarditis

What is the earliest cardiac marker to increase after an MI?

Myoglobin increases at 1-2 hours

Cardiac enzymes are raised for:

Myoglobin within 1-4h of MI and normalizes by 1 day CK-MB, Troponin I and T all rise about 4-9h following MI; CK-MB normalizes by 2-3 days, troponins take approximately 1-2 weeks

Treatment of pericarditis (viral, Dressler's syndrome)

NSAIDs are first line!

Tension HA treatment

NSAIDs or ASA combined with caffeine. APAP less effective but preferred in pregnancy. 2nd line therapies are antidepressants (amitriptyline)

What is the first line therapy for pericarditis?

NSAIDs or aspirin

Migraine treatment

NSAIDs, Triptans, antiemetics

How do you treat NSCLC vs SCLC?

NSCLC - TOC is surgery. 5 year survival rate is 35-40% SCLC - TOC is chemo

coronary heart disease includes

NSTEMI STEMI/ unstable angina angina pectoris prinzmetal/variant angina

A blockage that causes a superficial ischemia of the myocardium

NSTEMI; fibrinolytics are not useful in these patients

Normal Left Ventricular Ejection Fraction:

NV: 55-70%; Reduced LVEF w sx is systolic congestive HF; in diastolic CHF - LVEF will be normal... If echo shows LVEF <40 - SCHF

PE findings of pulmonary HTN?

Narrow splitting and accentuation of the 2nd heart sound Systolic ejection click

What are clinical features of lung cancer?

New or changing cough Hemoptysis Pain Anorexia Weight loss Asthenia

Pt presents with unstable angina. What is the next best step?

Next best step: EKG Then: cardiac enzymes (CK-MB and troponin) Also: Give ASA or plavix, BB (IV or PO), nitrate, supp O2, heparin, or LMWH

How do you treat high BP with stroke?

Nitroprusside (short half life, easy to titrate) or IV labetalol

Do you need to start someone on DVT prophylaxis (Lovenox) if they are already on Dual Antiplatelet therapy (DAPT)?

No

Kussmaul sign? seen in what conditions

No fall in the JVP on inspiration - Ordinarily the JVP falls with inspiration due to reduced pressure in the expanding thoracic cavity and the increased volume afforded to right ventricular expansion during diastole. Kussmaul's sign suggests *impaired filling of the right ventricle* due to either fluid in the pericardial space or a poorly compliant myocardium or pericardium. This impaired filling causes the increased blood flow to back up into the venous system, causing the jugular vein distension (JVD) and is seen clinically in the external jugular veins becoming more readily visible. Constrictive pericarditis, restrictive cardiomyopathy

Ischemic stroke is the most common type (87%) and has what signs and symptoms?

No headache, not visible on CT without contrast until 6 hours post stroke

Treatment for C. diff? SEVERE C. diff?

Nonsevere C. diff first line is Flagyl 10-14 days Recurrent C diff use Flagyl again After that use *PO* vancomycin Severe C . Diff= elevated WBC and elevated creatinine Use PO vancomycin

Why is a normal PaCO2 in a COPD exacerbation a bad thing?

Normally, in a COPD exacerbation, you are hyperventilating --> blowing of CO2...If you have a normal CO2 after hyperventilating it means you started out with more than usual (CO2 retention)

Classification of a positive TB test

Note: measurement is induration not erythema >5 is positive if immunocompromised >10 if patient has risk factors (immigrant, IVDU, health care worker, etc) >15 if patient has no risk factors

What is therapy for reducing RV afterload?

O2

acute decompensated heart failure Tx for low perfusion

O2 BiPAP nitrates adjustment of oral meds -ACE/ARB -BB -diurectics (furosemide, bumetanide, torsemide)

What is Obesity hypoventilation syndrome (OHS)?

OHS exists when an obese individual (BMI>30) develops awake alveolar hypoventilation (PaCO2 >45 mmHg), which cannot be attributed to other conditions. Most patients with OHS present with chronic hypoventilation, although in about a third of cases the diagnosis is made during a hospitalization for acute respiratory failure

Obstructive sleep apnea vs. Obesity hypoventilation syndrome?

OSA - normal ABG Obestiy hypoventilation syndrome (Pickwickian syndrome) - elevated PaCO2, reduced PaO2

What are clinical manifestations of OHS?

Obesity <----hallmark Hypoexmia Co-existing OSA: (Excessive daytime sleepiness, loud snoring, choking during sleep, resuscitative snorting, fatigue, hypersomnolence, impaired concentration and memory, a small oropharynx, and a thick neck)

MI risk factors

Obesity, metabolic syndrome, fam hx of MI, tobacco abuse, male gender, hypercoagulable states (sickle cell and polycythemia)

What are PFT findings with COPD?

Obstructive pattern FEV1/FVC ratio is decreased

Pulmonary Embolism

Occlusion of pulmonary artery Most originate from thrombi in pelvis/LE >400K per year Current or previous , Malignancy, obesity, estrogen, immobility, trauma, surgery are risk factors S/Sx: Dyspnea, pleuritic chest pain, hemoptysis (late), tachypnea, tachycardia

Occupation, diagnosis, and complications from coal workers' pneumoconiosis?

Occupation: Coal miner Diagnosis: CXR will show nodular opacities in UPPER lung fields Complication: Progressive massive fibrosis

Occupation, diagnosis, and complications from Berylliosis?

Occupation: High-tech fields (aerospace, nuclear, ceramics, foundries, tool and die manufacturing) Diagnosis: CXR will show diffuse infiltrates and hilar adenopathy Complication: Requires chronic steroids

Occupation, diagnosis, and complications from asbestosis?

Occupation: Insulation, demolition, construction Diagnosis: Biopsy will show asbestos bodies. CXR will show linear opacities at BASES and pleural plaques. Complication: Bronchogenic lung cancer, Mesothelioma

Occupation, diagnosis, and complications from Silicosis?

Occupation: Mining, sand blasting, quarry work, stone work Diagnosis: CXR will show nodular opacities at UPPER lung fields Complications: Increased risk of TB. Progressive massive fibrosis

Postrenal Failure

Occurs primarily in elderly men with high grade prostatic obstruction. Significant permanent loss of renal function occurs over 10-14 days with complete obstruction and worsens with associated UTI.

Term for painful swallowing?

Odynophagia

Primary cause of Diastolic CHF

Often due to long standing HTN [heart grows stronger to pump against resistance, but in turn loses compliance]; other causes: aortic stenosis

Blood transfusion? Effect of 1 unit of blood?

Only when Hb less than 7 Give one unit at a time 1 unit of packed RBCs should raise Hb by 1, HCT by 3%

Differential diagnosis for eye emergencies?

Optic neuritis Central retinal artery occlusion (CRAO) Central retinal vein occlusion (CRVO) Retinal detachment (usually trauma) Papilledema Temporal arteritis

Pulmonary fibrosis Tx

Options include corticosteroids, cytotoxic agents (azathioprine, cyclophosphamide), antifibrotic agents (have not been shown to improve survival), and lung transplantation.

SE of inhaled steroids?

Oral candidiasis Growth stunting (with high dose ICS)

Where does SCLC originate from?

Originates in the central bronchi and metastasizes to regional lymph nodes.

Central Pontine Myelenosis is now called what?

Osmotic demyelination syndrome

Pain with pulling on the pinna?

Otitis externa

Diverticular disease

Outpouchings of mucosa and submucosa (false diverticula) that herniate through the colonic muscle layers in areas of high intraluminal pressure; most commonly found in the sigmoid colon.

Cluster headache treatment

Oxygen, Sumatriptan 6mg SQ, start preventive therapy as soon as headache onset to suppress attacks over expected duration of cluster period

Premature atrial contraction (PAC) vs. Premature ventricular contraction? (PVC)

PAC: P wave preceding extra beat is abnormal QRS normal No full compensatory pause --> R-R intervals not equal PVC: No p wave preceding extra beat Wide QRS Full compensatory pause --> R-R intervals equal

PCI vs. CABG?

PCI (coronary angiography with a balloon and stenting) 1, 2, 3 vessel disease CABG: - 3 vessel disease with >70% stenosis - Left main coronary artery diseae >50% stenosis - Left ventricular dysfunction - Cardiogenic shock post MI - Diabetes + multivessel disease

4 most common causes of pleural effusions

PE Pneumonia CHF COPD

Common Respiratory causes of chest pain:

PE, Pleuritis, Pneumothorax, Pneumonia

pulmonary embolism use ___ rule; PE is less likely if...

PERC rule: age < 50 pulse ox > 94% on room air HR < 100 bpm no prior VTE no recent surgery or trauma no hemoptysis no estrogen use no unilateral leg swelling

Diagnostic studies and findings for pneumoconiosis?

PFT- will show restrictive pattern CXR

What other condition does acute bronchitis present like?

PNA

What is community acquired pneumonia (CAP)?

PNA acquired from the home or a non-hospital environment

What are differentiating features of PNA and bronchitis?

PNA is more likely to have a high fever, more dyspnea, chills, chest pain, and hypoxia.

What is hospital acquired pneumonia (HAP)?

PNA that is caused by organisms that colonize ill patients, staff, and equipment producing clinical infection >48 hours after admission to the hospital. It is the 2nd MC cause of hospital acquired infections (after UTI from foleys)

After excluding cardiac causes of chest pain, a trial of what medication is warranted?

PPI

PPI triple therapy

PPI Amoxicillin (metronidazole if allergic to penicillin) Clarithromycin (tetracycline if allergic) 10 days - 2 weeks

1st degree block Dx

PR interval > 200 ms (0.2 sec)

Stereotypical pres cardiogenic pulm edema: Pt is being hospitalized and is a few days post MI, suddenly has developed severe dyspnea, pink frothy sputum, and new systolic murmur:

Papillary muscle rupture

What bug is most likely to cause pneumonia in children less than 2 years old?

Parainfluenza

Caused by an ectopic stimulation of the ventricles originating somewhere in the atria besides the SA node that is tachy and intermittent

Paroxysmal Supraventricular Tachycardia (PSVT)

Myocarditis H/P

Patient may report history of recent upper respiratory infection; pleuritic chest pain, dyspnea, pleuritic chest pain; S 3 or S4 heart sound, possible diastolic murmur, possible friction rub

Gastritis HISTORY/PE

Patients may be asymptomatic or may complain of epigastric pain, nausea, vomiting, hematemesis, or melena.

Reactive arthritis

Patients may have asymmetric arthritis that involves large joints usually below the waist (i.e., knee and ankle); *mucocutaneous lesions (balanitis, stomatitis), urethritis, and conjunctivitis are common.*

Who is at highest risk for HAP?

Patients on mechanical ventilation

Most common food allergies in both children and adults?

Peanut Allergies

What abx are first line for use in pregnant women? (since bactrim and cipro are both contraindicated)

Penicillin Nitrofurantoin

Treatment for refractory status epilepticus

Pentobarbital infusion, Isoflurane anesthesia

Its primary use in medicine is in treating the symptoms of intermittent claudication resulting from peripheral artery disease.

Pentoxifylline

Clue features: Pain is epigastric, hx of ASA/NSAIDs use, belching, bloating

Peptic Ulcer Disease

Clue features: positional; worse w laying down, pleuritic pain, sharp, EKG anomolies; feels better leaning forward

Pericarditis

2nd line treatment for active seizures in ED

Phenytoin/fosphenytoin, Magnesium (for seizures thought to be due to eclampsia), Valproate

Cholelithiasis Dx

Plain x-rays are rarely diagnostic; only 10-15% of stones are radiopaque. ■ RUQ ultrasound is the imaging modality of choice (85-90% sensitive).

Reactive thrombocytosis?

Platelets are *acute-phase reactants* therefore, they increase in response to various stimuli, including systemic infections, inflammatory conditions, bleeding, and tumors.

Why put someone with CAD history or risk on a statin even if their cholesterol is low?

Pleiotropic effect of statins improving endothelial function, enhancing the stability of atherosclerotic plaques, decreasing oxidative stress and inflammation, and inhibiting the thrombogenic response.

What CXR findings are seen in asbestosis?

Pleural based plaques

Pulmonary embolism symtoms/signs?

Pleuritic chest pain Hemoptysis Tachypnea Tachycardia Decreased pO2

Clue features: pain on respiration, fever, rapid shallow breathing, scratchy sound on auscultation of the lung

Pleuritis

What is the MC opportunistic infection in patients with HIV disease (especially if their CD4 count is <200)?

Pneumocystis jiroveci

What is the most common opportunistic infection in patients with HIV infection?

Pneumocystis jiroveci (formerly P. carinii)

Clue features: Fever, inc WBCs, crackles/rales

Pneumonia

Chest x-ray findings:

Pneumonia - Interstitial infiltrates, Consolidation, opacification, air bronchograms Emphysema - Hyperlucent lung fields with flattened diaphragms and increased A-P diameter CHF - Cardimegaly, Kerley B lines, cephalization, Pulomonary vascular congestion, bilateral pleural effusions Pleural effusion - blunting of costophrenic angles, 250 mL needed for blunting Pulmonary abscess (aspiration) - cavity containing an air-fluid level Tuberculosis - upper lobe cavitation, consolidation, +/- hilar adenopathy Pneumothorax - loss of normal lung markings in the periphery of the hemithorax; presence of a well-defined, visceral pleural line at some point b/w chest wall and the hilum

Clue features: sudden, absent breath sound on one side, hypoxia, hx rib fracture, hx penetrating wound, young + tall pt

Pneumothorax

Sudden onset of pleuritic pain with decreased breath sounds on affected side

Pneumothorax

Todd's paralysis

Postictal weakness or paralysis that is often unilateral and resolves over 24 hours

Valvular Disease: Lymphedema Hx/PE

Postmastectomy patients present with unexplained swelling of the upper extremity. ■ Immigrants present with progressive swelling of the lower extremities bilaterally with no cardiac abnormalities (i.e., fi lariasis).

Preventive therapies for cluster headache lasting less than 2 months

Prednisone 60-100mg daily x5 days, taper with 10mg

Treatment for Bell's palsy

Prednisone 60mg x 5 days, Valcyclovir 1000mg TID x 7 days, artificial tears, lacri lube at night

Sarcoidosis Tx

Prednisone is the clear drug of choice. Few patients fail to respond. Systemic corticosteroids are the treatment of choice. -Asymptomatic hilar adenopathy does not need to be treated.

This refers to the end diastolic volume and the resultant fiber length of the ventricles prior to the onset of contraction:

Preload

Tx for mitral stenosis:

Preload reduction w na restricted diet, diuretic (furosemide)

Things that worsen HOCM (decreases LV volume)

Preload reduction: hypovolemia, tachycardia, valsalva, arrhythmia Afterload reduction: positive inotropes, beta agonists, nitrates, ACEi

Premature atrial complexes are caused by what Triggers?

Premature activation of the atria originating from a site other than the SA node (early P wave) TObacco, alcohol, caffeine, stress Usually asymptomatic

Major clue asthma vs. COPD?

Presence of nocturnal symptoms in asthma

Bronchiectasis HISTORY/PE

Presents with chronic cough accompanied by frequent bouts of yellow or green sputum production, dyspnea, and possible hemoptysis and halitosis.

Pulmonary hypertension Presentation

Presents with dyspnea on exertion, fatigue, lethargy, syncope with exertion, chest pain, and symptoms of right-sided CHF (edema, abdominal distention, JVD).

Valvular Disease: Deep Venous Thrombosis (DVT) Hx/PE

Presents with unilateral lower extremity pain, erythema, and swelling. ■ *Homans' sign* is calf tenderness with passive foot dorsiflexion (poor sensitivity and specificity for DVT).

What are the 2 types of CHF heart failure and how to treat? Treatments for both?

Preserved ejection fraction (diastolic heart failure) - empiric treatment (control of hypertension, pulmonary congestion and peripheral edema with diuretics) Reduced ejection fraction (systolic heart failure) (prevention of further remodeling (ACE I, BB, diuretics)

Right sided CHF predominantly has this:

Primarily signs of fluid retention

What are causes of pulmonary HTN?

Primary (idiopathic) - rare and fatal Secondary - from obliteration and obstruction of the pulmonary arterial tree. Many causes.

Patient has IBD, and presents with Elevated Alk Phos and GGT

Primary Sclerosing cholangitis

Antimitochondrial Ab

Primary biliary cihrrosis

What is the difference between a primary and a secondary headache?

Primary has no underlying etiology. Secondary results from underlying disease, mass, infection, CVA, trauma, drug withdrawal, metabolic disorders.

Chest pain in cycles that may occur during rest that is due to vasospasm. Dx on angiogram and tx w CCBs

Prinzmetal's angina

What causes poor R wave progression?

Prior anteroseptal MI

What prodrome suggests acute viral hepatitis infection?

Prodrome of N/V, malaise, fever followed by liver enlargement, abdominal pain, jaundice

Prerenal failure

Produced by conditions that decrease renal perfusion and is most common cause of community acquired acute renal failure (70%)

Pneumonia H/P

Productive or nonproductive cough, dyspnea, chills, night sweats, *pleuritic chest pain; decreased breath sounds* , rales, wheezing, *dullness to percussion* , egophony (i.e., change in voice quality heard during auscultation over a consolidated region of lung), tactile fremitus, *tachypnea*

Cholecystitis Info

Prolonged blockage of the cystic duct, usually by an impacted stone, that leads to obstructive distention, infl ammation, superinfection, and possibly gangrene of the gallbladder (acute gangrenous cholecystitis). Acalculous cholecystitis occurs in the absence of cholelithiasis in patients who are chronically debilitated, those who are critically ill in the ICU or on TPN, and trauma or burn victims.

Essential tremor does not occur at rest, occurs bilaterally, and has no indication of other neuro signs. How do you treat essential tremor?

Propranolol, Primidone (can combine these 2 if needed)

Benzodiazapenes can cause what toxicity?

Propylene glycol toxicity --> metabolic acidosis

What is tested in the fluid obtained from a thoracentesis?

Protein Lactate dehydrogenase (LDH) pH total WBC with diff Glucose Cytology Gram stain with culture and sensitivity

PQRST of pain:

Provoking/palliating, quality, region, severity, time

What bug is most likely to cause pneumonia in a patient with cystic fibrosis?

Pseudomonas

What is the most likely pathogen for ICU acquired pneumonia?

Pseudomonas

Irritable bowel syndromeTx

Psychological: Patients need reassurance from their physicians. They should not be told that their symptoms are "all in their head." ■ Dietary: Fiber supplements (psyllium) may help. ■ Pharmacologic: Treat with TCAs, antidiarrheals (loperamide), and antispasmodics (anticholinergics such as dicyclomine).

How do you diagnosis Bronchiectasis?

Pt will have very high volume of purulent sputum production is the key to the suggestion to the diagnosis. Dynpnea and wheezing are present in 75%. Other finding: Weight loss, anemia, crackles on lung exam, rales, rhonchi, purulent mucus, and occasional hemoptysis.

Pt has + stress test, has an angiogram and is found to have 3-vessel disease or left main artery disease:

Pt will require a coronary artery bypass graft [other pts can get angioplasty w/stenting]

Pain is sudden onset, pleuritic, and associated with hypoxia

Pulmonary Emboli

Pulmonary Vascular Diseases:

Pulmonary Embolism

What is the most accurate diagnostic test for asthma?

Pulmonary Function testing / Spirometry

What is Pulmonary hypertension?

Pulmonary arterial pressure rises to a level that is too high for a given cardiac output. Often self-perpetuating

Sx of LV dysfunction:

Pulmonary edema, rales, SOB, dyspnea

Clue features: sudden increased RR, hypoxia, air hunger, dyspnea, calf swelling, surgical pt, pain on respiration

Pulmonary embolism

Diagnostic studies for asthma?

Pulmonary function testing (PFTs): Forced expiratory volume in 1 second (FEV1)/ Forced vital capacity (FVC). ABGs CXR - may show hyperinflation. Usually normal. PEF meter Histamine or methacholine challenge test - helps when spirometry isn't diagnostic

What is Pulmonary hypertension?

Pulmonary hypertension is defined as a mean pulmonary arterial pressure of > 25 mmHg (normal = 15 mmHg). It is classified as either 1° (if the etiology is unknown) or 2°. 1° pulmonary hypertension most often occurs in young or middle-aged women. The main causes of 2° pulmonary hypertension include the following: (View Pic)

Bazett's formula for corrected QT prolongation?

QTc = QT/ square root of RR

Corrected QT interval? What does a prolonged QT interval predispose you to?

QTc is prolonged if > 440ms in men or > 460ms in women QTc > 500 is associated with increased risk of *torsades de pointes* QTc is abnormally short if < 350ms *A useful rule of thumb is that a normal QT is less than half the preceding RR interval*

prominent nail pulsations

Quincke's pulse

What bug is most likely to cause pneumonia in children less than 1 year old?

RSV

Signs and symptoms of acute pancreatitis

RUQ pain with "boring" quality radiating to upper back, nausea, vomiting Severe: hypotension, sepsis, multiorgan failure

Signs and symptoms of Hepatitis

RUQ pain, epigastric/abd pain, N/V/D, jaundice, pruritus, pale stools, dark urine

What will EKG show with cor pulmonale? (3)

RV hypertrophy (R axis deviation) Atrial hypertrophy RV strain (inverted T waves and ST depression in V1 and V3)

How do female sex, kidney failure, old age, and obesity affect BNP levels?

Raise, Raise, Raise, Lower (can cause false negatives)

Tx for Atrial Flutter:

Rate control: CCB; Warfarin for anticoagulation (same as a fib)

Spironolactone MOA

Reduce Na reabsorption in nephron, therefore reducing fluid volume (comp inhibitor of aldosterone)

MC cause of secondary hypertension:

Renal Artery Stenosis

Side effects of Amphotericin B?

Renal Toxicity Hypokalemia Infusion reaction AmphoTERRIBLE

Management of solitary pulmonary nodule that has a high probability of malignancy?

Resection ASAP. Biopsy is not recommended.

Clinical features of sarcoidosis?

Respiratory sx (cough, dyspnea, chest discomfort) Malaise, fever, sx consistent with organ involvement Extrapulmonary findings - erythema nodosum, enlargement of parotid glands, lymph nodes, liver, or spleen

What is asthma?

Reversible airway obstruction secondary to bronchial hyperactivity, acute airway inflammation, mucous plugging, and smooth muscle hypertrophy. -Abnormal bronchoconstriction of the airway

#1 cause of Mitral Stenosis

Rheumatic fever (usually secondary to an untreated GABHS infection)

More than 90% of cases of acute bronchitis are caused by what three viruses?

Rhinovirus Coronavirus RSV

Main side effects of the TB drugs?

Rifampin - hepatotoxicity, orange body fluids Isoniazid - hepatotoxicity, peripheral neuropathy Pyrazinamide - hepatotoxicity Ethambutol - optic neuritis, color blindness

TB treatment? Latent TB?

Rifampin, Isoniazid (+ B6 pyridoxyl phosphate), Pyrazinamide, Ethambutol daily for 2 months + Rifampin, Isoniazid 3x week for 4 months Latent TB: Isoniazid alone fr 6-9 months

What is Cor Pulmonale?

Right ventricular enlargement from primary lung disease, commonly from pulmonary hypertension. Eventually leads to RV failure. Note that Right-sided heart disease due to left-sided heart disease or congenital heart disease is NOT considered cor pulmonale.

Valvular Disease: Aortic Aneurysm PE

Risk factors include hypertension, high cholesterol, other vascular disease, a family history, smoking, gender (males > females), and age. ■ Exam demonstrates a pulsatile abdominal mass or abdominal bruits.

______ is the major cause of nonbacterial gastroenteritis, peaking in cooler months causing profuse diarrhea, vomiting and low grade fever.

Rotavirus

1° (Essential) Hypertension Tx

Rule out 2° causes of hypertension, particularly in younger patients. ■ Begin with lifestyle modifications (e.g., weight loss, smoking cessation, salt reduction). Weight loss is the single most effective lifestyle modification. ■ Diuretics (which are inexpensive and particularly effective in African-Americans), ACEIs, and β-blockers (which are beneficial for patients with CAD) have been shown to ↓ mortality in uncomplicated hypertension.

Acute cause of mitral regurgitation post MI

Rupture of chordae tendinae; sudden, rapid onset of dyspnea. Get echo and send to surgery

What is a buzz word for pneumococcal pneumonia? (aka streptococcus pneumonia)

Rust colored sputum

PE EKG

S in lead 1, Q in lead 3, and inverted T in lead 3 are characteristic findings

What are indications for hospitalization for PNA?

SBP <90 Pulse >140 O2 <90% Abscess or pleural effusion Metabolic abnormality Co-morbidities (CHF, renal failure, malignancy, DM, COPD) >65 yo Unreliable social situation

When do you use SCDs? When do you use Lovenox?

SCDs for surgical patients and low risk patients Lovenox for high risk medical patients with multiple VTE risk factors

Side effects of antiepileptics?

SJS LFTs (depakote) Cytopenias

How do you manage COPD?

SMOKING CESSATION <---most important! Anticholinergic inhalers (ipratropium and tiotropium) - best medicine SABAs - for short term relief Antibiotics when sick Supplemental O2 when sat is <88% Exercise Steroids for exacerbations Vaccinations (pneumovax, flu)

What are the symptoms and signs of asthma?

SOB, episodic wheezing, difficulty in breathing, chest tightness, and cough. Excess sputum production is common.

EKG diagnosis of an MI?

ST elevation - immediately T wave inversion - 6 hrs - 1 day Q wave - lasts forever

STEMI =

ST segment elevation myocardial infarction

LBBB + Elevated cardiac enzymes; treat like:

STEMI

MC cause of HF in the U.S.

STEMI (2nd MC cause if untreated or inadequately treated HTN)

A severe occlusion which causes transmural ischemia of the myocardium

STEMI; fibrinolytics are useful in these patients

What bacteria most commonly cause CAP?

STREP PNEUMO H. influenzae Moraxella catarrhalis Staph aureus Klebsiella pneumo

What is the SPHERE of lung cancer complications?

SVC syndrome (HA, AMS due to compression of SVC) Pancoast tumor (tumor of the lung apex, horner syndrome, shoulder pain) Horner syndrome (unilateral facial anhidrosis, ptosis, miosis) Endocrine (carcinoid syndrome: flushing, diarrhea, telangiectasias) Recurrent laryngeal nerve (hoarseness) Effusions (exudative)

How to confirm Lyme disease?

Same as HIV: 1. ELISA 2. Western Blot to confirm

What disease is a multiorgan disease of idiopathic cause characterized by noncaseating granulomatous (caseating = necrosis visible w/o microscope that appears like "turning to cheese") inflammation in affected organs?

Sarcoidosis

Sarcoidosis Etiology

Sarcoidosis is more common in African American women. It is idiopathic inflammatory disorder predominantly of the lungs but can affect most of the body.

Most common cause of hepatosplenomegaly in the world? Treatment?

Schistosomiasis Praziquantel

These conduction abnormalities need a pacemaker post MI

Second degree, Mobitz II Third degree (P/QRS dissociation)

Status Epilepticus

Seizure lasting more than 30 minutes or repetitive symptoms without lucid interval Tx: Benzo's (ativan), Fosphenytoin (faster than phenytoin), Barbiturate vs Propofol (intubated), Consider Toxin/Eclampsia if still refractory, Neuro ICU with EEG

Adverse effect of spironolactone:

Severe hyperkalemia; must carefully monitor K levels

What meds are for short acting control of asthma?

Short acting beta agonist (SABA) Ipratropium Bromide (anticholinergic) Systemic steroids

Mainstay of treatment for symptomatic COPD

Short acting beta agonists Muscarinic antagonists (ipratropium)

Correction of hyponatremia per day?

Should not exceed 9 mEq/L/day other sources say <0.5 mEq/L/day

CARDS study?

Showed that Type 2 diabetics with relatively normal LDL levels treated with Atorvastatin had 37% reduced risk of first CVD

Which cause of hypoxemia does NOT correct with supplemental oxygen?

Shunt

Restrictive Cardiomyopathy PE

Signs and symptoms of left-sided and right-sided heart failure occur, but symptoms of right-sided heart failure (JVD, peripheral edema) often predominate. Ascites.

Clinical features of HAP?

Similar to CAP

Papillary muscle dysfunction/rupture leads to this murmur:

Simulates severe mitral valve regurgitation (new systolic murmur - holosystolic)

What drugs are eluted from a drug eluting stent?

Sirolimus Paclitaxel

Sjögren's syndrome

Sjögren's syndrome (keratoconjunctivitis sicca) is a common ocular manifestation of rheumatoid arthritis.

Clinically aggressive lung cancer?

Small cell carcinoma

Behcet's disease

Small vessel vasculitis that causes recurrent mouth ulcers and ocular disease

What is the #1 risk factor for lung cancer?

Smoking

What are causes of COPD? What is the most common?

Smoking <----most common Pollutants Recurrent respiratory infections Eosinophilia Bronchial hyperresponsiveness Alpha-1-antitrypsin deficiency

Causes of gynecomastia?

Spironolactone Risperidone (and other aytpical antipsychotics) Large cell carcinoma (lung --> paraneoplastic) Cannabis

Diagnostic studies and findings for atypical PNA?

Sputum gram stain and culture - will not detect organism CBC - WBC will be slightly elevated CXR - segmental unilateral lower lung zone infiltrates or diffuse infiltrates

Mycoplasma Pneumonia

Sputum gram stain often show increased WBC with little or no bacteria.

Diagnostic studies for CAP?

Sputum gram stain or culture CXR <---- study of choice. will show lobar or segmental infiltrates, air bronchograms, and pleural effusions Procalcitonin - rises in response to a proinflammatory stimulus, especially if bacterial

What are three kinds of NSCLC? What percentage do they account for?

Squamous cell carcinoma (25-35%) Adenocarcinoma (35-40%) Large cell carcinoma

Tx: Multifocal Atrial Tachycardia (MAT)

Stabilize pt, 100% O2 by face

Clue features: Acute substernal chest pain that improves w rest

Stable Angina

Treatment for PSVT

Stable patient: vagal maneuvers (carotid massage, sink patient's head in water) Unstable patient: if non-responsive to vagal maneuvers; start IV adenosine or IV CCB (verapamil, diltiazem)

Stages of COPD?

Stage 1 - FEV1 >80% Stage 2 - FEV1 50-80% Stage 3 - FEV1 30-50% Stage 4 - FEV1 <30% **all of them have an FEV1/FVC <70-%

CKD Stages 1-5

Stage 1: kidney damage with normal GFR >90 Stage 2: kidney damage with GFR 60-89 Stage 3: GFR 30-59 Stage 4: severe, GFR 15-29 Stage 5: kidney failure, GFR less than 15

Valvular Disease: Aortic Dissection classification

Stanford classification—Stanford A aortic dissection involves ascending aorta; Stanford B is distal to left subclavian artery

Acute endocarditis causative agent? Subacute endocarditis (native valve) IV drug use endocarditis?

Staph aureus Strep Viridans (Staph aureus) Tricuspid valve --> murmur worse with inspiration

What are the most common pathogens that cause HAP?

Staph aureus Gram neg bacilli (E. coli) Psueudomas - most likely in the ICU and has worst prognosis

Acute endocarditis is caused by

Staphylococcus aureus , Streptococcus pneumoniae , Staphylococcus pyogenes, Neisseria gonorrhoeae

What is used to relieve chronic alveolitis in silicosis?

Steroids

Giant Cell Arteritis

Steroids immediately, do not wait for biopsy results. Give low dose aspirin too.

What is the MC cause of bacterial pneumonia in HIV+ patients?

Strep pneumo

What is the most common cause of PNA in all types?

Strep pneumo

What is the most common cause of community acquired pneumonia (CAP)?

Strep pneumonia

Subacute endocarditis caused by

Streptococcus viridans, Enterococcus, fungi, and Staphylococcus epidermidis

Takotsubo cardiomyopathy?

Stress induced cardiomyopathy A bulging out of the left ventricular apex with a hypercontractile base of the left ventricle is often noted.

Pulmonary hypertension Tx

Supplemental O2, anticoagulation (warfarin), vasodilators (IV prostacyclins) , and diuretics if symptoms of right-sided CHF are present. Treat underlying causes of 2° pulmonary hypertension.

Management of pneumoconiosis?

Supportive (oxygen, vaccinations, rehab) Steroids for chronic alveolitis in silicosis Smoking cessation (especially for asbestosis)

Tx for uncomplicated bronchitis?

Supportive measures (hydration, expectorants, analgesics, Beta agonists, cough suppressants)

Treatment for carcinoid tumors?

Surgical excision Octreotide - for symptoms

The only definitive treatment for aortic stenosis:

Surgical intervention - aortic valve replacement

Torsed Gonad Male

Swollen, Firm, high-riding testicle Tansverse ie *Loss of cremasteric reflex* Pain Control Urology consult before ultrasound Reduction

Gastroesophageal reflux disease

Symptomatic reflux of gastric contents into the esophagus, most commonly as a result of transient LES relaxation. Can be due to an incompetent LES, gastroparesis, or hiatal hernia.

Aortic Aneurysm Signs and Symptoms

Symptoms: *Syncope and chest pain*, Sudden onset of abdominal, back, or flank pain, hematuria, scrotal mass, femoral neuropathy Signs: *Pulsatile mass*, ecchymosis, abdominal bruits, tenderness, distal extremity ischemia

Aortic Dissection Signs and Symptoms

Symptoms: Abrupt onset, Chest or back pain, *Tearing or Ripping* which propagates, *Neuroo deficits* Signs: Asymmetric BP, shock vs HTN, New murmur, Tamponade, may mimic AMI and CVA

Difference between synchronized cardioversion and non-synchronized

Synchronized is used for pts w/ an arrhythmia w a pulse; non-synchronized is for pts with an arrhythmia (vtach, vfib) w/o a pulse

Differential diagnosis for seizure?

Syncope TIA Pseudoseizure Complicated migraine

CXR: fluid in aveolar walls (cloudy), Kerley B lines, effusions, dilated pulmonary veins:

Systolic CHF

Px: Increased RR, rales, rhonchi, wheezing, edema, JVD, S3 gallop, displaced apical impulse, cyanosis/hypoxic signs (clubbing)

Systolic CHF

Stereotypical pres cardiogenic pulmonary edema: Pt presents w worsening dyspnea on exertion, hx of hyperlipidemia, DM, smoking, CAD:

Systolic CHF

Things to think of w enlarged cardiac silhouette on CXR:

Systolic CHF, chronic valvular disease; [normal cardiac silhouette in diastolic CHF or acute cause]

Heart that can "relax, but not contract"

Systolic CHF; reduced LVEF

What physical findings are suggestive of pulmonary HTN?

Systolic ejection click and splitting/accentuation S2

PE findings with asthma?

Tachypnea Tachycardia Accessory muscle use Nasal flaring Tracheal tugging Abdominal breathing

What medications can you use to help dilate the ureter to help pass kidney stones?

Tamsulosin Non-DHP Calcium channel blockers

What is flomax?

Tamsulosin (alpha 1 antagonist specific for alpha receptors in the prostate/ureter)

Medications used to help ureters relax and kidney stones to pass?

Tamsulosin (alpha 1 antagonist specific for alpha receptors in the prostate/ureter) Calcium channel blockers (Non DHP) - verapamil, dilitiazem

MC cyanotic congenital heart disease:

Tetralogy of Fallot; in ToF, a ventricular septal defect is present, as is an obstruction to right ventricular outflow. Tx: Closure of ventricular septal defect and pulmonary valvulotomy

TIMI score? What does it indicate?

The TIMI score is a risk of having an MI or death in pt. with unstable angina or NSTEMI within 14 days. AMERICA Age > 65 Markers (elevated cardiac enzymes) EKG (ST depression 0.5 mm) Risk factors (3+ risk factors for CAD) Ischemia (2+ anginal events in last 24 hours CAD (prior stenosis of 50% or more) Aspirin use in last 7 days The TIMI stratifies which patients will benefit with a GIIb/IIIa inhibitor + early PCI

Complicated pleural effusion

The criteria for a complicated parapneumonic effusion include the presence of pus or Gram stain-positive pleural fluid or a pleural fluid pH <7.0. Appropriate management includes chest tube drainage (tube thoracostomy).

Patients with systolic heart failure have this:

The left ventricular ejection fraction is reduced

Gastroenteritis Tx

The most important next step is rehydration. Mild cases: oral fluids Severe cases: IV fluids

Restrictive Cardiomyopathy Tx

Therapeutic options are limited and generally are palliative only. Medical treatment includes cautious use of diuretics for fluid overload, vasodilators to ↓ filling pressure, and anticoagulation if not contraindicated.

What is therapy for improving RV contractility?

There are no oral inotropic agents that are appropriate for routine use in patients with cor pulmonale. Digoxin is not recommended. An IV inotropic agent is indicated if the patient is in shock.

What is the pathogenesis of asthma?

There is an initial inflammatory event which involves the degranulation of presensitized mast cells. Mast cells then release inflammatory mediators (histamines, cytokines, leukotrienes, and platelet activating factors). This all leads to edema, mucus production, and bronchospasms.

What do you need to administer before giving glucose with hypoglycemic patients if history of alcohol abuse or malnutrition?

Thiamine

Treatment for calcium oxalate stones?

Thiazides (decreases calcium in the urine), citrate (binds calcium)

Esophageal varices

This is an enlarged, tortuous vein or, occasionally, an artery. Occur most frequently in the distal veins of the esophagus. Associated w/cirrhosis of the liver or conditions that cause chronic obstruction of drainage from esophageal veins into the portal veins.

Mallory-Weiss tear

This is not a cause of dysphagia, although Mallory-Weiss tear is clearly an esophageal disorder. It presents as sudden upper gastrointestinal bleeding with violent retching and vomiting of any cause. There may be either hematemesis or black stool.

Functional Residual Capacity:

This measures the *amount of air in your lungs at the end of a normal exhaled breath.*

Management of pleural effusion?

Thoracentesis is therapeutic and diagnostic Transudative effusions resolve when underlying condition is treated Malignant effusions may require drainage and pleurodesis (doxy and talc are most commonly used irritants) Empyema requires drainage and antibiotics

DVT

Thrombus embedded in one of the major deep veins of the lower legs,thighs, or pelvis. Unilateral pain, *edema*, tenderness. massive clots may cause ischemia (compression) or arterial spasm Prior DVT, Malignancy, age >40, obesity, estrogen, pregnancy, trauma, catheters, orthopedic surgery are all risk factors

EKG criteria for left atrial and right atrial enlargement?

To determine LAE and RAE, look at Lead II and lead V1: LAE: - In lead II, P wave duration >0.12 sec (3 small boxes) (NOTCHED P WAVES) - In lead V1, P wave with a broad (>0.04 sec or 1 small square) and deeply NEGATIVE (>1 mm) terminal part in V1 RAE: - P >2.5 mm in II - P >1.5 mm in V1 (POSITIVE DEFLECTION).

What is the most common etiology leading to COPD?

Tobacco Smoking. This destroys the elastin fibers.

Toxicology

Toxin and Antidote Salicylates- Sodium Bicarbonate Acetaminophen- N-acetyl cystine Opiates- Narcan Carbon Monoxide- O2, Hyperbaric oxygen Organophosphates- Atropine, 2-PAM Beta Blockers- Glucagon TCA's- Sodium Bicarbonate

HIV opportunistic infections?

Toxoplasmosis Cryptococcus neoformans PML CMV Cryptosporidium (diarrhea)

Initial diagnostic test of choice in a patient with hemochromatosis?

Transferrin

Crohn's Disease

Transmural inflammation of GI tract (mouth to anus). Commonly involves ileum.

Types of pericardial effusions:

Transudative: Pressure cause - CHF, fluid overload, hypoalbuminemia Exudative: Chest injury or trauma Serosanguineous: Infiltration of the myocardium - particularly TB, neoplasm Frank blood: coronary artery rupture, LV free wall bleed (post MI), ruptured aortic aneurysm, penetrative trauma

Management of secondary pulmonary HTN?

Treat underlying disorder in addition to treatments for primary HTN

Pericarditis

Treatment 1. Most cases are self-limited and resolve in 2 to 6 weeks. 2. Treat the underlying cause if known. *PE: May reveal friction rub, chest pain*

Endocarditis Tx

Treatment = long-term (4-6 weeks) IV antibiotics

Digitalis Toxicity Syndrome

Triggered by hypokalemia (happens from pt not getting enough K+ or taking too much dig); sx: hypersalivation, n+v, fatigue, visual halos; severe - bradyarrhythmias Tx: digoxin-immune fab (digibind); tx brady w atropine, lido, mg2+

Asthma etiology is unknown. There is an association with atopic disorders and obesity. Causes of acute exacerbations of symptoms include:

Triggers include pollens, house dust, molds, occkroaches, cats, dogs, cold air, viral infections, tobacco smoke, medications (,B-blockers, aspirin), exercise, and GERD.

Myocarditis in South and Central America is commonly caused by

Trypanosoma cruzi (Chagas' disease) and, in these cases, may be associated with achalasia.

Torsed Gonad

Twisting of ovary, testicle, or fallopian tube around is vascular pedicle Hx of enlarged ovary, recent physical activity, Sudden, severe, unilateral pain, *irreversible ischemia after about 6 hours*

You must have this many lipid panels to diagnose hyperlipidemia

Two

Chronic Hepatitis Tx

Tx: Alpha-Interferon and ribavirin

Tx: WPW syndrome

Tx: acute vagal maneuvers, IV adenosine; IV procainamide in refractive cases Long-term: radiofrequency ablation can be curative

Gastritis

Type B (90%): Occurs in the antrum and may be caused by NSAID use or *H. pylori infection*. Often asymptomatic, but associated with ↑ risk of PUD and gastric cancer.

Panacinar emphysema:

Typically affects the lower lungs.

Centrilobar emphysema:

Typically affects the upper lungs.

hyperlipidemia screening

USPSTF men > 35 women > 45 w/ CAD risk factors

Does UC or Crohn's usually involve the rectum?

Ulcerative Colitis

Which has the higher risk of colon cancer, Ulcerative Colitis or Crohn's?

Ulcerative Colitis

With Ulcerative Colitis or Crohn's when does the colon shorten and have loss of haustra seen on barium enema exam?

Ulcerative Colitis

What do I have? Bloody diarrhea x4 weeks, colicky abdominal pain, tenesmus, fever, weight loss. On barium enema there is a "stove pipe" appearance to my colon.

Ulcerative Colitis (UC)

Clue features: Acute substernal chest pain that does not improve w rest

Unstable Angina

Vtach

Unstable- cardioversion stable- meds or cardioversion

What does melena (black tar) indicates?

Upper GI bleed.

Cause of ammonium magnesium phosphate kidney stones (Struvite)? Treatment?

Urease + organisms (proteus, klebsiella) can convert urea --> ammonia (urine alkalinazation) Can from Staghorn calculi Treatment is treating the underlying infection

Hypertension Urgency & Emergency

Urgency: Diastolic BP >120, Systolic >140, No end organ damage Emergency: *End organ Damage*, CNS, CV, Renal, AMI *Increased ICP, HTN, Wide PP, bradycardia,irregular respirations* Pulmonary Edema, Aortic dissection, Eclampsia

How to diagnose Legionella?

Urine antigen test

Prinzmetal's angina typically responds to nitrates and calcium channel blockers.

Use of a beta blocker such as propranolol is contraindicated in Prinzmetal's angina.

Chemical stress tests:

Used in pts who cannot perform the exercise stress test or in pts who have pre-existing anomalies; Dobutamine echo stress test: + inotrope, increases HR. Positive = decreased cardiac wall movement Dipyridamole-Thallium stress test (scintigraphy) [all + stress tests get angiograms]

What are the three idiopathic pulmonary fibrosis types?

Usual interstitial pneumonia Respiratory bronchiolitis-associated interstitial lung diseae Acute interstitial pneumonitis

Sclerotherapy involves the injection of an irritating solution into these to promote an inflammatory response, scarring, and obliteration of the lumen

Varicose veins

Virchow's Triad

Venostasis Hypercoagulability Vessel Wall injury

Sx of RV dysfunction:

Venous stasis, hepatomegaly, ascites, etc.

Most common cause of death in CHF How to prevent?

Ventricular fibrillation Implantable cardioverter defrillator indicated if EF<35% and optimal medical management has been maximized to prevent sudden cardiac death.

Stereotypical pres cardiogenic pulm edema: Pt is very tachycardic, presents w anxiety, chest pain, and SOB, and has EKG that shows wide QRS complexes w no discernible P or T waves:

Ventricular tachycardia

Causes of sudden cardiac death in MI pts.?

Ventricular tachycardia Ventricular fibrillation **within 1 hour of MI**

Tachycardia with very wide and bizarre QRS complexes w/no distinctly discernable P or T waves

Ventricular tachycardia; likely will require cardioversion - stable pt gets IV amiodarone (lidocaine and procainamide if refractory)

Preventive therapies for cluster headache lasting more than 2 months

Verapamil 240-960mg given in 3 daily doses

Cause of recurrent UTIs in young children?

Vesicoureteral reflux

Most common causes of acute hepatitis are what?

Viral causes. Toxins like alcohol are the second most common cause. Other causes: Tylenol and toxic mushroom ingestion

Causes of transaminitis (acutely severe elevated liver enzymes?

Viral hepatitis Durg induced hepatitis (Toxic (tylenol) Shock liver (ischemic damage) Autoimmune (Wilsons, Hemochromotosis, AIH, Alpha 1 antitrypsin)

Pneumonia Tx

Viral pneumonia is self -limited and only requires supportive care; bacterial and fungal pneumonias require antibiotics (started as broad coverage and changed to pathogen -specific therapy as culture results become available).

Risk factors for pulmonary embolism?

Virchow's triad: 1. Stasis 2. endothelial injury 3. Hypercoagulable states

Mechanism of vomiting --> hypokalemic hypochloriemic metabolic alkalosis?

Vomiting results in loss of H+ and Cl- --> hypochloremic metabolic alkalosis Volume loss from vomiting --> RAAS system --> dumps K+ and H+ --> hypokalemic, metabolic alkalosis "Contraction alkalosis refers to the fact that there is the same amount of HCO3- in a smaller amount of fluid

Aortic Aneurysm

Weakened and bulging area in the aorta, true aneurysm (all 3 layers), Genetic, structural, metabolic milieu Age >60, atherosclerosis, HTN, smoking, lipids, other vascular disease *97% Infrarenal*

pulmonary embolism ____ criteria

Well's criteria: > 6 = high risk < 2 = low risk HR > 100 bpm (1.5 pts) hemoptysis (1 pt) hx of VTE (1.5 pts) malignancy (1 pt) leg swelling, pain w/ deep veins (3 pts) immobilization w/n prior 4 wks (1.5 pts) alt diagnoses less likely (3 pts)

Wells criteria and probability of DVT? Diagnostic test based on Wells score?

Wells score of: >3 - high probability DVT - doppler ultrasound 1-2 - moderate probability of DVT - doppler ultrasound 0 - low probability DVT - D-dimer assay (if positive --> doppler ultrasound)

Second Degree AV Block, Mobitz Type I is typically called:

Wenckebach (1,2,3 drop - you've got a Wenckebach)

When is coronary angiography indicated in the setting of angina?

When medications are maximally dosed but symptoms persist.

Tachycardia, wide complex QRS with sloping "delta wave"

Wolff-Parkinson-White Syndrome

Early excitation due to accessory pathways between the atria and ventricles

Wolff-Parkinson-White syndrome

This causes a delta wave that is pathognomonic for this condition caused by early depolarization of the left ventricle from an accessory pathway (bundle of Kent) that bypasses normal physiological delay of the AV node

Wolff-Parkinson-White syndrome

Aortic Dissection workup and Treatment

Work up: CXR, CTA (stable pt), TEE (unstable), Pain control, Control shear forces if hypertensive (esmolol, labetolol, 60 bpm goal HR)-beta blocker plus nitroprusside, Fluids for hypotension, Prepare PRBC, Consult surgeon. Need to Type and Cross 10 units of PRBC

HTN Work up and Treatment

Work-up: CBC, CMP (creatinine), UA, EKG, Trop, CXR, head CT (based on complaint) Exotica Tx: Arrange F/U in majority of pts Lower SBP 25% in 24 hours Labetolol (IV form is fast onset, oral is spontaneous) Nitrates, Hydralazine, ?benzos

Acute pericarditis pain

Worse with inspiration Better w/ leaning forward Friction rub Diffuse ST elevation

Who is the pneumococcal polysaccharide vaccine recommended for?

Young and old, sick, sickle cell, smokers, no spleen and liver disease

Clinical features of CF?

Young patient with hx of chronic lung disease, pancreatitis, infertility Cough, excess sputum Decreased exercise tolerance Sinus pain Purulent nasal discharge Steatorrhea Diarrhea Abdominal pain Clubbing of fingers, increased AP diameter, apical crackles

Jugular venous pulses?

a wave - atrial contraction c wave - ventricular contraction against a closed tricuspid valve (tricuspid valve bulging back into right atrium) x descent - atrial relaxation v wave - atrial filling (diastole) y descent - atrial emptying

Secondary causes of hyperlipidemia

a. Endocrine disorders-hypothyroidism, DM, Cushing's syndrome b. Renal disorders-nephrotic syndrome, uremia c. Chronic liver disease d. Medications-glucocorticoids, estrogen, thiazide diuretics, 8-blockers e. Pregnancy

case: > 60 yo Caucasian M pt who smokes w/ atherosclerosis that presents w/ sudden onset of severe back or abd tearing/ripping pain, hypotension, nausea/vomiting, and pulsatile abd mass

abdominal aortic aneurysm (AAA) note: normal abd aorta diameter is ~2cm AAA > 3 cm most discovered incidentally may also have... Cullen (periumbilical ecchymosis) grey-turner sign (flank ecchymosis)

Orthopnea:

ability to breathe only in an upright position

what is the preferred therapy for prevention of recurrent preexcited atrial fibrillation?

ablation of accessory pathway

Anal Fistula

abnormal inflammatory tract originating from infected anal gland

cheyene-stoke

abnormal pattern of breathing characterized by progressively deeper and sometimes father breathing

Dysphagia for solids/liquids should alert you for motility disorders such as what?

achalasia or collagen vascular disorders

what is the organism in tb

acid fast bacillus mycobacterium tuberculosis

ventricular tachycardia Tx

acute -lidocaine -procainamide -amiodarone -empiric mg unstable -shock now! -w/out a pulse > unsynchronized cardioversion starting at 100 J -w/ a pulse > synchronized cardioversion -if recurrs > load w/ antiarrhythmic stable -amiodarone -cardiologist -tx underlying cause (ischemia, low K/Mg/Ca, hyperthyroid, digoxin toxicity) long term -ICD (defibrillator) -anti-arrhythmics -catheter ablation torsades de point -IV mg

pericarditis Sx

acute sharp pleuritic substernal radiating chest pain relieved by SITTING UPRIGHT & LEANING FWD cardiac FRICTION RUB slowly progressive -dyspnea -fatigue -weakness -edema -hepatomegaly -ascites maybe fever

Wheezing and rhonchi are frequent findings in adults with________________.

acute bronchitis and do not represent adult-onset asthma in most cases.

takotsubo DDx

acute coronary syndrome cocaine-related ACS myocarditis pheochromocytoma

aortic regurgitation etiology

acute most common cause -endocarditis chronic most common cause -rheumatic heart disease

Acute respiratory distress syndrome (ARDS) criteria?

acute onset hypoxemia Diffuse alveolar damage --> increased capillary permeability --> protein rich leakage into alveoli and noncardiogenic pulmonary edema --> formation of intraalveorlar hyaline membrance Noncardiogenic pulmonary edema --> normal PCWP ARDS is the presence of pulmonary edema in the absence of volume overload or depressed left ventricular function. (non cardiac source) 1. Acute onset 2. Bilateral alveolar infiltrates on CXR 3. Pulmonary capillary wedge pressure < 18 4. PaO2:FIO2 ratio < 200 Tx: Mechanical ventilation with PEEP

Valvular Disease: Aortic Dissection H/P =

acute, "ripping" chest pain, syncope; decreased peripheral pulses, normal or increased blood pressure

Treatment of Intrinsic Renal Failure

adequate circulating volume must be restored first, as hypovolemia potentiates and exacerbates all forms of renal failure

how do u treat botulism toxin

administer toxoid, do not wait for lab results, but gather specimens

NSTEMI/unstable angina Tx

admit and monitor best rest oxygen serial EKG, troponins ASA, BB, NTG, ACEI, anticoagulation cardiac cath plavix (clopidogrel) following cath

Ascending cholangitis is an emergency treated how?

admit to ICU, give broad spectrum antibiotics, consult surgery

atrial fibrillation why do pts need to be on anticoagulation?

afib > blood stagnate > thrombus > stroke (embolic event)

3rd degree/complete heart block etiology

aging ischemia (acute MI) post-surgical (esp valve) congenital electrolytes digoxin toxicity high vagal tone

dilated cardiomyopathy Tx

alcohol abstinence tx underlying dz CHF & takotsubo requires supportive tx

3rd degree/complete heart block Tx

almost always requires pacemaker

Side effects of Prazosin? Mechanism of Prazosin?

alpha 1 antagonist orthostatic hypotension

MOA of clonidine? Side effects?

alpha 2 agonist Dry mouth Rebound hypertension with sudden discontinuation b/c of downregulated alpha 2 receptors

how do u treat systemic candidiasis

amphoterici b, fluconazole

treatment for cryptococcosis

amphotericin b with flucystosine for two weeks, followed by oral fluconazole. duration depends on follow up csf cultures

Restrictive Cardiomyopathy caused by infiltrative disease

amyloidosis, sarcoidosis, hemochromatosis) or by scarring and fi brosis (2° to radiation or doxorubicin).

Extreme, generalized edema:

anasarca

____abnormal dilation of an artery

aneurysm * >1.5X normal size *true aneurysms involve all layers of the arterial wall-intima, media, adventitia

aortic stenosis Sx

angina syncope CHF thready pulses displace API harsh, medium pitched midsystolic ejection murmur heard best at RUSB, radiates to neck

ARB

angiotensin II receptor blocker

ACEI

angiotensin converting enzyme inhibitor

Test of choice to diagnose hemorrhoids

anoscopy

Pts with aortic stenosis and aortic regurgitation need this prior to dental procedures:

antibiotic prophylaxis

Define stroke

any disease process that interrupts blood flow to brain

Where is Crohn's Disease located in the intestines?

anywhere from mouth to anus; terminal ileum most common

so if in systole, can have ___ stenosis and ___ regurg

aortic and pulmonary stenosis mitral and tricuspid regurgitation

high-pitched, blowing, early diastolic decresendo murmur at the 2nd-4th LICS/LUSB and radiating to the apex and RSB accentuated w/ leaning fwd, expiration, & hand grip

aortic regurgitation

most common valvular dz

aortic stenosis

harsh midsystolic ejection murmur heard best at RUSB and 2nd RICS, radiates to neck, subclavian a, and LSB accentuated by leaning fwd decreased w/ standing and valsalva

aortic stenosis murmur (most common valvular dz in US)

where does reactivation tb show up in the lungs

apical and posterior

what is used as a prognostic tool for PCP in hiv patients

arterial po2. po2<70 , pts need prednisone

Examples of lower GI bleeds

arteriovenous malformation, diverticulosis, colon cancer, hemorrhoids

Signs and symptoms of Guillian Barre Syndrome

ascending symmetric weakness and loss of DTR's, proximal muscles affected more than distal. Sensory abnormalities, pain, tachycardia, sweating, impaired pulmonary function, paralytic ileus

Causes of respiratory acidosis

aspiration, atelectasis, abdominal distention, drug overdose, CNS disease, pleural/lung disease, trauma Tx: spirometry, deep breathing, pain control, ventilation

in pts w/ myocardial infarction, which 3 drugs have been shown to decrease mortality?

aspirin beta blockers ace-inhibitors

dilated cardiomyopathy etiology

associated w/ reduced strength of ventricular contraction, resulting in dilation of left ventricle genetic abnormalities (25-30%) excessive alcohol postpartum chemotherapy toxicity endocrinepathies myocarditis idiopathic takotsubo (stress-induced/broken heart)

Convulsive (grand-mal or tonic-clonic) seizures

associated with postictal obtundation and confusion lasting minutes to hours. Tonic-clonic muscle contractions of all extremities, LOC, incontinence, flaccid/unconscious after attack

ventricular tachycardia

asymptomatic dizziness syncope

DVT Sx

asymptomatic swelling of involved area heat/redness over site homan's sign -unreliable -pain in calf w/ forced dorsiflexion of foot

hyperlipidemia Sx

asymptomatic xanthomas (esp near eyelids) premature arcus senilis

Solitary pulmonary nodules are

asymptomatic and are discovered as an incidental finding.

peripheral arterial disease etiology

atherosclerosis thrombotic dz -trauma -hypovolemia -inflammatory arteritis -polycythemia -dehydration -repeated arterial punctures -hypercoaguble states

long-standing mitral stenosis can lead to left atrial dilation which is a prominent risk factor for ___

atrial fibrillation thromboembolism

mid-to-late diastolic rumble at apex

austin flint murmur

hypertrophic cardiomyopathy is passed on through what genetic inheritance pattern?

autosomal dominant pattern

premature atrial contractions Tx

avoid offending agents tx underlying dz BB cardiology

Paroxysmal Nocturnal Dyspnea:

awakening from sleep with SOB and needing to be upright to achieve comfort

what is the treatment for chlamydia

azithromycin po x1dose, or oral doxy x 7days

which valvular disorder? -low-pitched diastolic murmur heard best over the apex -early high-pitched, blowing diastolic murmur heard best over the left sternal border -wide pulse pressure -Corrigan's pulse a) aortic stenosis b) aortic regurgitation c) mitral stenosis d) tricuspid stenosis

b) aortic regurgitations

Treatment for esophageal varices

banding

Causes of Subarachnoid hemorrhagic stroke are what?

berry aneurysm rupture, vascular malformation rupture

Asterixis

bilateral but asynchronous flapping motions of outstretched, dorsiflexed hands

how can u diagnose botulism

bioassay positive for toxin in serum, gastric contents or stool

Melena

black, tarry, malodorous stool that occurs when blood has been in digestive tract for awhile

what do ARBs do?

block interaction of angiotensin II beneficial in DM and CKD does not increase bradykinin; no cough

how is MAC (mycobacterium avium) diagnosed?

blood cultures of mycobacteria

which disease has the hallark of descending flaccid paralysis

botulism!

Treatment for Diverticulosis

bowel rest, morphine for pain, fluids/electrolyte replacement, antibiotics (Metronidazole and Cirpo or Levo)

Signs and symptoms of non-convulsive (absence) seizures

brief, often unnoticeable, episodes of impaired consciousness lasting seconds occurring up to 100 times per day. Present as staring spells.

Hematochezia

bright, red blood per rectum

peripheral arterial disease Sx of occulsion depend on the artery?? buttock/hip- thigh- upper calf- lower calf- foot-

buttock/hip-aortoiliac a thigh-common femoral a upper calf-superifical femoral a lower calf-popliteal a foot-peroneal a

which organism has hyphae and psuedohyphae

candida albicans

in setting of aortic dissection what is the most common cause of death?

cardiac tamponade

pulsus paradoxus can be indicative of what?

cardiac tamponade constrictive pericarditis chronic sleep apnea croup obstructive lung dz vena cava thrombus pulmonary embolism

hypertension complications

cardiac: -each increase in 29 mm Hg SBP or 10 mm Hg DBP doubles the risk of CV complications -CAD -LVH -HF -Afib neurologic: -TIA/CVA -ruptured aneurysms -vascular dementia retinopathy: -stage I = arteriolar narrowing -stage II = copper-wiring, AV nicking -stage III = hemorrhages & exudates -stage IV = papilledema vascular: -aortic dissectioin -aortic aneurysm renal: -proteinuria -renal failure

Jones Major Criteria

carditis polyarthritis chorea subcutaneous nodules erythema marginatum

Intrinsic Renal Failure (ATN)

caused by severe and prolonged prerenal etiologies. Nephrotoxins 2nd most common cause (NSAIDs, contrast).

Celiac disease

causes B12, folate, and iron deficiency

Bell's Palsy

causes cranial nerve 7 dysfunction

at what point is an hiv infection considered progressed to aids

cd4<200

what is the most common cns mass lesion in aids pts

cerebral toxoplasmosis

2nd degree block

characterized by intermittent AV node conduction-some impulses reach the ventricles whereas others are blocked thereby causing a "grouped beating"

hypertension PE

check BP in both arms funduscopic exam cardiac (LVH, murmurs) vascular (bruits, radial-femoral delay) abd (masses or bruits) neuro exam

prinzmetal/variant angina Sx

chest pain occurs at rest typically early morning may wake pt up from sleep no correlation w/ exertion/emotional stress *often affects women < 50 yo *may be associated with other vasospastic disease -migraines -Raynaud's

If serum alkaline phosphatase levels are elevated greater than 3 times normal, think what?

cholestasis

___ aka Syndenham's or St Vitus Dance; abrupt, purposeless movements

chorea

hypertension secondary causes

chronic kidney dz renal artery stenosis coarctation of the aorta hyperaldosteronism cushing's syndrome pheochromocytoma (catecholamine producing adrenal tumor) obstructive sleep apnea

___ is characterized by loss of wall tension in veins, which results in stasis of venous blood and often is associated w/ hx of DVT, leg injury, or varicose veins

chronic venous insufficiency

How is chronic bronchitis clinical diagnosis?

clinical diagnosis defined by excessive secretion of bronchial mucus and is manifested by daily productive cough for 3 months or more in at least 2 consecutive years.

what can one use if allergic to aspirin?

clopidogrel

Endoscopy findings for Crohn's Disease

cobblestoning (late), skip lesions, edema, deep linear ulcers, fissures

Where is Ulcerative Colitis located in the intestines?

colon, terminal ileum only 10% of time

premature atrial contractions Sx

common in all ages often in absence of heart dz palpitations intermittent "sinking" or "fluttering" of chest

AV septal defect characeristics

common in down syndrome murmur variable w/ degree of defect maybe cyanosis infants present w/ CHF

ventricular fibrillation etiology

commonly seen in pts w/ severe ischemic dz digoxin/quinidine toxcity hypothermia chest trauma hypokalemia hyperkalemia mechanical stimulation

3rd degree block aka

complete heart block; no conduction btw the atria and the ventricles

mobitz (type 2 2nd degree block)

complete interruption in AV conduction with resulting AV dissociation; can be symptomatic and often progresses to complete heart block

What is the usual cause of a paraesophageal hernia?

complication of surgical dissection

Endocarditis more common in patients with

congenital heart defects, intravenous drug abuse , or prosthetic valves.

___ is a clinical syndrome characterized by abnormal retention of water and sodium resulting in venous congestion which causes dyspnea and edema

congestive heart failure note: affects... myocardial contractility structural integrity of the valves preload/afterload of the ventricle heart rate cardiac output left atrial pressure

CHADSVAS stands for...

congestive heart failure hypertension age > 65 diabetes CVA/TIA (2 pts) vascular dz age > 75 (2 pts) sex (female)

cardiovascular topic list

congestive heart failure hypertension murmurs valvular heart dz coronary heart dz cardiac arrhythmias myocarditis endocarditis pericarditis cardiomyopathy hyperlipidemia rheumatic heart disease vascular dz

mobitz (type 2 2nd degree block) Dx

constant PR interval w/ randomly dropped ventricular beats blocks usually located below AV node in Bundle of His

patent ductus arteriosus murmur

continuous (machinery) murmur

Endoscopy findings for Ulcerative Colitis

continuous involvement, friable hyperemic mucosa, shallow ulcers, pseudopolpys (chronic)

what does nitroglycerin do?

coronary artery dilation/vascular smook muscle relaxation, decreases preload, decreases myocardial O2 demand

"water hammer" = rapid rise/fall or distention/collapse

corrigan's pulse

ACEI major adverse effect

cough

Symptoms of Crohn's

crampy abdominal pain, hematochezia, wt loss, less diarrhea, symptoms of complications such as obstruction

tetralogy of fallot murmur

crescendo-decrescendo holosystolic murmur left sternal border (LSB) radiates to back

in which disease seen in aids/hiv does the ICP increase?

cryptococcal meningitis.

what should always be on the ddx of an aids pt a fever and headache?

cryptococcosis

which organism do pigeon keepers need to worry about?

cryptococcus neoformans

tetralogy of fallot Sx

cyanosis clubbing loud S2 tet (hypercyanotic) spells

pulmonary atresia Sx

cyanosis w/ tachynpea at birth tachypea w/out dyspnea hyperdynamic apical impulse single S1 and S2

transposition of the great vessels Sx

cyanosis* of newborn tachypnea w/out dyspnea CHF (w/ large VSD) poor feeding (w/ large VSD) single loud S2 absent LE pulses (w/ aortic arch obstruct)

heart murmurs etiology

cyanotic: tetralogy of fallot pulmonary atresia hypoplastic left heart syndrome transposition of the great vessels noncyanotic: atrial septal defect ventricular septal defect av septal defect patent ductus arteriosus coaraction of the aorta

pulmonary embolism Dx

d-dimer CT chest-test of choice V/Q scan

when cd4 count falls below 100, and pts have positive tooplasma serology, what do u use for ppx

daily dosing tmp-smx

head bobbing w/ systole

de Musset sign

Myocardial Infarction:

death of myocardial tissue due to sudden rupture of plaque with thrombus build up.

aortic regurgitation Tx (Rx objective)

decrease afterload

what do BBs do?

decrease heart rate decrease cardiac output reduce mortality after MI caution in pts w/ DM or pulm dz

what does beta blockers do?

decrease myocardial O2 demand decrease ventricular fibrillation note: IV indications... tachydysrhythmias intractable HTN

what does morphine do?

decrease preload/afterload decreases sympathetic activity note: no mortality benefit

constrictive cardiomyopathy Sx

decreased exercise tolerance right-sided congestive heart failure (advanced) often pulmonary HTN

what does heparin do?

decreases DVT, reinfarction, stroke, LV thrombus, reocclusion

what does aspirin do?

decreases mortality, infarct size, reinfarction rate

angina pectoris Sx

deep pressure-like pain substernal region, may radiate to jaw, neck, or arm transient, lasts 2-30 min SOB precipitated by physical exertion or emotional stress responsive to rest responsive to sublingual nitroglycerin PE often normal

The most common cause of a heart murmur is a

defective heart valve. Stenosis Regurgitation

IgE:

defense *against parasitic worms*, present in allergic reactions.

ventricular tachycardia

defined as 3 or more consecutive PVCs at a rate faster than 100 bpm. rate 150-250

pulmonary atresia murmur

depends on presence of tricuspid regurgitation

ABCD2

determine the risk for stroke in the days following a transient ischemic attack. Age >60 Blood pressure >140/90 Clinical features (unilateral weakness = 2 pts) Duration of TIA (> 60 minutes = 2 pts) Diabetes

What do you use in conjunction with abortive therapies for migraine to reduce risk of early headache recurrence?

dexamethasone 10-25 mg IV/IM

what is an AFB stain used for?

diagnosis of tb

aortic and pulmonic regurgitation occurs during ___

diastole

acute heart failure exacerbating factors

dietary indiscretion medical nonadherence myocardial ischemia or infarction renal failure (increases preload) hypertensive crises (increases left-sided afterload) drugs (BB, CCB, NSAIDs, TZDs) chemo (anthracyclines, trastuzumab) toxins (EtOH) arrhythmias valvular dysfunction (mitral/aortic regurg) COPD/PE (increases right-sided afterload) anemia systemic infection thyroid dz

coarctation of aorta pathognomonic PE

difference btw arterial pulses and blood pressure in the upper extremities and lower extremities

Caput Medusae

dilated abdominal veins

most common type of cardiomyopathy?

dilated cardiomyopathy

cardiomyopathy types?

dilated cardiomyopathy hypertrophic cardiomyopathy constrictive cardiomyopathy

aortic dissection etiology/risk factors

disease process in which the intima of the aorta tears and allows blood entry into the media of the aorta -*HTN -*Marfan syndrome -connective tissue dz -bicuspid aortic valve -trauma -aortic manipulation -catheter injury -*increased age

____ occurs when a defect in the intima allows blood to enter btw the layers of the wall

dissection

constrictive cardiomyopathy Tx

diurectics may help

peripheral arterial disease Dx

doppler flow studies ankle/brachial index (ABI) CT/MRA

what is treatment for non severe cases of lyme disease

doxy 100 mg, bid, amoxicillin 500mg tid, cefuroxime 500mg bid, emycin 250mg qid

DVT Dx

duplex ultrasound-preferred venography-definitive

systolic murmur heard over femoral a when compressed

duroziez sign

hypertrophic cardiomyopathy Sx

dyspnea angina syncope arrhythmias asymptomatic SUDDEN CARDIAC DEATH may be initial presentation ( < 30 yo, 2-3%) sustained PMI triple apical impulse loud S4 gallop variable systolic murmur bisferiens carotid pulse jugular venous pulsations w/ "a" wave

pulmonary embolism Sx

dyspnea pleuritis nonpleuritic chest pain anxiety cough syncope hypoxemia tachypnea tachycardia hemoptysis diaphoresis low-grade fever

mitral stenosis Sx

dyspnea on exertion hemoptysis *opening snap, diastolic apical murmur

IgM:

early stage secretion, *B cell surface receptor.*

Restrictive Cardiomyopathy Dx

echocardiography is key to diagnosis and reveals rapid early filling with a normal or near-normal EF.

premature atrial contractions Dx

ectopic P wave -appears sooner than expected -has a different shape and direction -may or may not be conducted through the AV node

systolic failure is ____

ejection problem

pericarditis Dx

elevated WBC CXR -normal or water bottle shaped enlarged cardiac silhouette -cardiac effusion EKG -diffuse ST elevation -PR depressions pericardiocentesis/bx -if tuberculosis is suspected

hyperlipidemia... ____ LDL ____ HDL are ____ ____ TG

elevated low density lipids elevated high density lipids are protective elevated triglycerides *increased risk of CAD

myocarditis Dx

elevated markers echocardiogram -cardiomegaly -contractile dysfunction myocardial bx -inflammatory pattern MRI -role in diagnosis

endocarditis Tx

empiric antibiotics -staphylococcus -streptococci -enterococci *gentamycin w/ ceftriaxone or vancomycin prophylaxsis -dental work -prosthetic cardiac valves -previous endocarditis valve replacement anticoagulation contraindicated

Test of choice to diagnose UGI bleed?

endoscopy

non-pruritic rash affecting the trunk and extremities, face is spared

erythema marginatum

Examples of UGI bleeds

esophageal varices, mallory-weiss tear, PUD

hypertension exacerbating factors

excessive alcohol cigarettes lack of exercise, sedentary lifestyle polycythemia use of NSAIDs low potassium intake high sodium intake sleep apnea renal insufficiency diabetes CHF CAD TIA/CVA males AA

congestive heart failure left-sided Sx

exertional pulmonary vascular congestion -low cardiac output -elevated pulmonary venous pressure dyspnea exertional orthopnea paroxysmal noctural dyspnea cough fatigue exercise intolerance basilar rales gallops *most of the time CHF is left-sided failure

atrial septal defect Sx

failure to thrive fatigability RV heave wide fixed split S2

hyperlipidemia Dx

fasting complete lipid profile LDL < 100 = optimal >160 = high > 190 = very high HDL > 60 = protective < 40 = at risk total cholesterol < 200 = desirable > 239 = high

How are Hep A and E transmitted?

fectal-oral usually from improper food handling

Jones Minor Criteria

fever arthralgia elevated ESR or CRP prolonged PR interval on EKG previous rheumatic fever

Endocarditis H/P

fever (very high in acute form), chills, night sweats, fatigue, arthralgias; possible new murmur; small, tender *nodules on finger and toe pads* (i.e., Osler's nodes ); peripheral petechiae (i.e., Janeway lesions ), subungual petechiae (i.e., splinter hemorrhages), retinal hemorrhages (i.e., Roth's spots)

Charcot's Triad

fever, RUQ pain, jaundice -Cholangitis

Signs and symptoms of bacterial gastroenteritis

fever, abdominal cramping, tenesmus, stool may have mucus and be guaiac positive or blood streaked

what is D-dimer?

fibrin degradation product that is elevated int he presence of a thrombus good to rule out PE nonspecific

diastolic failure is ____

filling problem

ventricular fibrillation Dx

fine-to-coarse zigzag pattern w/ no discernible Ps or QRS complexes

takotsubo epidemiology

first coined in Japan, but worldwide occurs in 1-2% of pts that present w/ suspected MI W > M mean age of 61-76 yo (post-menopausal)

Stable Angina Pectoris:

fixed artherosclerotic lesion. ∙*Symptoms*: pain with pressure with exertion. lasting 1-15 min and improved with rest. ∙*Diagnosis*: EKG usually normal. Stress test. Cardiac catherization. Coronary Angiography. ∙*Treatment*: lifestyle modification, aspirin, beta blockers, nitrates revascularization.

Grey Turner Sign

flank bruising; may be seen in acute pancreatitis

Pleural Effusion:

fluid in the pleural space caused by a disease process or trauma.

what is the drug of choice for pt's with severe shigella or salmonella

fluoroquinolones (cipro/levofloxacin)

myocarditis etiology

follows an upper respiratory infection (viral)

what do CCBs do?

for peripheral vasodilation maybe preferred in blacks and elderly

Signs and symptoms of Giardia

frequent, foul smelling, water stools (rarely have blood or mucus), abdominal pain, N/V, anorexia, gas

IgD:

function is uncertain.

loop diurectics Rx

furosemide bumetanide ethacrynic acid torsemide

Acute Pancreatitis most common causes

gallstones and alcohol

hyperlipidemia etiology

genetics diabetes alcohol hypothyroidism obesity/sedentary lifestyle renal dz liver dz drugs -estrogen -thiazides -beta-blockers

adbominal aortic aneurysm (AAA) Tx

gold standard > surgical placement of vascular prosthesis endovascular repair -stented graft via open femoral access -distal limbs placed in both iliac arteries

varicose veins Tx

graduate elastic stockings leg elevation regular exercise endovenous radiofrequency laser ablation sclerotherapy surgical stripping (old technique)

hypoplastic left heart syndrome characteristics

group of defects small left ventricle normally placed great vessels M > F murmur variable

Signs and symptoms of Giant Cell Arteritis

headache, scalp tenderness, jaw claudication, throat pain, diplopia Symptoms of polymyalgia rheumatica: pain of shoulder/pelvis

aggrenox (aspirin/dipyridamole) is another antiplatelet agent, but what is the biggest SE complaint?

headaches

how does cryptococcal meningitis manifest?

headaches, personality changes, visual disturbances, will have increased ic pressures

Symptoms of Ulcerative Colitis

hematochezia, diarrhea with mucus/pus, tenesmus, rectal urgency

what are some s/e of tb treatment

hepatitis, hyperuricemia, thrombocytopenia

Hiatal hernia

herniation of elements of abdominal cavity through the esophageal hiatus opening of diaphragm

What serum marker correlates with CHF

high BNP (levels > 150)

what will csf show for meningitis from tb

high protein, lymphocyte predominance, low glucose

popliteal SBP-brachial SBP > 60 mm Hg

hill's sign

which organism is a dimorphic fungus with septate hyphae

histoplama capsulatum

which disease is characterized by hepatosplenomegaly, flu like symptoms and erythema nodosum

histoplasmosis

Anal Fissure treatment

hot sitz baths, fiber, topical analgesics/steroids, surgical excision if area doesn't heal

thiazide diuretics Rx

hydrochlorothiazide chlorthalidone

hemodynamic hallmark of established primary ___ is elevated peripheral vascular resistance (PVR)

hypertension note: need at least two readings

difference btw hypertensive emergency and urgency?

hypertensive emergency is high BP w/ acute target-organ ischemia and damage; tx w/ IV agents to reduce BP w/n mins urgency is high BP (>180/120) w/out damage; tx w/ PO agents to reduce BP in hrs

Causes of respiratory alkalosis

hyperventilation, sepsis, asthma, PE, DKA, infection, stroke, toxins, lung dz Tx: pain control, CO2 rebreathing

diuretics can cause ___ and thus may need ___ supplementation

hypokalemia potassium

Cholangitis progression to sepsis is shown by Reynolds Pentad which is what?

hypotension, AMS, fever, RUQ pain, jaundice

which agent can be used for rhythm control in hemodynamically stable atrial fibrillation w/ concomitant Wolff-Parkinson-White Syndrome?

ibutilide -prolongs refractoriness of AV node -if concern for prolong QT interval or PVC, can use procainamide -if use AV nodal blockers like BB, CCB, adenosine, or digoxin...may go into VTACH

takotsubo pathophys

idiopathic theories: -catecholamine excess from physical or emotional stress -coronary artery spasm -microvascular dysfunction -mid-cavity or LV outflow obstruction

atrial fibrillation etiology

idiopathic longstanding HTN ischemic heart disease rheumatic heart disease alcohol use "holiday heart" COPD thyrotoxicosis

pericarditis etiology

idiopathic (~90%) viral infection bacterial infection autoimmune connective tissue dz neoplasm radiation therapy chemotherapy drug toxicity cardiac surgery myxedema tuberculosis (underdeveloped countries) M > F < 50 yo

myocarditis Tx

if LV EF < 40% TX W/ ACEI and BB NSAIDs for myocarditis-related chest pain antibiotics specific to infecting agent maybe corticosteroids maybe immunosuppressants

wenckebach (type 1 2nd degree block) Tx

if symptomatic and signs of hypoperfusion -atropine -elective pacemaker -immediate transcutaneous pacing good prognosis

why would you check for ebv DNA in a aids pt csf?

if you are trying to diagnose cns lymphoma in a pt without getting a brain biopsy. ebv is present in 90% of cases of cns lymphoma

ventricular fibrillation Tx

immediate electrical defibrillation (unsynchronized) at 200 J (biphasic) and 360 J (monophasic) -amiodarone -lidocaine (2nd line) epinephrine/vasopressin magnesium sulfate (refractory vfib)

Hypoxic drive in COPD?

in cases where there are chronically high carbon dioxide levels in the blood such as in COPD patients, the body will begin to rely more on the oxygen receptors and less on the carbon dioxide receptors. And that in this case, when there is an increase in oxygen levels the body will decrease the rate of respiration. The premise for hypoxic drive is that elevated oxygen levels depress the peripheral chemo receptors located in the aortic arch and carotid bodies. High oxygen saturations in a patient who has a diminished respiratory drive could pose a further deterioration of the drive. Patients identified as most at risk are those who are chronically hypoxic with moderate to severe hypercarbia. As a patient retains CO2, they buffer the increasing acidosis by retaining more bicarbonate, which in turn reduces the H-ion stimulation of the respiratory center. With a decreased stimulus, the need to maintain an elevated minute ventilation is reduced. This results in mild hypoventilation, which in turn results in mild hypoxia. The body then attempts to strike a balance between overventilation (alkalosis) and hypoventilation (acidosis).

when are IV abx indicated for lyme dz? what is the med/dose

in severe cases with 3rd degree heart block or neurologic manifestations. ceftriaxone-2g/daily, or cefotaxime 2g q8, pen g 5 million units q6

takotsubo prognosis

in-hospital mortality is ~4% pts who survive an episode have 2% risk/yr of recurrence

congestive heart failure PE

increased JVD S3/S4 rales, dullness secondary to pleural effusion hepatomegaly ascites jaundice peripheral edema cheyne-stokes resp.

Diagnosis of Intrinsic Renal Failure

increased urine sodium >40, decreased BUN:Cr ratio (<15:1)

how do u confirm a diagnosis of cryptococcal meningitis

india ink stain, fungal culture, measure level of cryptococcal antigen from csf.

Cholecystitis

infection and inflammation of the gallbladder, constant pain progressive worsening, fevers, chills, sweats Imaging: US shows pericholecystic fluid, thickened gallbladder wall, Murphy's sign

Huntington Disease

inherited disorder, develops after 30 years old, progressive chorea and dementia, usually fatal within 15-20 years

what do ACEIs do?

inhibit bradykinin degradation stimulate synthesis of vasodilating prostaglandins good in pts w/ comorbidities; CKD (renal protective) reduces mortality after MI

aortic dissection Tx

initally IV esmolol Type A (Ascending) -surgery Type B (Below) -medical management

takotsubo triggers

intense emotional stress -death of relatives -unexpected events -domestic abuse -arguments -catastrophic medical diagnoses -panic/fear -devastating financial loss -natural diasters -acute medical illness intense physical stress -acute respiratory failure -post-surgical -fracture -central nervous system condition -infection

Skip Lesions

involves discontinuous segments of abnormal mucosa; seen in Crohn's disease

What is Myasthenia gravis?

involves muscle weakness and fatigue improves with rest. Common in young women and old men.

What is CHF?

is a dysfunction of the of the heart as a pump of blood. This results in insufficient oxygen delivery to tissues accompanied by the accumulation of fluid in the lungs.

Crohn's Disease

is a type of inflammatory bowel disease that may affect any part of the gastrointestinal tract from mouth to anus, causing a wide variety of symptoms.

What is Endocarditis?

is an inflammation of the inner layer of the heart, the endocardium. Endocarditis is characterized by a prototypic lesion, the vegetation, which is a mass of platelets, fibrin, microcolonies of microorganisms, and scant inflammatory cells.

What is Cor pulmonale?

is defined as right ventricular hypertrophy with eventual RV failure resulting from pulmonary HTN, secondary to pulmonary disease.

Pneumoconiosis

is defined as the accumulation of dust in the lungs, and the tissue reaction to its presence. Risk factors include prolonged occupational exposure and inhalation of small inorganic dust particles.

Idiopathic pulmonary fibrosis

is one of the most common forms of interstitial pneumonia. Has an unrelenting progression, with death usually occurring within 5-10 years. Fibrosis can be idiopathic or secondary to a large number of inflammatory conditions, radiation, drugs, or from inhalation of toxins. All of them thicken the septum. Only some have white cell infiltrates with lymphocytes or neutrophils. Chronic conditions lead to fibrosis and thickening.

CHF especially its worst form, pulmonary edema,

is the *clinical* diagnosis.

Dyspnea (shortness of breath)

is the indispensible clue to the diagnosis of CHF.

Gastroenteritis

is the inflammation of the GI tract secondary to microbiologic infiltrate and spread.

Idiopathic pulmonary fibrosis define

is thickening of the interstitial; septum of the lung between the arteriolar space and the alveolus. Fibrosis interferes with the gas exchange in both directions.

peripheral arterial disease Sx

ischemia pain limitation in activity disability intermittent claudication (leg pain w/ activity, relieved w/ rest) weak distal pulses aortic, iliac, or femoral bruits erectile dysfunction (Leriche's syndrome) numbness tingling

atrial flutter etiology

ischemic heart disease CHF acute MI pulmonary embolus myocarditis blunt chest trauma digoxin toxicity

When can I use tPA?

ischemic stroke presenting within <3 hrs and CT head negative

Signs and symptoms of cirrhosis

jaundice, spider angiomata, gynecomastia, ascites, hepatosplenomegaly, asterixis, caput medusae, testicular atrophy, extremity changes, Dupuytren's contracture

Cor pulmonale Presentation

l. Decrease in exercise tolerance 2. Cyanosis and digital clubbing 3. Signs of right ventricular failure: hepatomegaly, edema, JVD 4. Parasternal lift 5. Polycythemia is often present if COPD is the cause of cor pulmonale.

Cor pulmonale Etiology

l. It is most commonly secondary to COPD. 2. Other causes include recurrent PE, ILD, asthma, CF, sleep apnea, and pneumoconioses.

Hyperlipidemia Clinical features

l. Most patients are asymptomatic. 2. The following may be manifestations of severe hyperlipidemia. a. Xanthelasma-yellow plaques on eyelids b. Xanthoma-hard, yellowish masses found on tendons (finger extensors, Achilles tendon, plantar tendons) 3. Pancreatitis can occur with severe hypertriglyceridemia.

Cor pulmonale Tx

l. Treat the underlying pulmonary disorder. 2. Use diuretic therapy cautiously because patients may be preload-dependent. 3. Apply continuous long-term oxygen therapy if the patient is hypoxic. 4. Administer digoxin only if there is coexistent LV failure. 5. A variety of vasodilators have been studied; no definite improvement has been shown with their use.

what will a tissue biopsy of cryptococcus show

lack of an inflammatory response

95% of people who develop TB will contain the bacterium and be asymptomatic. This is called ______.

latent TB infection (LTBI)

what are some expected lab findings for reactivation tb

laukocytosism anemia, hyponatremia 2/2 siadh

aortic stenosis increases afterload to the ___ ventricle resulting in ____

left concentric left ventricular hypertrophy (increased relative wall thickness compared to cavity size)

constrictive cardiomyopathy Dx

left ventricle is small or normal w/ mildly reduced function often pulmonary HTN CXR -mild/mod enlarged cardiac silhouette ***ECHO EKG -low voltage changes cardiac MRI cardiac cath -normal or reduced LV function endomyocardial bx -may be necessary to differentiate from pericarditis or other forms of cardiomyopathy

The most significant prognosticating factor for survival following an MI is:

left ventricular ejection fraction

what 2 components of the cardiac cycle are impaired in heart failure w/ preserved ejection fraction?

left ventricular filling and relaxation

what is cardiac tamponade?

life-threatening medical condition in which blood or fluids fill the space btw the sac that encases the heart

hyperlidemia Tx

lifestyle changes 1st line -smoking cessation -modest alcohol -daily exercise -mediterranean diet -reduce LDL: soluble fiber, garlic, soy, pecans, plant sterols, vit C Rx STATINS Niacin (SE: flushing) bile acid sequestrants fibric acid derivatives ezetimibe blocks

how do u diagnose candidiasis

local-KOH prep systemic-blood/tissue cultures

Differentiate bundle branch blocks on EKG by:

looking at the QRS complexes on the precordial leads: V1-V4: RBBB V3-V6: LBBB

___ diuretics are more effective in pts w/ impaired kidney function

loop

what do alpha blockers do?

lower peripheral vascular resistance may be initial drug of choice in men w/ symptomatic BPH

what infectious etiology is associated w/ complete heart block?

lyme dz

what are the most common sites for extrapulmonary tb

lymph nodes, then pleura, then GU tract

deep venous thrombosis (DVT) risk factors

major surgical procedures (hip replacement) prolonged bed rest oral contraceptives/hormone replacement therapy factor V ledien def cancer air travel advanced age type A blood obesity multiparity IBD lupus erythematous

hypertrophic cardiomyopathy

massive hypertrophy esp the septum small left ventricle systolic anterior mitral motion diastolic dysfunction

Signs and symptoms of Subarachnoid hemorrhagic stroke are what?

may be preceded by warning headache, neck/back pain, "worst headache of my life," thunderclap, may have loss of consciousness

Signs and symptoms of Huntington Disease

mental changes, restless, dystonic posturing, severe choreiform movements

which areas of the lung does primary pulmonary tb affect

middle and lower lung lobes in the peripheral zones

What is a Tension Headache?

mild to moderate intensity, located bilateral-frontal areas, dull band like pain, lasting for hours, associated with stress. No nausea, vomiting, neuro deficits.

so if in diastole, can have ___ regurg and ___ stenosis

mitral and tricuspid stenosis aortic and pulmonary stenosis

high-pitched blowing holosystolic murmur heard best at the apex, w/ radiation to the axilla (L) +/- thrill increased w/ handgrip decreased w/ valsalva

mitral regurgitation

low-pitched, mid diastolic ruble near the apex, heard best in the LLD position accentuated w/ exercise, LLD, expiration accentuated S1, opening snap following S2

mitral stenosis

What heart valve is most affected by rheumatic heart disease?

mitral valve

high-pitched, midsystolic click +/- mid-to-late systolic murmur (mitral regurg) exaggerated by valsalva reduced by swatting

mitral valve prolapse

which valvulopathy is commonly associated w/ premature ventricular contractions?

mitral valve prolapse

IgG:

most *abundant.*

Giardiasis

most common intestinal parasitic dz in US, water related outbreaks. Transmission person to person, contaminated food/water.

Diverticulosis

most frequently occurs in the sigmoid colon and is the most common cause of acute lower GI bleeding in patients over 40 yr of age. Risk factors include a *low-fiber and high-fat diet*, advanced age (65% occur in those > 80 years of age), and connective tissue disorders (e.g., Ehlers-Danlos syndrome).

sinus sick syndrome etiology

most often cause by scarring of the heart's conduction system infants with heart surgery drugs -digitalis -CCB -BB -sympatholytic agents -antiarrthythmic drugs -aerosol propellant abuse underlying collagen vascular/metastatic dz surgical injury coronary dz (rare)

sinus sick syndrome Tx

most required permanent pacing

systolic pulsations of the uvula

muller's sign

Major concerns of atrial fibrillation

mural embolism (may result in TIA), rapid ventricular response (can cause hemodynamic instability)

head bobbing w/ ea heartbeat

musse'ts sign

cd4 levels less than 50, hiv pts are more susceptible to...

mycobacterium avium intracellulare complex, CMV retinitis colitis esophagitis, or cns lymphoma

angina pectoris represents ___

myocardial ischemia, usually from atherosclerosis

coarctation of the aorta characteristics

narrowing of the proximal thoracic aorta infants may present w/ CHF older children may have systolic HTN or underdeveloped lower extremities

how do you collect sputum for a positive tb diagnosis

need 3 samples of early morning sputum

how do u diagnose cryptococcosis

need LP. latex agglutination will show crytptococcal antigen in csf. india ink smear shows encepasulated yeasts

Parkinson's Disease

nerve cell damage in brain causes dopamine levels to drop leading to sings and symptoms

pts w/ severe aortic regurgitation may benefit from which long-acting vasodilators?

nifedipine XL long acting calcium channel blckrs

prinzmetal/variant angina Tx

nitrates CCB BB

How do you diagnose Guillian Barre?

no fever at onset, CSF protein >45 and low WBC, MRI shows selective enhancement of anterior spinal nerve roots

NYHA functional classification class 1

no sx w/ ordinary activity

1st degree block Tx

no therapy required prognosis good

unstable angina/NSTEMI =

non ST elevation myocardial infarction

Bacterial gastroenteritis

non typhoidal Salmonella, Shigella, Campylobacter, E coli, Vibrio (developing nations from eating undercooked shellfish)

1st degree block

not truly a "block", just slowed conduction down normal pathway (delay in AV conduction)

how do u treat oral thrush

nystatin mouth wash, clomitrazole troches

pulsus paradoxus Sx (clinical finding)

one can detect cardiac beats upon auscultation during inspiration but they cannot be palpated at the radial pulse

When do you see Hepatitis D?

only seen with Hepatitis B and associated with more severe course

myocarditis Sx

onset several days to a few weeks after an acute febrile illness or respiratory infections maybe heart failure maybe chest pain tachycardia gallop rhythm arrhythmias

what drug can reduce the effects of treatment for tb?

oral contraceptives

how do u treat candidial esophagitis

oral fluconazole, ketoconazole

Pathophysiology of Angina:

oxygen demand outweighs the oxygen supply; it's due to vasoocclusion (MC atherosclerotic plaques)

Metabolic Alkalosis

pH high CO2 increases to compensate Bicarb high

Respiratory Alkalosis

pH high CO2 low Bicarb may decrease to compensate

Metabolic Acidosis

pH low CO2 decreases to compensate Bicarb low

Respiratory Acidosis

pH low CO2 high Bicarb may decrease to compensate

"Pack Year"

packs of day x number of years = pack year

Biliary Colic

pain when gallbladder contracts against obstructing stone in neck of gallbladder (no infection)

Internal hemorrhoids

painless, bright red blood per rectum, fecal/mucus leakage, pruritus, insensate

___ : failure or delayed closure of the channel bypassing the lungs, which allows placental gas exchange during the fetal stage

patent ductus arteriosus

Felbamate typically reserved for what patients with seizures?

patients unresponsive to other meds because of serious side effects

what is the test of choice for diagnosing chlamydia

pcr-molecular diagnostics

percarditis Tx

pericardiocentesis-to relieve fluid accumulation pericardial window-for recurrent effusions strictly inflammatory can be tx w/ NSAIDs or steroids (give w/ PPI to protect against ulcers) pericardiectomy recurrent > consider colchicine w/ NSAIDS

Anal Fistula signs and symptoms

persistent, malodorous, bloody discharge

sinus tachycardia etiology

physiologic stimuli -pain -exertion drugs -sympathomimetics -caffeine -bronchodilators pathologic stimuli -fever -hypoxia/MI -anemia -hypovolemia -pulmonary embolism -hyperthyroidism

sinus sick syndrome

physiologically inappropriate sinus bradycardia, sinus pause, sinus arrest, or episodes of alternating sinus tachycardia and bradycardia; often in elderly

heard over femoral a

pistol shot sounds

STEMI Tx

place pt on CARDIAC MONITOR to recognize any dysrhuthmias establish PERIPHERAL IV give OXYGEN administer ASPIRIN "MONA" greets all pts w/ chest pain at the door: MORPHINE OXYGEN NITROGYLCERIN ASPIRIN -emergent PCI = percutaneous coronary intervention (door to balloon time < 90 mins) -thrombolytics w/n 12 hrs of onset -beta blockers w/n 24 hrs -ACEI w/n 24 hrs -NTG +/- morphine maintenance anticoagulation -heparin -enoxaparin (LMWH) maintenance antiplatelet therapy -aspirin -clopidogrel GP IIb/IIIa inhibitors statins aldosterone inhibitors cardiac monitor IV line supplemental O2 way to remember...5 Ls lopressor-BB lisinopril-ACEI lipitor-statin lovenox-anticoagulation little aspirin

at cd4 levels less than 200, hiv patients are more susceptible to infections like...

pneumocystic jirovecii, toxoplasmosis, crytococcosis, histoplasmosis or cryptosporidiosis

in hiv infection, what does lack of sputum production and elevated ldh usually point towards

pneumocystic pneumonia

a giemsa or silver stain can be used to diagnose what condition for hiv pts

pneumocystis pneumonia

how do u know your tb treatment has failed?

positive cultures after 3 months of tx, or positive afb stains after 5 months

endocarditis Duke Minor criteria

predisposing factor fever > 100.48 F (38.8 C) vascular phenomena (embolic dz or pulmonary infarc) immunologic phenomena (glomerunephritis, osler nodes, roth spots) positive blood culture not meeting major criteria

premature ventricular contractions Dx

premature and wide QRS complex no preceding P wave ST segment and T wave of PVC are directed opposite the preceding major QRS deflection

Cholelithiasis

presence of gallstones in gallbladder

what is pyridoxine used for in tb treatment

preventing peripheral neuropathy that is caused by isoniazid therapy

10% of people exposed to TB will develop the disease. This is called ______ TB

primary TB

in which disease will you see hilar and paratracheal lymphadenopathy?

primary pulmonary rb

vasospasm of coronary artery(myocardial ischemia) leading to episodic chest pain unrelated to exertion

prinzmetal/variant angina *occurs in diseased or normal arteries

Diaphoresis

profuse sweating

chronic venous insufficiency Sx

progressive edema starting at the ankle itching dull pain w/ standing pain w/ ulceration skin is shiny, thin, atrophic w/ dark pigmentary change and subcutaneous induration

5% of patients with latent Tb will not contain the bacterium and progress to active TB. This is called ______ ______ TB.

progressive primary TB

wenckebach (type 1 2nd degree block)

progressive prolongation of PR interval (with gradual falling RR) until AV node fails to conduct and a ventricular beat is dropped

wenchkebach/mobitz I EKG

progressive prolongation of PR interval until QRS dropped

Antibiotic-associated diarrhea is almost always caused by Clostridium difficile colitis, which in the most severe cases causes the classic...

pseudomembranous colitis.

Signs and symptoms of Myasthenia gravis?

ptosis, diplopia, difficulty chewing, limb weakness, respiratory difficulties

low-pitched diastolic murmur in 3rd-4th intercostal space adjacent to sternum increases w/ inspiration

pulmonary regurgitation

soft to loud, hard mid-systolic crescendo-decrescendo murmur heard best at the 2nd-3rd LICS, radiating to the left shoulder and neck

pulmonary stenosis

___ is an abnormally large decrease (> 10 mm Hg) in systolic blood pressure and pulse wave amplitude during inspiration

pulsus paradoxus

subungual capillary pulsations

quincke's puses

atrial fibrillation

rates > 300 bpm *if wide QRS complexes are concerning for a preexcitation syndrome such as Wolff-Parkinson-White syndrome

atrial flutter

rates btw 250-350 bpm saw tooth note: AV block is usually 2:1

ankle/branchial index (ABI)

ration of SBP ankle/ SBP arm normal ABIs > ankle SBP is 10-15 mmHg higher than brachial systolic pressure ABI < 0.9 = significant dz > 1.0 = normal < 0.9 = diagnosis of PAD < 0.7 = intermittent claudication < 0.4 = rest pain < 0.1 = impending tissue necrosis

at cd4 levels less than 500 what are hiv patients susceptible to?

recurrent pneumonia, tb, vaginal candidiasis, herpes zoster

diuretics do what?

reduce plasma volume reduce peripheral vascular resistance recommended for initial therapy of essential hypertension "water pill"

Signs and symptoms of complex regional pain syndrome are what?

regional pain in affected limb, restricted mobility, edema, color changes of skin, spotty bone thinning. Hallmark is severe burning or throbbing pain with associated allodynia in affected extremity.

wenckebach (type 1 2nd degree block) Dx

regularly irregular

takotsubo Tx

resolution of emotional or physical stress management of cardiogenic shock varies depending upon whether significant left ventricular outflow tract obstruction is present if w/ intraventricular thrombus > anticoagulation heart failure tx as per standard guidelines

Essential features of Parkinson's Disease

resting tremor, bradykinesia, rigidity, postural instability

constrictive cardiomyopathy etiology

results from fibrosis or infiltration of the ventricular wall because of collagen-defect dz (amyloidosis), radiation, post-op changes, diabetes, endomyocardial fibrosis

rheumatic fever etiology

results from molecular cross-reactivity btw streptococcus and host cell proteins resulting in antibody formataion type II hypersensitivity rxn mitral valve most commonly involved

_____ is a systemic immune response occurring usually 2-3 weeks following a beta-hemolytic streptococcal pharyngitis; most commonly affects the heart, joints, skin, and CNS; recent immigrants and children 5-15 yo

rheumatic fever

mitral stenosis etiology most common

rheumatic heart dz

bundle branch block/fasicular blocks

right bundle branch block (RBBB) -prolongs the QRS duration > 0.12 sec -causes RSR in early precordial leads (V1-2) left bundle branch block (LBBB) -a bifascicular block

angina pectoris Tx

risk factor modifications meds -81-325 mm ASA daily -beta blocker -ACEI -nitrates, PRN or long acting -statins consider revascularization (PCI/stent vs CABG)

CHADS2 score?

risk of stroke in patients with non-rheumatic atrial fibrillation (AF), since AF can cause stasis of blood in the upper heart chambers, leading to the formation of a mural thrombus that can dislodge into the blood flow, reach the brain, cut off supply to the brain, and cause a stroke. CHF HTN >140/90 Age >75 Diabetes Stroke (2 pts) 0 = aspirin/or nothing 1 = aspirin or warfarin >2 = warfarin

saccular vs fusiform aneursym

saccular- portion of artery forms an outpouching or "mushroom" or "berry" fusiform-entire arterial diameter grows

in which stage of lyme disease would you see CN palsies and pericarditis?

second stage, early disseminated

in which stage of lyme disease would you see meningitis

second stage, early disseminated

Any angina during post-MI hospitalization should be:

sent out for CABG

What is a Cluster Headache?

severe, unilateral headache localized to periorbital/temporal area accompanied by lacrimation, rhinorrhea, ptosis, myosis, nasal congestion, eyelid edema -Occurs in clusters meaning 1-8 daily attacks lasting 15-90 minutes for 4-6 weeks... followed by pain free interval 3-6 months

How is Hepatitis B transmitted?

sexually, transfusion, IV drug use

Causes of metabolic acidosis

shock, renal failure, ischemic bowel, DKA, starvation, EtOH Tx: volume resuscitation, correct underlying cause

contraindication to nitroglycerin?

sildenafil use w/n 24 hrs RV infarction

_____ is normal sinus P waves and PR intervals w/ atrial rate btw 100-160 bpm

sinus tachycardia

How do you confirm diagnosis of Celiac disease?

small bowel biopsy

peripheral arterial disease modifiable risk factors

smoking diabetes HTN lipids sedentary lifestyle obesity *PAD risk factor for cerebrovascular and coronary artery disease

peripheral arterial disease Tx

smoking cessation progressive exercise lipid-lowering meds (statins) anticoagulation -unfractionated heparin -aspirin -clopidogrel catheter-directed embolectomy for erectile dysfunction -revascularization -phosphodiesterase (sildenafil)

varicose veins Sx

spider veins distal edema abnormal pigmentation fibrosis atrophy skin ulceration (prolonged dz)

potassium sparring diurectic/ adolesterone receptor antagonist Rx

spironolactone

Valvular Disease: Aortic Dissection Tx

stabilize blood pressure (e.g., nitroprusside, β-blockers) if unstable; Stanford A dissections need emergency surgery; Stanford B dissections can be treated medically unless rupture or occlusion develops

premature ventricular contractions Tx

stable -no tx pts w/ 3 or more PVCs in a row, manage as VT unstable -lidocaine (unless allergic to amide anesthetics)

aortic dissection 2 classifications?

stanford debakey

Choledocholithiasis

stones in CBD S/S: dark urine, light stools

mobitz (type 2 2nd degree block) Tx

stop all nodal blockers *atropine is 1st line in asymptomatic pts place trancutaneous pacer pads in case of further deterioration into complete heart block typically requires pacemaker if... -symptomatic -exercise-induced -HR < 40 or pause > 3 sec

rheumatic fever Dx pt w/ evidence of prior group A _________infection along w/ either __ major criteria OR __ major and __ minor criteria

streptococcus 2 major OR 1 major and 2 minor

premature atrial contractions etiology

stress fatigue alcohol use tobacco coffee COPD digoxin toxicity CAD adenosine-converted paroxysmal SVT

aortic dissection Sx

sudden "ripping/tearing" chest pain back pain btw shoulder blades aortic regurgitation murmur changes in pulse asymmetric pulses/BP most are males, older than 50 yo, have hx of HTN OR young w/ connective tissue dz, congential heart dz, or pregnancy

Traditional signs and symptoms of a stroke

sudden numbness/weakness, sudden confusion/aphasia, memory deficit, visual deficit, dizzy, sudden HA with no cause

NSTEMI/unstable angina Sx

sudden onset chest pain/pressure SOB nausea diaphoresis chest pain at greater frequency, severity, or w/ less activity chest pain at rest or nocturnally chest pain previously controlled nitrates, now refractory

STEMI Sx

sudden onset left-sided, substernal chest pain radiates down L arm or up L jaw SOB nausea/vomiting diaphoresis HTN or hypotension tachycardia or bradycardia S3, S4, both signs of CHF systolic murmurs (mitral regurg, ventric septal defect) friction rub (usually day 2 or 3)

how do u treat cerebral toxoplasmosis

sulfadiazine and pyrimethamine, lesion should regress in two weeks or consider cns lymphoma

phlebitis/DVT/PE Tx

superficial dz -bed rest -local heat -elevation of extremity -NSAIDs DVT prophylaxis -elevation of foot of the bed -leg exercises -compression hose -anticoagulation (LMWH) -IVC filter if contraindications to anticoagulation -frequent ambulation

Anal Fissure

superficial linear tears of anal canal usually caused by local trauma (passage of hard stools) and are most common cause of painful rectal bleeding

Treatment of Subarachnoid hemorrhage

surgery, control hypertension, analgesics

aortic dissection stanford type A Tx

surgical repair aortic graft replacement of aortic valve

NYHA functional classification class 3

sx w/ minimal activity

NYHA functional classification class 2

sx w/ ordinary activity

NYHA functional classification class 4

sx w/ rest

sinus sick syndrome Sx

syncope dizziness confusion heart failure palpitations decreased exercised tolerance

MC presenting sx of HCM

syncope; other sx: angina, palpitations, dizziness

congestive heart failure right-sided Sx

systemic vascular congestion peripheral edema pitting edema RUQ discomfort bloating satiety/decreased appetite

mitral and tricuspid regurgitation occurs during ___

systole

atrial septal defect murmur

systolic ejection murmur at 2nd left intercostal space (early to middle systolic rumble)

coarctation of the aorta murmur

systolic murmur LUSB and left interscapular area (left upper sternal border) murmur maybe continuous

transposition of the great vessels murmur

systolic murmur associated with VSD systolic murmur w/ pulm stenosis

ventricular septal defect murmur

systolic murmur at LLSB (lower left sternal border)

aortic dissection stanford type B

tear is distal to the left subclavian artery in the descending aorta

aortic dissection stanford type A

tear is in the ascending aorta distal to the aortic valve

aortic dissection debakey type 1

tear is in the ascending aorta distal to the aortic valve

aortic dissection debakey type 2

tear is only in the ascending aorta

aortic dissection debakey type 3

tear is only in the descending aorta

STEMI etiology

thrombotic obstruction of epicardial coronary arteries

how is cryptococcus neoformans transmitted

thru inhallation of fungus into lungs

what is the treatment for PCP

tmp-smx

what do u use for ppx in pts with cd4 count less than 200

tmp-smx three times per week

ventricular fibrillation

totally disorganized depolarization and contraction of small areas of ventricular myocardium during which there is no effective ventricular pumping activity

what is the study of choice in an unstable patient to confirm the diagnosis of aortic dissection?

transesophageal echocardiogram

Variant (Prinzmetal) Angina:

transient coronary vasospasm. Can lead to dysrhythmias or infarction. Can be treated with calcium channel blockers.

Delirium

transient disorder characterized by impaired attention, perception, memory and cognition. Sleep wake cycles interrupted ("sundowning"). Reduced alertness, activity levels change rapidly.

double sound heard over femoral a when compressed distally

traube's sound

Myocarditis Tx

treat infection; stop offending medications; avoid exertional activity; treat heart failure symptoms as for acute exacerbation of heart failure

rheumatic fever Tx

treat residual group A streptococcus w/ antibiotics IM PCN treat pain and inflammation w/ NSAIDs salicylates steroids

Treatment of Delirium

treat underlying cause, Haloperidol 5-10mg for agitation, Lorazepam 0.5-2 mg

Treatment of chronic pancreatitis

treat underlying problem (quit drinking); low fat diet, surgical removal of part of pancreas

holosystolic (pansystolic), blowing murmur at LLSB, radiates to sternum and xiphoid area increases w/ inspiration

tricuspid regurgitation

soft, high-pitched diastolic murmur at LSB increases q/ inspiration

tricuspid stenosis

metabolic syndrome includes _______________ and is associated with the development of diabetes and increased risk of cardiovascular complications

truncal obesity hyperinsulinemia insulin resistance hypertriglyceridemia hypertension

peripheral arterial disease 3 patterns of dz

type 1 -15-20% of pts -limited to aorta and common iliac artery -40-55 yo -smokers -hyperlipidemia type 2 -25% of pts -involves aorta, common iliac artery, external iliac artery type 3 -60-70% of pts (most common) -multilevel disease -affects aorta, iliac, femoral, popliteal, and tibial arteries

2nd degree block types

type 1 = wenckebach type 2 = mobitz

1st degree block etiology

typically related to aging of the conduction system other transient causes -high vagal tone -ischemia -electrolyte abnormalities -meds: BB, CCB

abdominal aortic aneurysm (AAA) Dx

ultrasound -size of aneurysm -presence of clot -non-invasive CT/MRI/MRA -anatomic detail -precise location of aneurysm -CT is best way to monitor growth

Cullen Sign

umbilical bruising associated with hemorrhagic pancreatitis

aortic dissection stanford type B Tx

uncomplicated: medical tx w/ tight BP control -beta blockers -afterload reducers (sodium nitroprusside) complicated: maybe surgical repair

Describe a Migraine Headache?

unilateral location, pulsatile quality, moderate to severe intensity, aggravated by movement, nausea, vomiting, photophobia, phonophobia lasting 4-72 hours

patent ductus arteriosus Tx

unlike other congenital anomalies, surgical tx is NOT indicated as many pts respond to IV indomethacin

sinus tachycardia Tx

unstable -synchronized cardioversion stable -1st perform valsalva maneuvers (strain, face in cold water, carotid sinus massage if no bruits) -adenosine 6 mg IV rapid bolus followed by 20 mL saline flush -if narrow complex SVT and normal cardiac function, can use these 2nd line agents: CCBs (diltiazem, verapamil), BB (esmolol, metoprolol, propranolol), digoxin) -if wide complex SVT, use procainamide (contraindicated in myasthenia gravis)

atrial flutter Tx (same as Afib)

unstable -synchronized cardioversion stable -anticoagulate w/ heparin before cardioversion -consider TEE to rule out atrial thrombus before cardioversion -control rate w/ diltiazem (used procainamide in WPW pts) -amiodarone or digoxin in pts w/ EJ < 40% (increased risk of rhabdomyolysis if given w/ simvastatin) note: carotid sinus massage or valsalva maneuver are useful techniques to slow the ventricular response by increasing the degree of AV block which can unmask flutter waves in uncertain cases

atrial fibrillation Tx (acute)

unstable -synchronized cardioversion stable -anticoagulate w/ heparin before cardioversion -consider TEE to rule out atrial thrombus before cardioversion -control rate w/ diltiazem (used procainamide in WPW pts) -amiodarone or digoxin in pts w/ EJ < 40% (increased risk of rhabdomyolysis if given w/ simvastatin) -afib < 48 hrs may be chemically/electrically converted (amiodarone, ibutilide, procainamide, flecainide, or propafenone)

Diagnosis of Prerenal failure

urine Na <20, 20:1 BUN/Cr ratio, urine specific gravity >1.020

wenckebach (type 1 2nd degree block) Sx etiology

usually asymptomatic occurs in -athletes (high vagal tone) -elderly (slowed conduction) -ischemia -drugs (BB, CCB, anti-arrhythmics)

pulmonary atresia characteristics

usually intact ventricular septum pulmonary valve is closed atrial septal opening patent ductus arteriosus

dilated, tortuous veins that develop superficially in the lower extremities, particularly in the distribution of the great saphenous vein

varicose veins

most common congenital heart defect

ventral septal defect

3rd degree/complete heart block Dx

ventricular (wide, slow) escape rhythm

premature atrial contractions etiology

very common mostly in pts w/ ischemic heart dz/acute MI CHF digoxin toxicity hypokalemia alkalosis hypoxia sympathomimetic drugs

How is Hepatitis C transmitted?

via exposure to contaminated blood or blood products

Causes of Prerenal failure

volume loss (N/V/D, hemorrhage, diuretics), decreased cardiac output (MI), renal artery/small vessel disease, renal artery stenosis or embolic disease

Causes of metabolic alkalosis

volume loss, K loss, excessive diuresis, GI loss Tx: volume replace (NaCl solution and K replacement)

Treatment of Prerenal failure

volume resuscitate

Hematemesis

vomiting of blood

long term anticoagulation for Afib Rx

warfarin aspirin dabigatran rivaroxaban

JNC 8 Hypertension Tx Algorithm -lifestyle interventions throughout; ex: -set BP goal and initiate BP lowering meds based on __, __, __, and __

weight loss aerobic physical activity (> 30 mins/day, most days) low fat diet more fruits, veggies, and fiber low salt diet (< 2 g/d) limit alcohol smoking cessation age DM CKD race

heart block second degree type I aka

wenckebach mobitz I

when is a gastric lavage helpful in botulism

when suspected ingestion of toxin was within several hours

transposition of the great vessels characteristics

when the aorta and pulmonary artery switch (complete transposition)

ventricular tachycardia Dx

wide QRS complex rate faster than 100 bpm (most common 150-250) a regular rhythm constant QRS complex

varicose veins risk factors

women pregnancy obesity fam hx prolonged sitting/standing hx of phlebitis mechanism -superficial venous insufficiency -valvular incompetence

Unstable Angina:

worsening of stable angina or symptoms at rest.

How long does Guillian Barre Syndrome last?

worst at 2-4 weeks after onset, plateaus next 2-4 weeks, remits weeks-moats

Spirometry Results:

→A: Normal patient. →B: Restrictive defect. →C: Obstructive defect.

Treatment: Asthma

→Airway →Breathing →Fluids →SQ or IM Epinephrine 1:100 solution

Forced Vital Capacity:

→Amount of air exhaled after minimal inspiration.

Exacerbation:

→An acute asthma exacerbation is commonly referred to as an asthma attack. →SOB, wheezing, and chest discomfort. →Acute severe asthma=status asthmaticus, does not respond to standard treatments of bronchodilators and steroids.

After Hospital (Angina)

→Aspirin →Nitrates →Beta-blockers →Statins to decrease LDL cholesterol to below 70. →30 minutes exercise →Weight management

Hypersensitivity Responses: Type I

→Asthma, allergic rhinitis anaphylaxis.

Cystic Fibrosis:

→Autosomal recessive gene. →Alteration in the Cystic Fibrosis transmembrane conductance regulator. *→Abnormality in Na and Cl balance.* *→Increased pulmonary infections due to increased mucus in the lungs.* →Life expectancy now to 33 years. →Genetic Treatment-try to replace abnormal gene with normal one.

Blood tests to order: Angina

→Blood count. →Troponin I →Chemistry

Cor pulmonale (Right Sided Heart Failure):

→COPD, Sleep Apnea, etc...→Decreased Oxygen→Increased BP in the arteries of the Lung→Pulmonary Hypertension→Cor pulmonale

Hypersensitivity Responses: Type IV

→Contact dermatitis or TB skin test.

Obstructive Lung Diseases: Associated with Dyspnea

→Emphysema. →Chronic Bronchitis. →Asthma. →Bronchiolitis. →Cystic Fibrosis. →Bronchiectasis.

Hypertension:

→Essential or secondary

Hypersensitivity Responses: Type II

→Graves disease. ∙IgG or IgM bound to cell surface antigens, with subsequent complement fixation. ∙drug-induced hemolytic anemia

Sting Reaction:

→IgE mediated. →May respond to course of oral steroids. →50% chance of similar reaction will occur, but unlikely..

Arterial Blood Gas:

→Measures the *acidity (pH) and the levels of oxygen and carbon dioxide* on the blood from an *artery.* →Used to check* how well your lungs are able to move oxygen into the blood and remove the carbon dioxide* from the blood.

Immune Responses: Innate Immunity

→Non-specific. →Primitive →Activates immediately when body is exposed to threatening agent i.e. skin. →Nonspecific, first line of defense against microbes. →Rapid but limited responses. *→Neutrophils, macrophages and plasma proteins.*

Anaphylaxis:

→Occurs in 4% of population. →Treatment includes assessment of ABC's and injection of 0.3-0.5mL ro 1:1000 solution of epinephrine. →Desensitization therapy can be offered to those with know anaphylaxis, as their risk of future severe reactions can be reduced up by 50%.

Common causes of Angioedema and Urticaria: Non-immunologic causes

→Physical stimuli: exposure to sun, water, temperature →Direct mass cell degranulation: opiates, vancomycine, aspirin, radiocontrast media →Foods containing high levels of histamines: strawberries, tomatoes

Community Acquired Pneumonia: Most common organisms

→S. pneumoniae--60-70% →H. influenza →S. aureus →M. catarrhalis →Atypical organisms: influenza virus, mycoplasma, chlamydia, legionella, adenovirus, or other inidentified microorganism. ∙Outpatient treatment unless hypoxic, over 50, and with other underlying medical problems.

Hypersensitivity Responses: Type III

→Serum sickness. ∙involve circulating antigen-antibody immune complexes that deposit in *postcapillary venules, with subsequent complement fixation.*

Laboratory Tests: Anaphylaxis

→Supported by documentation of elevated concentrations of serum or plasma total tryptase or plasma histamine. →History is best for documentation

Peak Flow:

→The peak flow meter is a simple device that can measure the maximum volume rate of air during the first second of expiration. →Follow "personal best" →Helps monitor attacks. →Monitors control with medication.

Hymenoptera Stings:

→Wasp, yellow jackets, hornets, honeybees, bumble bees, fire ants. →Local reactions occur as a result of toxic properties of venom. →Promptly remove stinger. →Grasping base of stinger can result in compression of the venom containing sac →Scrap or brush it off instead

Exercise Induced Asthma:

→airways narrow as a result of exercise →aka: exercise-induced bronchoconstriction. →can be difficult to diagnose clinically give the lack of symptoms. →Is this asthma or heart problems?

Third-Degree (Complete) Heart Block:

∙*Absence of conduction of atrial impulses to ventricles.* ∙Ventricular pacemaker (escape rhythm) maintains a ventricular rate of 25-40 BPM. ∙Treatment is *ventricular implantation.*

Immunity Types:

∙*Active* Immunity: →"Self-generated" →*Exposure to an antigen.* →Ex., "flu shot" ∙*Passive* Immunity: →"Borrowed immunity" →*Transfer to preformed antibodies.* →Can provide immediate protection or bolster resistance. →Ex., transfer of IgG antibodies from mother to fetus.

Pathobiology:

∙*Chronic Bronchitis*: cough and sputum for 3 months of the years for 2 consecutive years. →1/3 of smokers 35 to 59 years. ∙Emphysema enlargement of bronchioles and alveoli. →Alpha 1 antitrypsin deficiency develop this without smoking. ∙Reduction in expiratory flow. ∙Chronic Bronchiolitis-infection of the bronchioles.

Epinephrine:

∙*Drug of choice for the treatment of anaphylaxis.* ∙Acts as an agonist at alpha-1 receptors to mediate increased vasoconstriction, increased peripheral vascular resistance, and decreased mucosal edema. ∙Agonist effects at beta-2 receptors result in bronchodilation and decreased mediator release from mast cells and basophils.

Stress Test:

∙*Exercise ECG is the mainstay test for the diagnosis of CAD*. ∙Carried out on a treadmill. ∙The patient exercises according to a set protocol from rest to maximum exertion (Bruce Protocol). ∙*Focus on the ST segment, which must fall more than 1mm from rest and be horizontal to be significant.* *∙If patient can't do stress test, it can be induced with: Persantine or Dobutamine.* ∙Cardiolite Stress ∙Echo Stress.

Pathogenesis:

∙*Immunoglobin E type I hypersensitivity and mast cell mediatory* response to aeroallergens. ∙Histamine triggers nerve receptors which causes: →itching →sneezing →increased secretion, rhinorrhea (also mediated by D4 [LTD4] →nasal obstruction (mediated by protaglandins and leukotrienes) ∙biphasic allergic inflammation (15-30 minutes then several hours).

Pericardial Diseases: Pericarditis

∙*Inflammation* of the pericardial sac. ∙Most common finding is *chest pain that is pleuritic* (associated with breathing). ∙Pain is positional, aggravated by lying flat and relieved by sitting up and leaning forward. ∙Most common EKG finding is *diffuse ST elevation and PR depression.*

Pulmonary Disease:

∙*Lung Cancer*-most common. ∙*Emphysema and Chronic Bronchitis*-second most common cause of disability in US and forth leading cause of death in the US. ∙*Asthma.* ∙*Sleep Disordered* breathing.

Second-Degree AV Block: Mobitz II

∙*P wave fails to conduct suddenly*, without a preceding PR interval prolongation. ∙Often progresses to complete heart block. ∙*Site of block is within the His-Purkinje system.*

Waves:

∙*P*-atrial depolarization. ∙*Q*-first downward deflection. Pathological Q waves takes days to hours to days to develop. Pathologic Q waves >1mm and >25% the height of QRS complex. ∙*R*-first upward deflection. ∙*T*-ventricular repolarization.

Right-Sided Heart Failure:

∙*Result of left-sided heart failure.* ∙Increased fluid pressure transferred back to the lungs, damaging the right side of the heart. ∙Right sided loses pumping power,* blood backs up in the body's veins.* *∙Swelling or congestion in the legs, ankles and swelling within the abdomen.*

Types of Rhinitis:

∙*Seasonal* allergic Rhinits: aeroallergen trigger which varies based on location and climate. ∙*Perennial Rhinitis*: common triggers include dust mite. ∙*Episodic*: in response to which patient is occasionally exposed. ∙*Mixed Rhinitis*: presence of both allergic and nonallergic rhinitis in the same patient.

Bradyarrhythmias:

∙*Sinus Bradycardia*-HR <60. *Clinically significant. <45.* Cardiac pacemaker may be required for chronic treatment.* ∙Sick Sinus Syndrome-persistent spontaneous sinus bradycardia. ∙AV Block-conduction between the atria and ventricles is impaired.

Types of congestive heart failure:

∙*Systolic*: dilated left ventricle with *impaired contractility.* ∙*Diastolic*: normal or intact left ventricle that has an *impaired ability to relax, fill and eject blood.*

Diuretics:

∙*Thiazide*-inhibits *distal convoluted tubule sodium and chloride resorption.* ∙*Loop*-*inhibits loop of Henle and proximal and distal convoluted tubule sodium and chloride resorption.* ∙*Potassium Sparing*-*antagonizes aldosterone on distal convoluted tubule*, decreasing sodium and water resorption and increasing postassium retention.

Oral Antihistamines:

∙1st generation or 2nd generation. ∙2nd generation preferred over 1st because of reduced side-effects.

Obstructive Sleep Apnea Syndrome: Studies

∙2% women and 4% men over the age of 50 years have symptomatic obstructive sleep apnea. ∙Noctural polysomnography is the gold standard for diagnosing obstructive sleep apnea.

Causes of Heart Failure: Diastolic dysfunction

∙20 to 60% of HF cases. *∙Occurs when LV all thickness and LV compliance decreases.* ∙This impairs LV filling and CO ∙Due to hypertension, AS inflitrative cardiomyopathy.

Acute Bronchitis:

∙5% of adults annually. ∙Last 1-3 weeks. ∙Believed to be viral though most patients are treated with antibiotics. ∙Cough, sputum, malaise, wheeze and may last for 10-14 days.

Lymphocytes:

∙80% T-cells (thymus dependent). →Helper T cells. →Suppressor T cells. ∙10-15% B cells (bone marrow derived). →Plasma cells. →Memory cells and antibodies. ∙5-10% Natural Killer (NK) cells. →Distinguish between normal cells and those altered by cancer or virus and kill them.

How much aspirin is usually prescribed?

∙81 mg/d

Pneumothorax:

∙A collapsed lung can occur spontaneously in a health person or in someone who has lungs compromised by trauma, asthma, bronchitis, or emphysema.

Hypovolemic Shock: Hemorrhagic vs. Nonhemorrhagic

∙A condition in which low blood volume, due to massive internal or external bleeding or extensive loss of body water, results in inadequate perfusion.

Obstructive Sleep Apnea Syndrome:

∙A disorder in which complete or partial obstruction of the airway during sleep causes loud snoring, oxyhemoglobin desaturations and frequent arousals due to apneic episodes leading to daytime fatigue. ∙Underrecognized. ∙Most often overweight, with associated peripharyngeal inflitration of fat and/or increased size of the soft palate and tongue. ∙Whether the obstruction is incomplete (hyopnea) or total (apnea), the patient struggles to breath and is aroused from sleep.

Wheeze:

∙A musical sound that lasts longer than 80 to 100 msec. ∙"Not all that wheezes is Asthma". ∙Pulmonary problems, cardiac problems, upper respiratory problems, foreign body parasites.

Paroxysmal Supraventricular Tachycardia:

∙AV nodal rentrant tachycardia. ∙Treatment includes vagus nerve stimulation. ∙*Adenosine* can be used for diagnosis and treatment. Acts as AV nodal blocking agent. ∙Radiofrequency catheter ablation of the AV node or accessory pathway.

Immune Disease:

∙Abnormal functional of the immune system. ∙Two general ways. →Immunodeficiency diseases. Too little immune response. Ex: SCID, AIDS →Inappropriate immune attacks. Overstimulated or misdirected immune response. Categories of inappropriate attacks. Ex: autoimmune response, immune complex, allergies.

Innate-Complement System:

∙Activated by antibodies and carbohydrate chains on foreign cell surfaces. ∙Forms membrane attack complexes that punch holes in victim cells. ∙Cascade sequence of events: →Embeds in surface membrane. →Hole results. →Victim cell swells and bursts.

Glycoprotein IIb/IIIa inhibitors:

∙Administered through an intrevenous injection or IV infusion during hospitalization. ∙Oral form is associated with increased mortality. *∙used in treating patients who have unstable angina, certain types of heart attacks.* *∙potent platelet inhibitors.* ∙platelets to adhere to abnormal surfaces and aggregate is mediated by surface membrane glycoprotien (GP) receptors.

Treatment:

∙Allergen avoidance. ∙Intranasal corticosteroids ∙Intranasal antihistamines ∙Oral antihistamines ∙Intranasal Cromolyn ∙Oral Leukotriene Receptor Antagonist ∙Decongestants ∙Topical Saline ∙Allergy shots ∙Sublingual immunotherapy

Wolf-Parkinson White Syndrome:

∙An accessory conduction pathway from atria to ventricles through the bundle of Kent, *causing premature ventricular excitation because it lacks the delay seen in the AV node.* ∙Treatment is *radiofrequency ablation.* ∙Want to avoid AV nodal blocking agents. ∙Look for delta wave.

Adaptive Immunity:

∙Antibody-mediated or humoral immunity. →B lymphocytes. →Antibodies. ∙Cell-mediated immunity. →T lymphocytes. →Directly attack unwanted cells.

Antibody-mediated Immunity:

∙Antigen detection. ∙B cells stimulated to form active plasma cells or dormant memory cells. →Plasma cells produce specific antibodies which last 5-7 days. →Memory cells are important in re-exposure to the antigen.

Diagnosis of Pulmonary Embolism:

∙Arterial Blood Gas →Hyocapnia →Hypoxemia →Alveolar-arterial gradient ∙CXR often normal. ∙Spiral CT →contrast-enhanced spiral CT misses clots in the middle and lingular, nearly horizontal. ∙Spiral CT with imaging for DVT ∙V/Q scan. →high probability, intermediate probability, low probability, nearly normal and normal ∙Arteriogram.

How can you help patients to stop smoking?

∙Ask at each visit about tobacco use. ∙Advise to quit through personal message. ∙Assess willingness. ∙Assist them in the process. ∙Arrange for follow up.

To further reduce infarct:

∙Aspirin ∙Heparin ∙Nitroglycerin IV ∙B-blocker ∙ACE: Angiotensin-converting enzyme. ∙Magnesium replacement if low to prevent abnormal rhythm.

Asthma:

∙Atopic: Type I hypersensitvity. ∙Asthma is the result of chronic inflammation of the airways which subsequently results in increased contractability of the surrounding smooth muscles. ∙Prevent by avoiding triggers. ∙Incompletely understood environmental and genetic interactions. ∙Twins 25%.

Tachyarrhythmias:

∙Atrial Fibrillation ∙Atrial Flutter ∙Multifocal Atrial Tachycardia ∙Paroxysmal Supraventricular Tachycardia ∙Wolf-Parkinson-White Syndrome ∙Ventricular Tachycardia ∙Ventricular Fibrillation

Prevention and Treatment:

∙B-adrenergic agent (albuterol): rescue inhaler. ∙Anticholinergic agents (ipratropium bromide)-not FDA approved. ∙Inhaled steroids. ∙Antileukotrienes (montelukastO ∙Long acting B agonists (salmeterol) ∙Theophylline ∙Cromolyn Sodium used in pediatrics when an identified allergen is noted.

Topical Saline:

∙Beneficial in treatment of symptoms of chronic rhinorrhea and rhinosinusitis when used as sole modality or for adjunctive treatment.

Cardiac Output:

∙Blood ejected out the heart each minute. ∙SV (amount of blood ejected from heart with each ventricular contraction)*HR

Myocardial Infarction:

∙Blood flow is interrupted by clot. ∙Heart cells die affecting electrical path. ∙Muscle can no longer contract as it should. ∙Patient at risk for aneurysm or arrhythmia. ∙Look for Troponin I and creatinine kinase MB elevation.

Sarcoid of the Feet (and other bones):

∙Bone involvement in 5% (distal first) ∙Asymptomatic or aches/pain. ∙Symmetrical ∙Not reversible. ∙Steroids may help pain.

Central Sleep Apnea (CSA):

∙Brain not signaling the body to breathe. ∙No airflow because there is no effort to breathe. ∙Sleep may also be disrupted by CSA.

Treatment of COPD:

∙Bronchodilators, oxygen, steroids, antibiotics. ∙Smoking cessation is very important. ∙Weight loss. ∙CPAP once PaO₂ is below 55mm Hg or the PaCO₂ is above 45mm Hg. The use of sedatives and hypnotics should be avoided. ∙Lung transplantation and lung volume reduction.

Aspiration Pneumonia:

∙Bronchopulmonary infection association with orals and gastric contents entering the lung. ∙Mostly associated with those with impaired swallowing. ∙Patient have cough and SOB. ∙Usually RML and RLL pneumonia. ∙Early prophylactic use of antibiotics is controversial because no evidence indicates that bacterial infection plays a role in the initial events. ∙Generally involve anaerobacteria, which normally colonize the upper respiratory passages. →Penicillin G and Clindamycin.

What can patients take to help them stop smoking?

∙Bupropion. ∙Varenicline. ∙Nicotine gum.

Cardiac Index:

∙CO/surface area ∙4-6L per sq meter average. ∙Can increase with exercise.

Diagnostic Test for Heart Disease:

∙CXR ∙EKG ∙ECHO ∙BNP: secreted by then ventricles of the heart in response to excessive stretching of heart muscles.

Intranasal antihistamines:

∙Can be considered first-line treatment ∙Equal or superior to oral second-gen antihistamine ∙Less effective than intranasal corticosteroids in treatment. ∙Side Effects: sedation, can inhibit skin test reactions.

Angioedema:

∙Can occur alone or with urticaria. ∙non-pitting, non-pruritic, well-defined, edematous swelling. ∙Involves subcutaneous tissues (e.g, face, hands, buttocks, gentitals), abdominal organs, or the upper airway (i.e., larynx).

Congestive Heart failure: Chest X-ray

∙Cardiomegaly ∙Congested blood vessels ∙Kerley lines ∙Hilar fullness with haziness

Tobacco is Category __ in pregnancy.

∙Category C. →Adverse effect on fetus (animal reproduction studies).

Second Generation Antihistamines:

∙Certrizine: (Zyrtec) ∙Desloratadine (Clarinex) ∙Fexofenadine (Allegra) ∙Levocertirizine (Xyzal) ∙Loratadine (Claritin)

Angina Symptoms:

∙Chest Pain-substernal →Increased with exercise →decreased with rest →radiated to jaw or left arm ∙Shortness of Breath ∙Diaphoresis ∙Nausea/indigestion ∙Fatigue ∙Palpitations ∙Syncope

Hyperlipidemia:

∙Cholesterol: →HDL →LDL →VLDL ∙Triglycerides ∙Lipoprotein a-not routinely measured ∙C Reactive Protein

Other Obstructive Conditions:

∙Chronic Bronchitis-persistent cough or sputum for more than 3 months over past 2 years. ∙Chronic Bronchiolitis-inflammation of bronchioles. ∙Bronchiectasis-*abnormality of the bronchioles* usually associated with infections.* ∙Cystic fibrosis. ∙Asthma

Mitral Valve Prolapse:

∙Common finding of mid to late *systolic click.* ∙Benign condition in most cases.

Rheumatic Heart Disease:

∙Complication of *streptococcal pharyngitis.* ∙Most common valvular injury is *mitral stenosis.*

Prevention of DVT:

∙Compression devices alone represents inferior prophylaxis for high-risk patients. ∙Standard unfractionated heparin (dose 5,000 units), subQ 2-3 times a day. ∙Coumadin ∙New oral anticoagulants.

Other Interstitial Lung Diseases:

∙Connective Tissue Diseases such as SLE, RA ∙Drug Induced-such as Amiodarone. ∙Diffuse Alveolar Hemorrhage Show + ANCA ∙Vasculitis such as Wegener's Granulomatosis + ANCA ∙Occupational diseases: Pneumoconiosis, Silicosis, Asbestosis, Berylliosis.

CABG

∙Coronary Artery Bypass graft ∙Saphenous vein, Internal Mammary arteries. ∙New Minimally Invasive procedures.

Chest Pain (Substernal Tightness) Differentials: *Musculoskeletal*

∙Costochondritis ∙Pectoral Muscle Strain ∙Rib fracture ∙Cervical of Thoracic radiculopathy ∙Mysitis

Food Allergies:

∙Cow's milk ∙Hen's egg ∙Soy ∙Wheat ∙Peanut ∙Tree nuts ∙Seafood (Shellfish and fish)

Emphysema:

∙Damage to alveoli. ∙Airways collapse during forced expiration. ∙Types: →*Panacinary (or panlobar)* emphysema is related to the destruction of alveoli, because of an *inflammation or deficiency of alpha 1 antitrypsin younger people.* →*Centroacinary (or centrilobar)* emphysema is due to *destruction of terminal bronchioli muchosis*. Elderly people.

Immune System Activities:

∙Defends against invading pathogens. ∙Removes "worn-out" cells and tissue damaged by trauma. ∙Identifies and destroys abnormal or mutant cells. *∙Produces inappropriate immune responses that lead either to allergies or to autoimmune disease.*

Ventricular Tachycardia:

∙Defined as rapid and *repetitive firing of 3 more PVC's in a row.* ∙*QRS complexes are wide varying shapes.* ∙Treatment is based on current ACLS guidelines and whether or not patient is hemodynamically stable.

Atelectasis:

∙Deflated alveoli in part of a lung. ∙Usually from trauma to the lung due to tumor, infection. →Acute →Postoperative →Trauma →Chronic ∙Symptoms: cough, fever, chest pain. ∙Treatment: symptom relief. In post surgical-incentive spirometry, ambulation CPAP.

Sarcoidosis: Diagnosis, Treatment, and Prognosis

∙Diagnosis →CXR occasionally CT or gallium scan. →Biopsy of granulomas-histiocytes, lymphocytes, and multinucleated giant cells. ∙Treatment →Steroids →Cytotoxic agents. →Immunomodulators such as anti malaria agents. ∙Prognosis →Spontaneous resolution within 3 years. →Chronic Disease Progressive to development of pulmonary fibrosis. ∙Stable ∙Progress to other organs.

Dyspnea:

∙Difficult, labored, and unpleasant breathing. ∙Increased Respiratory rate. ∙Use of accessory muscle to breathe. ∙Unable to speak full sentences. ∙Pulmonary and cardiac etiologies.

Disease of the Heart Muscle:

∙Dilated Cardiomyopathy. ∙Hypertrophic Cardiomyopathy. ∙Restrictive Cardiomyopathy. ∙Myocarditis-inflammation of the myocardium.

Anti-Hypertensive Medications:

∙Diuretics ∙ACE Inhibitors ∙ARB's ∙Beta-blockers ∙Calcium Channel Blockers ∙Alpha Blockers ∙Central Alpha 2 agonist ∙Vasodilators

7 Simple Tips to get an accurate blood pressure reading:

∙Don't have a conservation. ∙Put cuff on bare arm. ∙Support arm at heart level. ∙Empty bladder first. ∙Support back. ∙Keep legs uncrossed. ∙Support feet.

Autoimmune Diseases:

∙Due to an overactive immune response. ∙The body usually attacks its own cells. ∙80 to 100 disorders or more. ∙50 million people. ∙Chronic illness with no cure. ∙50% of people not diagnosed. ∙Women>men.

Cardiac Catherization:

∙Dye into inguinal artery. ∙Patient is awake. ∙Complications: stroke, MI, arrhythemias, tamponade, renal failure.

Associated Symptoms of Chest Pain:

∙Dyspnea ∙Diaphoresis ∙Palpitations ∙Syncope ∙Edema ∙Cyanosis

Clinical Findings for Heart Failure:

∙Dyspnea ∙Orthopnea ∙Paroxysmal Nocturnal Dyspnea ∙Pulmonary Edema-rales on exam ∙Fatigue ∙Fluid Retention ∙Abdominal Symptoms ∙Apnea associated with hypoxemia ∙Diaphoresis ∙Jugular Vein Distention

Cardiac Testing:

∙ECG ∙Echocardiogram ∙TEE ∙Stress Test ∙Cardiac Catherization

Stress Testing:

∙EKG's are done before, during and after exercise. *Looking for signs of ischemia.* ∙Patients need to be able to get heart rate to 85% of predicted maximum (220-age). ∙If unable to exercise pharmacologic stress can be done. *Drugs (adenosine, dipyridamole, dobutamine) can be used to increase heart to goal.* ∙Any patient with *positive stress test (EKG changes), undergo cardiac catherization.*

D-dimer

∙Enzyme linked immnunoabsorbant assay (ELISA). ∙It reflects the breakdown of fibrin and thrombolysis. ∙Elevated in >95% of patients with PE. ∙It is useful in excluding PE when it is negative so it has a high negative predictive value. ∙It can be elevated in MI, pneumonia, heart, failure, cancer.

Diseases of the Vasculature:

∙Essential Hypertension. ∙Peripheral Vascular Disease. ∙Deep Venous Thrombosis ∙Chronic Venous Insufficiency ∙Lymphedema

Inducers of Angina:

∙Excercise ∙Cold weather ∙Extreme Moods: anger, stress, excitement ∙Large Meal

Hemoptysis:

∙Expiration of blood from the lung parenchyma or airway. ∙Scant or massive. ∙Causes: →Heart disease. →Lung disease. →Infections. →Trauma. →Iatrogenic from chest tube placement.

Neurogenic Shock:

∙Failure of the sympathetic nervous system to maintain adequate vascular tone. ∙Usually caused by spinal cord injury or severe head injury.

Non-thrombotic PE:

∙Fat Emboli ∙Amniotic Fluid Emboli

Chest Pain (Substernal Tightness) Differentials: *Breast*

∙Fibroadenoma ∙Gynecomastia

Hypertrophic Cardiomyopathy:

∙Genetic-autosomal dominant ∙Clinical expression occurs during periods of rapid growth. *∙Unexplained LVH-often thickened sputum* *∙pathology: myocyte disarray and fiberous tissue* ∙Symptoms: →Syncope →Palpitations →Sudden death ∙Treatment →Surgery if considerable outflow obstruction.

Innate-Inflammation:

∙Goal: phagocytes and plasma proteins to threatened area. ∙Defense by macrophages. ∙Localized vasodilation and edema. ∙Increased capillary permeability. ∙Migration of leukocytes. ∙Marking and destruction of pathogen.

EKG Cardiomyopathy

∙Going of the chart because the leads have thicker muscle to go through.

Sublingual Immunotherapy:

∙Grass allergen tablets reduces symptoms in adults ∙used for grass pollen, improved rhinoconjunctivitis.

Congestive Heart Failure:

∙Heart is unable to meet body's circulatory demands. ∙*Systolic dysfunction (decreased ejected fraction) due to impaired contractility. Usually caused by previous MI resulting in noncontractile heart muscle.* ∙Ejection fraction is measured by echocardiogram or nuclear scan. ∙Diastolic dysfunction is due to impaired ventricular filling. *∙Treatment includes ACE-I, Beta-blockers, loop diuretics and digoxin.*

Non-controllable or modifiable risk factors for heart disease:

∙Heredity ∙Gender: Male>female ∙Age: >55 men>65 women

Essential Hypertension:

∙High blood pressure not caused by other medical condition. ∙Obstructive sleep apnea. ∙Primary aldosteronism. ∙Renovascular hypertension. ∙Cushing's syndrome. ∙Pheochronchromocytoma. ∙Coartation of the aorta. ∙Thyroid disease.

Diagnosis of Heart Disease:

∙Hypertenion, CAD, alcohol abuse, drug abuse and adriamycin. ∙Signs of volume overload. ∙S3 gallop: occurs when a large volume of blood rushes from the LA into LV at start of diastole ∙S4 *∙JVD: Highly specific 95% of the time. Defined at >3cm of elevation above sternal angle.*

Thyroiditis: Graves Disease

∙Hyperthyroidism ∙>women ∙Antibodies stimulated the TSH receptor causing. →Increased size of the gland. →Increased production of thyroid hormone. ∙Treatment. →I₁₃₁ →Antithyroid hormones: PTU →Thyroidectomy.

Cardiomyopathy: Types

∙Hypertrophic ∙Dilated ∙Restrictive Due to: →Ischemia →Genetic problems →Systemic Disease

Thyroiditis: Hashimoto's

∙Hypothyroidism ∙>women ∙Antibodies destroy the thyroid cells. →causing decreased production of hormones. ∙Treatment. →Thyroid replacement.

Obesity Hypoventilation Syndrome:

∙Hypoventilation associated with hypoxia, obesity, and low carbon dioxide. ∙Etiology unknown. ∙Formal criteria for diagnosis of OHS are: →BMI over 30 kg/m² →Arterial carbon dioxide level over 45 mmgHg on ABG →No alternative explanation for hypoventilation. ∙Treatment: diet, exercise, CPAP.

Allergic Rhinitis:

∙IgE response ∙Rhinorrhea, nasal congestion, sneezing, itchy or eyes, ears and nose. ∙Also called: →hay fever. →pollinosis.

Five Subclasses: Antibodies

∙IgM ∙IgG ∙IgE ∙IgE ∙IgA ∙IgD

Chest Pain (Substernal Tightness) Differentials: *Pulmonary*

∙Infection ∙Neoplasm ∙Pneumothorax ∙Embolism ∙Pulmonary hypertension

Infective Endocarditis:

∙Infection of endocardial surface of the heart. ∙Treatment required *extended periods of antibiotics.*

Restrictive Cardiomyopathy:

∙Infiltration of the myocardium results in *impaired diastolic ventricular filling due to decreased ventricular compliance.* ∙Common infiltrates are amyloidosis, sarcoidosis, hemochromatosis, scleroderma, chemotherapy, or radiation induced.

Innate Immunity:

∙Inflammation ∙Interferon ∙Natural Killer Cells ∙Complement System

Introducing peanuts to babies:

∙Introduce foods 4 to 6 months of age. →delayed introduction of solid foods may increase the risk of allergy. ∙baby has tried other foods and tolerated them. ∙Try at home with available anithistamine ∙Gradually increase amount if no problems.

Atrial Fibrillation (A. Fib):

∙Irregular irregularity ∙Problems with atrial conduction. ∙Can cause emboli to brain-stroke. ∙Treat with medication or cardiovert.

Atrial Fibrillation:

∙Irregularly Irregular pattern. ∙*Multiple foci in the atria fire continuously in a chaotic pattern, causing totally irregular, rapid ventricular rate.* ∙Atria quiver continuously. ∙Atrial rate is over 400 but the AV node blocks most impulses for ventricular rate. ∙*Treatment*: initially with *beta blocker or calcium channel blocker.*

Important questions to ask:

∙Is chest pain substernal? ∙Are your symptoms made worse by exercise? ∙Does rest (about 10 minutes) cause prompt relief of pain?

RV, LV or both:

∙LV: CAD, hypertension, alcoholic cardiomyopathy ∙RV: COPD and advanced LV failure. ∙Peripheral edema, JVD, and fatigue in both RV and LV heart failure. ∙Pulmonary edema is only seen in LV failure.

Complications: Hypertension

∙LVH ∙CHF ∙Coronary heart disease ∙Renal Dis

Lung Cancer:

∙Leading cause of cancer related death in US. ∙Most common cause of cancer related deaths. ∙Symptoms: cough, DOE, hemoptysis, weight loss but may present with other symptoms. ∙Small cell and non small cell. ∙Causes tobacco smoke, asbestos and air pollution.

Treatment: Angina

∙Lifestyle Modification ∙Pharmacotherapy to improve symptoms. ∙Antiplatelet therapy ∙MONA B: →*M*orphine →*O*xygen →*N*itroglycerin →*A*spirin →*B*eta-blocker

Treatment in stable angina:

∙Lifestyle modifications →Smoking cessation →Lipid lowering →Glycemic control in DM →exercise ∙Symptomatic: →*B-blockers* or *calcium channel blockers*: decrease oxygen demand. →*Nitrates* to increase oxygen supply. ∙*Inhibit disease progression: aspirin/clopidrogrel.*

DVT:

∙Lower leg swelling, warmth, tenderness and redness. ∙Diagnosis: →Impedance plethysomography →Doppler Ultrasound →MRI

Lymph Nodes:

∙Lymph tissue not surrounded by a capsule. ∙Center of lymph tissue dividing. ∙Purify lymph before returns to the blood.

Lymphadema:

∙Lymphadema occurs when your lymph vessels are unable to adequately drain fluid, usually from an arm to or leg. Occur on its own (primary) or it can be caused by another disease or condition (secondary). Secondary lymphedema is far more common than primary. ∙Any condition that damages you lymph nodes can cause lymphadema.

Chest Pain (Substernal Tightness) Differentials: *Mediastinal* Structures

∙Lymphoma ∙Thymoma

Chest Pain (Substernal Tightness) Differentials: *Cardiac*

∙MI/Angina ∙Pericarditis/Myocarditis ∙Aneurysm

Duke Criteria for Infective Endocarditis:

∙Major Criteria: Positive blood cultures, evidence of endocardial involvement by echocardiography. ∙Minor Criteria: Predisposition, Microbiologic evidence, Fever, Vascular phenomena, echocardiographic findings

Hypertension: Causes

∙Major risk factor for CAD. →Increased intracellular fluid volume →Renal dysfunction →Vasconstriction with

Decongestants:

∙Many side effects ∙combination oral antihistamines + decongestants more effective ∙nasal formulations: intermittent or episodic therapy

Pulmonary Embolism:

∙Material that travels through the lungs though the pulmonary circulation. ∙Most common-clot from a DVT: →Tumor, air fat, amniotic fluid, right ventricle ∙Classic symptoms: *dyspnea, syncope, hypoxia, hypotension.* ∙D dimer: 96% sensitive. ∙ECG changes: ST changes, axis deviation.

Pulse Oximetry:

∙Measures oxygen saturation of a patient's blood. ∙Acceptable normal ranges are from 95 to 100 percent. ∙Oximetry is not a complete measure of respiratory sufficiency. →Insufficient a blood flow. →Increased CO₂

Angioplasty:

∙Mechanically widening the artery narrowed due to arteriosclerosis. ∙balloons are passed into the artery then inflated to a fixed size using water pressures some 75 to 500 times normal blood pressure.

Diagnosis of Asthma:

∙Methacholine challenge. →inhalation of increasing concentration of a substance that causes airway narrowing in those predisposed. →if negative, person does not have asthma; if positive, however, it is not specific for the disease.

Valvular Heart Disease:

∙Mitral Stenosis. ∙Aortic Stenosis-LV outflow obstruction. ∙Aortic Regurgitation. ∙Mitral Regurgitation. ∙Mitral Valve Prolapse ∙Rhematic Heart Disease ∙Infective Endocarditis.

BNP:

∙Molecule produced by ventricular myocytes. ∙Elevated levels are indicative of increased ventricular stress of strain ∙Value <100 pg/ml excludes the diagnosis of congestive heart failure. ∙Obesity tends to attenuate BNP levels for reasons not understood. *∙Value >400 strongly suggests diagnosis of heart failure.*

Hypertrophic Cardiomyopathy:

∙Most cases are inherited. ∙Main problem is *diastolic dysfunction due to stiff ventricle.*

Causes of Heart Failure: Systolic dyfunction

∙Most common *∙66% associated with CAD* ∙Also longstanding hypertension ∙Alcohol abuse

Coronary Heart Disease:

∙Most common cause of mortality in the US. ∙Teens have it too. ∙Atherosclerosis changes in coronary artery. ∙Other causes: emboli, trauma, ateritis, coronary spasm, cocaine, triptans. ∙Increase oxygen demand can cause angina.

Obstructive Sleep Apnea (ASA):

∙Most common form. ∙Occurs when tissues in the throat collapse and block the flow of air in and out of the lungs during sleep. ∙No airflow despite efforts to breathe. ∙Struggling to breath. ∙Restful sleep difficult.

Sarcoidosis:

∙Most common granulomatous lung inflammatory disease. ∙Characterized by noncaseating granulomas in multipke organs esp. the lung. ∙Unknown cause: infections and environmental causes suspected. ∙An immune response causes increase in CD4 cells and formation of fibrosis. ∙Symptoms: →30-60% no symptoms at diagnosis →Lung involment causes restrictive lung disease on PFT, cough and dyspnea. →Various skin lesions →Also cardiac and neuro symptoms 5 to 10% →Electrolyte imbalance.

Cough:

∙Most common respiratory complaint. ∙Acute cough is <8 weeks. ∙Ask about timing of the cough. ∙May be associated with life threatening illness so do not dismiss.

Dilated Cardiomyopathy:

∙Most common type. ∙*Insult* (ischemia with MI, infection, alcohol) to the muscle causes dysfunction or left ventricular contractility. *∙Frequently associated with dysrthymias and sudden death.*

Cardiac Enzymes:

∙Most commonly used test to *"rule out myocardial injury."* ∙*Troponins (T and I)*-increases within 3-5hrs. and returns to normal 5-14 days. ∙*CK-MB-increases within 4-8hrs* and returns to normal in 48-72 hrs. ∙Enzymes drawn on admission and every 8hrs for 24 hrs. ∙Patients with "positive enzymes" are treated for infarction.

Intranasal Corticosteroids:

∙Most effective medication class for controlling symptoms of allergic rhinitis. ∙May be used as-needed bases, but more effective continuous.

Ventricular Fibrillation:

∙Multiple foci in the ventricles fire rapidly, leading to a *chaotic quivering* of the ventricles and no cardiac output. ∙Treatment is defibrillation! Chest compressions until defibrillation.

Tuberculosis:

∙Mycobacterium tuberculosis. ∙Symptoms: cough, blood tinged sputum, fever, night sweats, weight loss. ∙BCG vaccine. ∙PPD testing. ∙Treatment: INH and rifampicin. ∙Quantiferon-new test for TB and latent TB

Innate-Natural Killer Cells:

∙Naturally occurring lymphocyte-like cells. ∙Nonspecifically destroy virus-infected cells and cancer cells (activated by macrophage cytokines). ∙Lyse cell membranes.

DVT Prophylaxis:

∙Non-Pharmacologic: →Intermittent Pneumatic Compression. →Elastic Stockings. →Inferior Vena Cava. ∙Pharmacolgic: →Unfractionated Heparin (UH). →Low Molecular Weight Heparin (LMWH). →Oral Anticoagulants.

Peripheral Vascular Disease:

∙Occlusive athersclerotic disease of the lower extremities. ∙Usually associated with coronary artery disease. ∙Symptoms include pain with exertion or at rest. ∙Ankle to Brachial Index. ∙Systolic BP at ankle to the arm. ∙Normal ABI is between 0.9-1.3 ∙*ABI > 1.3 non compressible vessels and severe disease.* ∙*ABI<0.7 is claudication.*

First-Degree AV Block:

∙PR interval is prolonged (>0.2 sec).

Multifocal Atrial Tachycardia:

∙Patient with severe pulmonary disease such as *COPD.* ∙*Variable P-wave* morphology and variable PR and RR intervals. ∙*At least 3 different P-wave*morphologies are required to make accurate diagnosis.

Pulmonary Embolism: Risk Factors

∙Patients who have undergone a hip or know replacement are at highest risk of all medical and surgical patients. ∙Cancer patient ∙Birth Control Pills ∙Sedentary.

Chest Pain (Substernal Tightness) Differentials: *GI*

∙Peptic ulcer ∙GB ∙Abscess ∙Pancreatitis ∙GERD

Bronchiectasis:

∙Permanent dilation of the bronchial walls. ∙Results in airflow obstruction. ∙Usually results from necrotizing bacterial infections such as Staph, Klebseilla and Pertussis. ∙Diagnosis by CT scan. ∙Treatment goal to prevent infections and control mucus.

Pleural Effusion:

∙Pleura covers the chest wall, diaphragm, mediastinum. Usually contains 5-10mls of fluid to facilitiate expansion of the lung. ∙Pleural effusion develops when more fluid enters the pleural space than is removed. ∙Causes: →Heart Failure-increaased hydrostatic presure →Low albumin-decreased oncotic pressure →Pneumonia-increased permeability →Malignancy-impaired lymph drainage →Ascites-pushes fluid into lung space

Pneumoccoal Vaccine:

∙Pneumovax 23. ∙adults who are increased risk for pneumococcal disease or its complications. ∙Examples: Diabetics, patients with CHF, HIV, patients over 65. ∙The ACIP states that adults who smoke cigarettes are at substantially increased risk for invasive pneumoccoal disease.

Stroke volume is affected by?

∙Preload ∙Afterload ∙Contraction →Inotropy, independent of loading, changed by *catecholamines* or certain medications.

Community Acquired Pneumonia:

∙Present with cough, fever, chills, fatigue, dyspnea, rigors, and pleuritic chest pain. ∙Can be associated with hypoxia in certain patients. ∙can rapidly lead to respiratory failure.

Pulmonary Embolism: Mortality

∙Preventable death. ∙300,00 Americans suffer pulmonary embolism each year. ∙2 percent die withing the first day. ∙The charts of 70 percent of patients dying of pulmonary embolism recorded no suspicion of the diagnosis.

TEE-Transesophageal echocardiogram:

∙Probe inserted into esophagus. ∙Closer view of the heart structures. ∙Invasive-risk of perforation. ∙Sedation. ∙Fasting.

Heart Failure:

∙Problem with the contractility of the heart. ∙Decreased cardiac output to meet needs. ∙Fluids collects in the lungs and extracellular space.

Interpretation of EKG:

∙Rate: <60 Bradycardia, normal, >100 Tachycardia. ∙Rhythm: sinus or other. ∙Axis: normal, right, or left axis deviation. ∙Waves: P, Q, R, T. ∙Intervals: PR, QRS, QT

Asthma:

∙Recurrent reversible lung condition with associated bronchoconstriction and inflammation. ∙Common, ∙Most cases start prior to 25 yrs. ∙Incidence increased 45% since 1970's. ∙90% heritability. ∙Mediators of constriction: →Acetylcholine. →Histamine. →Leukotrienes. →Nitric Oxide.

Immunotherapy: "Allergy Shots"

∙Reduces symptoms ∙Consider for patients with allergic rhinitis who have evidence of specific immunoglobulin E antibodies ∙Insufficient evidence regarding efficacy ∙Fexofenadine pretreatment may present severe systemic reactions

Cardiogenic Shock:

∙Related to MI, serious arrhythmias, or heart failure. Decreased cardiac output. Heart can't pump, tissue perfusion ceases. Vitals: hypotension, urine output decreases, cold, clammy skin, poor peripheral pulses, pulmonary congestion, dyspnea, restless/confused

Septic Shock:

∙Related to endotoxins released by bacteria, which cause vascular pooling, diminished venous return, and reduced CO.

Secondary Hypertension:

∙Renal Disease such as renal artery stenosis.

Atrial Flutter:

∙Saw-tooth pattern. ∙One irritable focus in the atria fires at rate of 250-300 (mostly 300). ∙Every other third impulse to pass onto the ventricle. ∙Treatment similar to a-fib.

Exercise Induced Brochospasm:

∙Shortness of breath or wheezing. ∙Decreased exercise endurance. ∙Chest pan or tightness with exercise. ∙Cough Upset stomach or stomachache ∙Sore throat Symptoms must occur during or following exercise or at least five minutes of duration.

Diagnostic Tests:

∙Skin Testing ∙RAST ∙Enzyme Linked Allergosorbent Test (EAST).

COPD:

∙Slowly progressive, common airway obstruction. →The airways and alveoli *lose their elastic quality.* →The *walls* between alveoli are *destroyed.* →The walls of airways become *thick and inflamed.* →The airways *make more mucus than usual,* which can clog them. ∙First symptom-*cough with thick white mucus.* ∙Not reversible, Not contagious. ∙Correlates with tobacco use.

Controllable or modifiable risk factors for heart disease:

∙Smoking ∙Hypertension ∙Elevated cholesterol (Low HDL and high Lipoprotein a) ∙Obesity ∙Physical Inactivity ∙Type A personality ∙Diabetes Mellitus ∙Elevated homocystine ∙Elevated C reactive Protien ∙Cocaine use

Treatment for Emphysema: Varies for stable patients vs. acute exacerbation

∙Smoking cessation. ∙Inhaled broncodilators. ∙Theophylline ∙Inhaled and Oral Steroids ∙Home Oxygen ∙Surgery →Lung Transplant →Lung Reduction ∙Flu and Pneumococcal vaccine ∙Antibiotics ∙Treat associated GERD and allergies.

Pneumothorax: Types

∙Spontaneous →Primary: men, tall, thin between 10-30 years of age. Subpleural bullae →Secondary to other disease. ∙Iatrogenic →Needle aspiration. →barotrauma ∙Traumatic →Penetrating chest wound.

Ischemic Heart Disease:

∙Stable Angina Pectoris. ∙Unstable Angina. ∙Myocardial Infarction. ∙Variant (Prinzmetal) Angina.

Coronary Stent:

∙Stainless steel tube with slots to widen the artery. ∙After a few weeks epithelium grows over mesh. ∙drug-eluting stents ∙aspirin and clopidogrel

Artherosclerosis

∙Starts in early decades. ∙*Lipoproteins invade intimal layer.* ∙*Monocytes ingest* lipoproteins to become *foam cells.* ∙Plates join and secrete growth factor ∙Smooth muscle forms a fiberous cap. ∙Plagues develops and can rupture causing occlusion.

Pulmonary Edema:

∙Swelling or fluid accumulation in the lungs leading to respiratory failure. ∙Often due to heart failure but can be due to damage to the lungs. ∙Symptoms: SOB, hemoptysis, anxiety, diaphoresis. ∙Treatment: oxygen, diuretics, improve cardiac function.

Delayed Hypersensitivity:

∙T-cell mediated response. ∙Ex.: poison ivy, cosmetics, and household cleaning supplies. ∙Shoe leather dermatitis.

Initial Monotherapy:

∙Thiazide diuretics. ∙Long-acting calcium channel blockers (most often a dihydroproyridine

Cardiogenic Shock:

∙Tissue hyperfusion due to MI or heart failure. ∙Tissue deaths leads to arrethymia and or/insuffieicent pumping of the ventricle ∙Meds to increase perfusion used. ∙Signs →Decreased →Decreased urine output →Hypoxia →Decreased consciousness ∙Often fatal

Interstitial Lung Disease:

∙Topical name for a group of disorders that *lead to fibrosis of the lung parenchyma.* ∙120 distinct entities. ∙*Caused by chemicals such as silicone, antibiotics and other drugs* or may be idiopathic as in sarcoidosis. ∙A *restrictive lung pattern* is seen on PFT.

Echocardiogram:

∙Transthoracic Transducer ∙Non invasive ∙2D picture ∙Assessment of valves and heart size. ∙Assessment of wall motion. ∙Can be used with treadmill testing.

Treatment of CHF:

∙Treat underlying problem. ∙2 gram sodium diet (normal 6-10g/day) ∙Water restriction. *∙Loop Diuretics/thiazide diuretics.* ∙Vasodilators such as an ACE to improve cardiac output. *∙Inotropic agents such as digoxin.* *∙Beta blockers.* ∙Anticoagulation may benefit some ∙Implantable cardiac defibrillators. ∙NB: *NSAID and Calcium Channel* blockers may make HF *worse.*

Immediate Hypersensitivity:

∙Type 1: ∙Sources: pollen, bee sting, penicillin, animal saliva. ∙involve immunoglobulin E (IgE) mediated release of histamine and other mediators from mast cells and basophils. ∙Symptoms-varying →Hay fever →Asthma →Hives →Anaphylaxis

Common causes of Angioedema and Urticaria: Immunological causes

∙Type I, II, III, IV

Shock:

∙Underperfusion of tissues. It is a medical emergency that needs to be treated immediately. ∙Cardiogenic Shock ∙Hypovolemic Shock ∙Septic Shock ∙Neurogenic Shock ∙Anaphylactic Shock

Diagnosis: Food Allergies

∙Unequivocal history of an immediate reaction consisting of typical allergic symptoms. ∙Positive tests for specific immunoglobulin E (IgE) antibodies: →Either skin prick tests →In vitro tests

Treatment of PE:

∙Unfractionated heparin given IV is standard treatment. →Enoxaparin (outpatient treatment) →Drug readily absorbed into the circulating plasma and it undergoes minimal protein binding. →Little data for tx of obese patients.

Pulmonary Embolism: Clinical Features

∙Variable ∙Dyspnea ∙Pleuritic Pain ∙Cough and hemoptysis ∙Tachycardia ∙Fever ∙Hypotension ∙Evidence of DVT.

Prinzmetal Angina:

∙Variant Angina ∙Pain at rest ∙*ECG with chest pain shows ST elevation* and not depression. ∙Often do not have the risk factors. ∙During cath can be provoked by acetylcholine or methacholine. ∙*Treat with Calcium Channel blockers and nitrites* not B-blockers.

Differential Diagnosis:

∙Vasomotor rhinitis ∙gustatory rhinitis ∙nonallergic rhinitis ∙atrophic rhinitis ∙PDE-5 selective inhibitors ∙phentolamine ∙alpha-receptor antagonists ∙nonsteroidal anti-inflammatories including aspirin ∙oral contraceptives or hormonal rhinitis ∙rhinitis medicamentosa ∙infections rhinitis

Pathogenesis of Pulmonary Embolism:

∙Venous Thrombi develop when platelets fibrin and other clotting factors adhere to the vessel wall. ∙Due to stasis or vessel wall trauma. ∙20-35% of people have a genetic disposition. →Most common Factor 5 Leiden and the prothrombin gene mutation.

Dilated Cardiomyopathy:

∙Ventricular dilation and impaired contraction. ∙Left or right ventricles. ∙Develops as a result of infection, toxins, cardiac problems or systemic problems. →ETOH →Chemotherapy-Adriamycin →Cocaine ∙Symptoms: →Atypical chest pain →Decompensation

Deep Vein Thrombosis:

∙Virchow Triad: endothelial injury, venous stasis and hypercoagulable state. ∙Risk factors: age, malignancy, immobilization, obesity, smoking, oral contraceptives. ∙Complication-pulmonary embolism.

Restrictive Cardiomyopathy:

∙Walls of the ventricles become stiff but not necessarily thickened. ∙Ventricles cannot fill adequately during diastole. ∙May have genetic component. ∙Usually associated with medical problems that affect many systems such as amyloidosis of sarcoidosis.

Treatment for Sleep Apnea:

∙Weight loss. ∙CPAP is sued in most patients: →Sneezing and rhinorrhea are common but can be usually alleviated with steroid nasal sprays. →failure to may also be caused by perceived discomfort, claustrophobia and panic attacks. →Uvulopalatopharyngoplasty.

Clinical Findings of Asthma:

∙Wheeze. ∙Sputum. ∙Cough ∙History of allergies. ∙Decreased Peak flow and FEV₁ ∙Pulse Ox ∙Arterial blood gas.

Antibodies:

∙Y-shaped molecules. ∙Four interlinked polypeptide chains-2 long/heavy, 2 heavy short/light. ∙Functional antibody properties determined by properties of tail. ∙Identical antigen-binding fragments (Fab) at tip each arm. ∙Unique Fab on each antibody. ∙Identical tail regions within each subclass.

Inferior vena cava (IVC) filters:

∙diagnosed with deep vein thrombosis (DVT). ∙who are trauma victims. ∙who are immobile. ∙who have recently had surgery or delivered a baby.

Diagnosis of Asthma:

∙diagnosis based on history, physical and spirometry. ∙history →recurrent wheeze →difficulty breathing →tightness in chest →cough ∙symptoms may occur or worsen with common triggers such as allergens at night, possibly awakening patient during exercise. ∙physical exam →wheezing or prolonged phase of forced exhalation. →hyperexpansion of thorax ∙Spirometry →showing obstructive pattern and evidence of reversibility given bronchodialtors. →significant reversibility defined as >10% of predicted forced expiratory volume

Intranasal Cromolyn:

∙effective in some patients for prevention and treatment of allergic rhinitis

Oral Leukotriene Receptor Antagonist:

∙have been useful in treatment of allergic rhinitis. ∙montelukast

High cholesterol:

∙not a disease but a *risk factor.* ∙The *intensity of lowering the cholesterol is proportionate to their risk of heart disease.* ∙The higher the risk of heart disease the *lower the goal. ∙Someone with a previous heart attack has a higher risk of a second heart attack so their goal is the lowest.

Prevalence: Allergic Rhinitis

∙peak prevalence in children and young adults-rare <2 years old, avergae onset 9 years of age. ∙onset before age 20 years 80% cases, onset after age 65 years uncommon. ∙symptoms often diminish with older age.

Urticaria:

∙polymorphic, round or irregularly shaped pruritic wheals. ∙lesions can appear hyperemic in the center with a white halo along the circumference. ∙Size: varies ∙Histamine released from cutaneous mast cells and basophils in response to inciting stimuli. ∙Lab evaluation of little value.

Complications:

∙sinusitis ∙conjunctivitis ∙sleep apnea ∙increased facial length ∙poor sleep ∙high arched plate ∙dental malocclusion ∙school absenteeism ∙Impaired cognitive functioning.

History:

∙symptoms may have occurred within minutes of exposure to know or suspected allergen, producing "early phase" allergic response. ∙ask: pattern, seasonality, chronicity ∙environmental history ∙identification of precipitating factors ∙related symptoms: post nasal drip, loss of smell and taste, chronic cough, epistaxis

Serum Sickness:

∙type II hypersensitivity reaction that results from the *injection of heterogenous or foreign protein or serum.* ∙body then develops an immune response against the antiserum. ∙Symptoms of *fever, rash, lymphandenopathy, polyarthralgias.*

Acromegaly Hx/PE

■ *Bitemporal hemianopia* may result from compression of the optic chiasm by a pituitary adenoma. ■ Excess GH may also lead to *glucose intolerance or diabetes.*

Hyponatremia Dx

■ *Low (< 280 mEq/L)*: Applies to the majority of cases. Hypotonic etiologies are listed in Table 2.16-1.

Irritable bowel syndrome Dx

■ A diagnosis of exclusion based on clinical history. ■ Tests to rule out other GI causes include CBC, TSH, electrolytes, stool cultures, abdominal films, and barium contrast studies. ■ Manometry can assess sphincter function.

Hiatal hernia

■ Herniation of a portion of the stomach upward into the chest through a diaphragmatic opening. There are two common types: ■ *Sliding hiatal hernias (95%)*: The gastroesophageal junction and a portion of the stomach are displaced above the diaphragm. ■ *Paraesophageal hiatal hernias (5%)*: The gastroesophageal junction remains below the diaphragm, while a neighboring portion of the fundus herniates into the mediastinum.

Polymyositis Tx

■ High-dose corticosteroids with taper after 4-6 weeks to ↓ the maintenance dose. ■ Azathioprine and/or methotrexate can be used as steroid-sparing agents.

Irritable bowel syndrome History/ PE

■ Patients present with abdominal pain, a change in bowel habits (diarrhea and/or constipation), abdominal distention, mucous stools, and relief of pain with a bowel movement. ■ IBS rarely awakens patients from sleep; vomiting, significant weight loss, and constitutional symptoms are also uncommon. ■ Exam is usually unremarkable except for mild abdominal tenderness.

Cholelithiasis History/ PE

■ Patients present with postprandial abdominal pain (usually in the RUQ) that radiates to the right subscapular area or the epigastrium. ■ Pain is abrupt; is followed by gradual relief; and is often associated with nausea and vomiting, fatty food intolerance, dyspepsia, and flatulence. ■ Gallstones may be asymptomatic in up to 80% of patients. Exam may reveal RUQ tenderness and a palpable gallbladder.

Gastroesophageal reflux disease Dx

■ The history and clinical impression are important. ■ An empiric trial of lifestyle modification and medical treatment is often attempted first. Studies may include barium swallow (to look for hiatal hernia), esophageal manometry, and 24-hour pH monitoring. ■ EGD with biopsies should be performed in patients whose symptoms are unresponsive to initial empiric therapy, long-standing (to rule out Barrett's esophagus and adenocarcinoma), or suggestive of complicated disease (e.g., anorexia, weight loss, dysphagia/odynophagia).

Gastritis Dx

■ Upper endoscopy can visualize the gastric lining. ■ H. pylori infection can be detected by urease breath test, serum IgG antibodies (which indicate exposure, not current infection), H. pylori stool antigen, or endoscopic biopsy.

Polymyositis Dx

■ ↑ serum CK and anti-Jo-1 antibodies are seen (see Table 2.9-4). ■ Muscle biopsy reveals infl ammation and muscle fi bers in varying stages of necrosis and regeneration.

Gastritis Tx

■ ↓ intake of offending agents. Antacids, sucralfate, H2 blockers, and/or PPIs may help. ■ Triple therapy (amoxicillin, clarithromycin, omeprazole) to treat H. pylori infection. ■ Give prophylactic H2 blockers or PPIs to patients at risk for stress ulcers (e.g., ICU patients).

Hiatal hernia Tx

■*Sliding hernias*: Medical therapy and lifestyle modifications to ↓ GERD symptoms. ■ *Paraesophageal hernias*: Surgical gastropexy (attachment of the stomach to the rectus sheath and closure of the hiatus) is recommended to prevent gastric volvulus.

Peptic Ulcer Disease History/ PE

■Classically presents with chronic or periodic *dull, burning epigastric pain that improves with meals* (especially duodenal ulcers), worsens 2-3 hours after eating, and can radiate to the back. ■ Patients may also complain of nausea, hematemesis ("coffee-ground" emesis), or blood in the stool (melena or hematochezia). ■ Exam may reveal varying degrees of *epigastric tenderness* and, if there is active bleeding, a stool guaiac. ■ An acute perforation can present with a rigid abdomen, rebound tenderness, guarding, or other signs of peritoneal irritation.

acute decompensated heart failure Tx "LMNOP" for congestion

"LMNOP" lasik IV (decrease fluid volume) morphine (venodilator, decrease afterload) nitrates (venodilator, reduce preload/after load) oxygen +/- vent position (sitting up and legs dangling over side of bed)

Syncope:

"Passing out", loss of consciousness or fainting

Transudate vs. exudative pleural effusions?

(Light's criteria) Exudative: Pleural fluid to serum protein ratio > 0.5 Pleural fluid to serum LDH ratio > 0.6 Pleural fluid to serum upper limits of normal LDH ratio > 0.6

1st line treatments for chronic asthma vs. chronic COPD?

**Major point** Chronic asthma - inhaled steroids Chronic COPD - LAMA (long acting muscarinic antagonist) No real place for steroids in COPD

Pleural Effusion

*Decrease tactile fremitus compared to pneumonia that has increase tactile fremitus*

Shock Work up

*History, history, History* CBC, CMP, UA, Trop, Coag Panel, hCG, ABG, Lactate CXR EKG Cultures

Tx for Hypertensive Crises

*Hypertensive urgencies*: Can be treated with oral antihypertensives (e.g., β-blockers, clonidine, ACEIs) with the goal of gradually lowering BP over 24-48 hours ■ *Hypertensive emergencies*: Treat with IV medications (labetalol, nitroprusside, nicardipine) with the goal of lowering mean arterial pressure by no more than 25% over the first two hours to prevent cerebral hypoperfusion or coronary insufficiency.

What are the diagnosis of Hypertensive Crises?

*Hypertensive urgency*: Diagnosed on the basis of an elevated BP with only mild to moderate symptoms (headache, chest pain, syncope) and without end-organ damage. ■*Hypertensive emergency*: Diagnosed by a significantly elevated BP with signs or symptoms of impending end-organ damage such as ARF, intracra-nial hemorrhage, papilledema, or ECG changes suggestive of ischemia or pulmonary edema. ■ *Malignant hypertension*: Diagnosed on the basis of progressive renal failure and/or encephalopathy with papilledema.

CVA Workup and DDX

*Hypoglycemia*-CHO CBC, INR/aPTT, BMP EKG,Trop Stat Head CT Carotid US ECHO

Epidural Hematoma

*Lucid Interval* MMA Biconvex May be relieved by burr hole

What type of pleural effusion does TB present with? Next step in diagnosis?

*Lymphocyte predominatnt* exudative effusion Pleural biopsy

Legionella Pneumonia

*Spread through contaminated water (Air conditioning system)* Involves the GI tract

takotsubo Sx

*acute substernal chest pain dyspnea syncope tachyarrhythmias bradyarrhythmias significant mitral regurgitation cardiogenic shock (~10%) -hypotension -abnormal mental status -cold extremities -oliguria -respiratory distress note: pheochromocytoma is suspected in pts w/ HA, sweating, and tachycardia w/ or w/out HTN

Left Sided Heart Failure:

*decrease ejection of blood into the circulatory syste*m, increase in it ventricle & atrial pressure and *congestion of pulmonary circulation.*

IgA:

*found in milk, tears, and the respiratory, digestive and GI* tracts.

endocarditis etiology

*most pts usually have an underlying regurgitant cardiac defect that provides a nest for development of vegetation *occurs at areas of endocardial injury (turbulence, trauma, inflammation) native valve infections -viridans streptococci -S. aureus -enterococci -HACEK organisms Haemophilus Actinobacillus actinomycetamcomitans Cardiobacterium hominis Eikenella corrodens Kingella species IV drug users -S. aureus -tricuspid valve prosthetic valve during 1st 2 months after implantation -S. aureus -gram negatives -fungi prosthetic valve after 1st 2 months -strep or staph

Second-Degree AV Block: Mobitz I

*∙Progressive prolongation of PR interval until a P wave fails to conduct.* ∙Site of block is usually within the *AV node.* ∙Benign condition that does not require treatment.

Innate-Interferon:

*∙Transiently inhibits multiplication of viruses in most cells.* ∙Triggers production of virus-blocking enzymes released non-specifically from any cell infected by a virus. ∙Enhances macrophage phagocytic activity, stimulates production of antibodies, boosts power of killer cells. ∙Anticancer effects.

Heart sounds +S3, +S4

+S3 (CHF) +S4 (LVH, infarction)

Outpatient atrial fibrillation management:

- CCBs (diltiazem, verapamil) are used as anti-arrhythmic of choice (they slow AV conduction) - Anticoagulants to reduce the formation of mural thrombus (Warfarin) or if CI - plavix

in UA/NSTEMI; Catheterization for angiography if <12h of onset and:

- Cardiac enzymes come back positive (dx NSTEMI) - Pt has had prior intervention (PCI, CABG), has CHF, CKD, hypotension, or prolonged pain [administer clopidogrel and a GpIIb/IIIa inhibitor if you are going to do angiography and potentially PCI]

Associations of H. Pylori?

- Chronic gastritis - Peptic ulcers (70% H. Pylori/30% NSAIDs) - Duodenal ulcers (100% H. Pylori) - Gastric adenocarcinoma

R wave progression

- In the precordial leads, you should start mostly negative (V1) and progress to mostly positive (V6) - Also, as you progress from **The normal R wave transition occurs between V3 and V4** If you have early R wave progression, it can be caused by: - Posterior MI - RVH - RBBB - WPW If you have late R wave progression, it can be caused by: - Anterior MI - LVH - LAFB (left anterior fascicular block) - LBBB - Lung disease

Sarcoidosis diagnosis?

- Increased ACE - Hypercalcemia (increased 1alpha hydroxylase activation of Vitamin D in macrophages) - Noncaseating granulomas - Bilateral Hilar adenopathy Important associations: - Uveitis - Erythema nodosum - Bells Palsy - Parotid gland enlargement

Aortic Regurigation

-AR is caused by anything that make the "♥" dilate in size: MI, Hypertension, Endocarditis, Marfan Syndrome or cystic medical necrosis. -AR best heard lower left sternal border. -Other tx: ACEi/ARBs, Digoxin and diuretics -Diagnostic Test : ECHO

Esophageal cancer Tx

-Best initial therapy: *Surgical resection*. Surgery is performed if there are no local or distant metastases. -Follow surgery with *chemotherapy based on 5-fluorouracil.*

Pulmonary hypertension etiology

-COPD or fibrosis may ↑Pulmonary artery pressure. -An abnormal increase in pulmonary arteriolar resistance leads to thickening of pulmonary arteriolar walls -Hypoxemia causes vasoconstriction of the pulmonary pressure as a normal reflex which can cause poor oxygenation. This result in more hypoxemia.

chronic bronchitis and emphysema:

-Chronic bronchitis is a clinical diagnosis: chronic cough productive of sputum for at least 3 months per year for at least 2 consecutive years. -Emphysema is a pathologic diagnosis: permanent enlargement of air spaces distal to terminal bronchioles due to destruction of alveolar walls

Valvular Disease: Deep Venous Thrombosis (DVT) Dx

-Doppler ultrasound; spiral CT or V/Q scan may be used to evaluate for pulmonary embolism. -A negative D-dimer test can be used to rule out the possibility of pulmonary embolism in low-risk patients

Esophageal cancer presents with the following:

-Dysphagia: Solids first, liquids later -May have heme-positive stool or anemia -Often found in patients > 50 who are smokers and drinkers of alcohol

MI other Tx

-Give nitroglycerin (NTG) to vasodilate ... unless pt is hypotensive • Lowers preload • Can be given transdermal, sublingual, or IV -Give Aspirin - 162 to 325mg • Doesn't prevent clot from breaking up but prevents it from growing any bigger • Also give (81mg) every day AFTER a heart attack to prevent more clot formation -Thrombolytics!!! - (TPA, streptokinase) breaks up the clot; give within 30 minutes • Biggest risk is that pt can bleed out

Myocardial infarction

-Infarct = death of myocardial cells d/t obstruction of blood flow -Normally an ST segment elevation MI (AKA STEMI) *ST elevation (in localized in the lead) is myocardial injury/infarction until proven otherwise*

Gastroenteritis Presentation

-Inflammatory diarrhea will have fever, abdominal pain, and possible bloody diarrhea. -Noninflammatory diarrhea will have vomiting, crampy abdominal pain, and watery diarrhea.

Mitral Valve Regurigation

-MR is caused by anything that make the "♥" dilate in size: MI, Hypertension, Endocarditis. -Murmur (radiate to the axilla), which is pansystolic (holosystolic), obscuring the S1 and S2. -Handgrip will worsen murmur by pushing more blood backwards through the valve. -Squatting and leg raising will worsen by ↑ Venous return.

Mitral Valve Stenosis

-Most often caused by *Rheumatic fever* -Presentation: *Dysphagia* from left atrium (LA) pressing on the esophagus. *Hoarseness* (LA pressing on larnygeal nerve). *Atrial Fibrillation* and *Stroke* Hemoptysis -Diagnostic Test : ECHO

Aortic Stenosis

-Murmur is heard best at the right intercostal space, and radiate to the carotid artery. -Presentation: Angina, Syncope, CHF -Diagnostic Test : ECHO

Hyperlipidemia Risk factors

-Saturated fatty acids and cholesterol cause elevation in LDL and total cholesterol. -Age-Cholesterol levels increase with age until approximately age 65. The increase is greatest during early adulthood-about 2 mg/dL per year. -Inactive lifestyle, abdominal obesity -Family history of hyperlipidemia -Gender-Men generally have higher cholesterol levels than do women; when women reach menopause, cholesterol levels then equalize and may even be higher in women than in men.

Gastroenteritis Diagnostic Tests

-Send stool for blood and leukocyte count to detect the presence of invasive toxins. -Stool cultures with O&P for identifying the causative agent -Possible sigmoidoscopy to examine for pseudomembranes in the setting of C. difficile

MI PEX:

-Tachycardia, bradycardia, or nml HR -Hypertension, hypotension (be concerned for shock), or nml BP -Sweating, anxious, JVD, irregular heartbeat -Kussmaul sign (JVD does not decrease w/inspiration - think MI is in the R ventricle ) -Rales (if associated HF)

heart murmurs Tx

-most tx early w/ surgical repair -many anomalies required staged procedures as the pt grows -interventions such as extracorporeal membrane oxygenation and alprostadil (prostaglandin E1) to maintain a patent ductus can be helpful in stabilizing infants w/ cyanotic dz prior to surgery -indomethacin to close PDA

takotsubo aka

-stress-induced cardiomyopathy -broken heart syndrome -apical ballooning syndrome

"Which of the following is most likely to be associated with/found in this pt?"

-symptoms worse at night -Nasal polyps -Eczema or atopic dermatitis on PE -*increased length of expiratory phase* -Increase use of accessory respiratory muscles

takotsubo is characterized by...

-transient regional systolic dysfuction of the left ventricle, mimicking myocardial infarction -absence of angiographic evidence of obstructive coronary artery disease or acute plaque rupture

chronic venous insufficiency (stasis dermatitis) Tx

-wet compresses -hydrocortisone cream -zinc oxide w/ ichthammol and antifungal cream

...TBC...valvular dz

...

COPD is the presence of SOB from lungs destruction decreasing the elastic recoil of the lungs

...

Cholecystitis

...

Chrons vs Ulcerative colitis?

...

Delta delta gap

...

In patients diagnosed with acute bronchitis, inhaled b2-agonist therapy (albuterol).

...

Infarction, cardiomyopathy, and valve disease account for the vast majority of cases

...

Patients with chronic bronchitis are "*blue bloaters*," because secondary development of cor pulmonale causes cyanosis and peripheral edema; patients with emphysema are "pink puffers " because of their pursed-lip breathing, dyspnea, and barrel -chests.

...

Elevated troponin levels?

0.5 - 5.0

Dosing of tPA

0.9 mg/kg IV, max dose of 90 mg. 10% administered as bolus with rest infused over next 60 minutes

how long should u treat the third stage of lyme disease?

1 month

Patients with HIV+Tb require therapy for _____

1 year

takotsubo all 4 diagnostic criteria

1) transient left ventricular systolic dysfunction 2) absence of obstructive coronary dz or angiographic evidence of acute plaque rupture 3) new EKG abnormalities (ST elevation and/or T wave inversion) OR modest elevation in cardiac troponin 4) absence of pheochromocytoma or myocarditis

What is the typical clinical presentation of CAP?

1-10 day hx of increasing cough, purulent sputum, SOB, tachycardia, pleuritic chest pain, fever or hypothermia, sweats, and rigors

Bronchiectasis Tx

1. Antibiotics for acute exacerbations 2. Bronchial hygiene is very important.

Cor pulmonale Dx

1. CXR: enlargement of the RA, RV, and pulmonary arteries 2. ECG: right axis deviation, P pulmonale (peaked P waves), right ventricular hypertrophy 3. Echocardiogram: right ventricular dilatation, but normal LV size and function; useful in excluding LV dysfunction

Best first and second test for chest pain?

1. EKG 2. Cardiac enzymes

Causes of hyperkalemia in DKA?

1. Extracellular shift of K+ in exchange for H+ to go intracellular (b/c of the acidosis) 2. Impaired insulin-dependent cell entry of the K+ ion

What drugs most commonly cause C. Diff?

1. Fluoroquinolones 2. Cephalosporins 3. Clindamycin 4. PPIs

BNP levels do:

1. Give an assessment of LVF 2. Increase w severity of CHF 3. Correlate w LV EDP 4. Correlate w pulmonary capillary wedge pressure

Goodpasture's syndrome?

1. Glomerulonephritis (nephritic) 2. PUlmonary hemorrhage Linear deposition of the antiglomerular basement membrane IgG antibody

When starting Coumadin after a DVT, what else to start?

1. Heparin for at least 5 days. 2. Warfain should be started at the same time that heparin is administered and the 2 drugs should be overlapped until the INR reaches >2 measured on 2 occasion approx. 24 hours apart.

Treatment of hyperkalemia

1. IV Calcium Gluconate --> Stabilizes membrane potentials of cardiac myocytes 2. Insulin (+ glucose) --> drives K+ intracellulary 3. Diuretics (K+ wasting --> Lasix)

Types of Hemorrhagic Stroke

1. Intracerebral (10%): results from rupture of small arterioles 2. Subarachnoid (3%): rupture of arterial aneurysms (hemorrhage into subarachnoid space)

Pituitary adenoma Dx

1. MRI is the imaging study of choice. 2. Pituitary hormone levels

Cardiac enzymes?

1. Myoglobin - rises 1st, peaks in 2 hrs, normal by 24 hrs 2. CKMB - rises in 3-5 hrs, peaks 24 hrs, normal by 72 hrs. 3. Troponin I - rises in 3-5 hrs, peaks 24 hrs, normal by 7-10 days CKMB best for diagnosis of reinfarction

What are the three components of asthma?

1. Obstruction 2. Bronchial hyperactivity 3. Inflammation of airway

Tests for TB?

1. PPD 2. Chest X-ray 3. Acid fast stain of sputum

Major causes of seizures in adults? Young people?

1. Post stroke 2. Space occupying lesions 3. Degenerative disorders 1. Drugs 2. Alcohol withdrawal 3. Childhood seizure 4. Electrolytes

What are the only things to improve mortality in COPD? Why is our goal for O2 sat 94-95% instead of 100%? Important vaccinations for COPD?

1. Quitting smoking 2. Continuous O2 therapy > 18 hrs/day COPDers are chronic CO2 retainers. Hypoxia is the only drive for respiration. In COPDers they have a hypoxic drive to breathe based on low O2 as opposed to hypercapnia (which normal people have) Pneumococcus with 5 year booster and yearly influenza

1st line treatment for atrial fibrillation? If hemodynaically unstable?

1. RATE CONTROL (Dilitiazem Non DHP calcium channel blocker or Beta blockers) 2. Anticoagulation with Coumadin **RHYTHM CONTROL HAS NO EFFECT ON MORTALITY** **If patient is hemodynamically unstable --> direct cardioversion* 3. Digoxin

How do you test for H. pylori?

1. Rapid urease test: analyzes tissue samples obtained from endo for presence of urease which is marker for H. pylori 2. Histoligic staining 3. Serologic and fecal antigen tests: Fecal test for cure. Serologic tests good in younger individuals, but once you test positive, you always test positive. 4. Urea breath test

What are the three main goals of treatment for cor pulmonale?

1. Reduction of RV afterload (ie, reduction of the pulmonary artery pressure) 2. Decrease RV pressure 3. Improvement of RV contractility

The four major determinant of systolic function of the heart:

1. The contractile state of the myocardium 2. The preload of the ventricle 3. The afterload applied to the ventricles 4. The heart rate

What are the types of Ischemic Stroke?

1. Thrombotic: narrowing of damaged vascular lumen by an in situ process, usually clot. 2. Embolic (20%): obstruction of normal vascular lumen by intravascular material from remote source

3 causes of extremely elevated LFTs?

1. Toxic (tylenol) 2. Shock liver (perfusion defect) 3. Viral hepatitis (Hep A, Hep B, CMV, EBV, HSV)

Tests for H. Pylori?

1. Urea breath test 2. Stool antigen test (off of PPI) 3. Serum antibody test (can do this if pt. is on PPI) 4. Biopsy (endoscopy)

Potential classes of anticoagulation for afib/aflutter?

1. Warfarin 2. Direct thrombin inhibitors (Dabigatran (oral) 3. Factor X inhibitors (Rivaroxaban, Apixaban)

statin guidelines 3 groups most likely to benefit?

1. clinical atherosclerotic CVD -hx of CAD, MI, un/stable angina -coronary/other arterial revascularization -CVA/TIA -peripheral arterial dz 2. LDL > 190 mg/dl -targeting familial hypercholesterolemia -high-intestiny statins (atorva, rosuva; lower LDL by > 50%) 3. diabetic, ag 40-75, LDL 70-189 -calculate 10 yr risk of atherosclerotic CVD -if risk > 7.5% use high intensity statin -if risk < 7.5% use mod intensity statin (lowers LDL 30-50%)

2 major compensatory mechanisms for valvular disease

1. hypertrophy (from increased volume) in stenotic dz (normal LV wall thickness is < 12 mm) 2. dilation (from increased volume) in regurgitant dz

peripheral arterial disease 6 Ps of extremity occulsion

1. pain 2. pallor 3. pulselessness 4. paresthesias 5. poikilothermia 6. paralysis note: also cool skin and abnormal hair growth

virchow's triad

1. stasis 2. vascular injury 3. hypercoagulability

valvular heart disease can be split into two major types

1. stenosis 2. regurgitation/insufficiency

tetralogy of fallot 1. 2. 3. 4.

1. ventricular septal defect 2. aorta arising out of both ventricles (overriding aorta) 3. right ventricular outflow obstruction (pulmonary stenosis) 4. right ventricular hypertrophy

What change in FEV1 after bronchodilation is supportive of the diagnosis of asthma?

10%

Epistaxis Posterior

10%, Elderly, coagulopathy *Sphenopalatine artery* Posterior packing, ENT consult, admit, antibiotics

what is considered a positive tb test for healthcare workers

10mm or more of induration

Pt that are DM, are screen for microalbuminuria every_________months.

12

Diabetic pt or someone with chronic renal disease hypertension is

130/80 mmHg

how long should treatment be for the first stage of lyme disease

14 days

Goal BP in pts. > 60? Goal BP in pts. <60, CKD, stroke, diabetes?

150/90 140/90

Target BS for diabetics in the hospital?

180

What is empiric first line abx tx for acute exacerbations of chronic bronchitis? 2nd line?

1st line: 2nd gen cephalosporin 2nd line: 2nd gen macrolide or Bactrim

Duke criteria for endocarditis?

2 major criteria OR 1 major, 3 minor OR 5 minor Major criteria: 1. 2 postitive blood cultures drawn 12 hours apart 2. Evidence for vegetation on ECHO 3. New murmur Minor criteria: 1. Predisposing factor 2. Fever 3. Immunologic phenomenon (glomerulonephritis) 4. Embolism (janeway lesions, osler nodes, roth spots 5. Microbiological evidence that doesn't meet major criteria

how do you treat tb in areas of low drug resistance

2 months of INH, followed by 4 months of INH and rifampin. pyrazinamide commonly added

endocarditis Duke Major criteria

2 positive blood cultures of a typically causative microorganism evidence of endocardial involvement on echo development of a new regurgitant murmur

What range of time does it take for a PPD test to become positive as an immune response?

2-12 weeks

how long should treatment be for the second stage of lyme disease

21 days

What % of effusions are associated with malignancy?

25%

SCLC (oat cell) accounts for how many cases? What are characteristics?

25-35% - more likely to spread early and be more aggressive. Mean survival is 6-18 weeks if untreated.

endocarditis Dx

3 sets of blood cultures at least 1 hr apart (4 antbx) TEE -presence of vegetation is diagnostic CXR -maybe cardiac abnormality -maybe pulmonary infiltrates if right heart involved EKG -no specific features anemia leukocytosis elevated ESR hematuria proteinuria positive rheumatoid factor antibody

What is the time limit of tPA in stroke?

3-4.5 hours

in which stage of lyme disease would you see neurocognitive dysfunction and peripheral neuropathy

3rd, late infection

in which stage of lyme dz would you see migratory polyarthritis

3rd. late infection

How long do you bridge with LMWH?

5 days with INR >2 for at least 24 hours

Acute Renal Failure

50% increase in baseline creatinine. Deterioration in renal function (GFR) leads to excessive accumulation of nitrogenous waste products in serum (urea, creatinine) referred to as azotemia. Patients categorized as prerenal, renal, or postrenal.

Does UC or Crohn's involve perianal area?

50% of the time when colon involved with Crohn's disease; never in UC

at what cd4 count do u start to see immunocompromisation

500 or less

what is a positive ppd for hiv pts or people in close contact with tb

5mm or greater of induration

what is a normal cd4 count

600-1500

Ruptured saccular berry aneurysm counts for _____% of nontraumatic cases of subarachnoid hemorrhage.

75%

Airway obstruction exists when the peak flow is <___% of predicted value based on patient's age, gender, height

80%. Greater than 10% increase in FEV1 after bronchodilator therapy is supportive of diagnosis of asthma.

Tx for sarcoidosis?

90% are responsive to steroids

Chronic Pancreatitis causes

90% caused by alcohol abuse

Epistaxis Anterior

90%, Trauma, dry air, URI, infection *Kiesselbach's Plexus* Look for source, Cautery, Gelfoam/Surgicel, Packing, ENT follow up

Cholecystitis

95% Caused by Cholelithiasis 5Fs- * Fat, Forty, Female, Fertile, and Fair* Sudden RUQ pain, radiates to back, associated with fatty meals, N/V, *Arrest of inspiration on palpation RUQ-Murphy's sign*

JNC 8 hypertension Tx Algorithm all ages w/ CKD BP goal is ___ Tx (all races)

< 140/90 mm Hg Tx (all races) ACEI OR ARB OR combination w/ other class

JNC 8 hypertension Tx Algorithm all ages w/ DM but no CKD BP goal is ___ Tx (black) ___ Tx (non-black) ___

< 140/90 mm Hg Tx (black) thiazide-type diuretic OR calcium channel blocker OR combination Tx (non-black) thiazide-type diurectic OR ACEI OR ARB OR calcium channel blocker OR combination

JNC 8 hypertension Tx Algorithm < 60 yo and no DM/CKD BP goal is ___ Tx (black) ___ Tx (non-black) ___

< 140/90 mm Hg Tx (black) thiazide-type diuretic OR calcium channel blocker OR combination Tx (non-black) thiazide-type diurectic OR ACEI OR ARB OR calcium channel blocker OR combination

JNC 8 Hypertension Tx Algorithm > 60 yo and no DM/CKD BP goal is ___ Tx (black) ___ Tx (non-black) ___

< 150/90 mm Hg Tx (black) thiazide-type diuretic OR calcium channel blocker OR combination Tx (non-black) thiazide-type diurectic OR ACEI OR ARB OR calcium channel blocker OR combination

Interpretation of BNP levels:

<100 pg/mL - CHF excluded 100-400 - inconclusive >400 pg/mL - diagnostic of CHF

Guidelines for platelet transfusion?

<20,000 + clinical signs of bleeding <10,000 regardless, due to risk of spontaneous intracranial hemorrhage

Viral meningitis opening pressure of LP

<300mm

AAA <4cm and AAA >7cm

<4cm: medical management >7cm: surgical management

TB tests positive? > 15 mm > 10 mm > 5 mm

> 15 mm - normal people with no risk factors > 10 mm - prison, healthcare worker, nursing home, DM, alcoholic > 5 mm - HIV, immunosuppression

indications for operation for carotid artery dz

> 75% stenosis > 70 % stenosis + Sx bilateral dz + Sx > 50 % stenosis + recurring TIAs despite aspirin tx

Bacterial meningitis opening pressure of LP

>300mm

Who is at increased risk for a malignant solitary pulmonary nodule?

>45 yo Smokers

Chronic Hepatitis

>6 weeks, usually results from viral infection (Hep B, C, D) or inherited disorders (Wilson's disease), autoimmune dz of liver

Sarcoidosis

A 25-year-old black woman presents with nonproductive cough, shortness of breath, fatigue, and malaise; she has bilateral hilar lymphadenopathy on chest radiography and elevated ACE levels. What do you diagnose?

Children, adolescents, and immunocompromised who have been in close contact with a person with active TB should be offered treatment until....

A TST is negative in 12 weeks after exposure

What is Sarcoidosis?

A chronic systemic granulomatous disease characterized by noncaseating granulomas, often involving multiple organ systems. Lungs are almost always involved. Etiology unknown.

What is Chronic obstructive pulmonary disease (COPD)?

A clinical and pathophysiologic syndrome that includes emphysema and chronic bronchitis.

BP meds in CHF:

A combination of a diuretic and and ACEi are initial tx in most symptomatic patients; CCBs can worsen CHF and should be avoided

What is bronchiectasis?

A disease caused by cycles of infection and inflammation in the bronchi/bronchioles that lead to fibrosis, remodeling, and *permanent dilation of bronchi*.

Atherosclerosis

A disorder in which cholesterol and calcium build up inside the walls of the blood vessels, forming plaque, which eventually leads to partial or complete blockage of blood flow.

Diabetes Mellitus Type 2 Hx/Dx

A dysfunction in glucose metabolism that is best characterized as varying degrees of insulin resistance that may lead to β-cell burnout and insulin dependence.

Treatment for patients with A fib undergoing PCI with stenting?

A fib + stent = triple therapy = DAPT + coumadin

Tx of atrial fibrillation in emergency settings:

A fib w hemodynamic instability: cardioversion A fib w rapid ventricular response: CCB Stroke/TIA - stroke protocol

A fib with RVR?

A fib with rapid ventricular rate is basically when you have a fib with a ventricular rate >100 bpm.

Hypoparathyroidism USMLE

A patient presents with signs of hypocalcemia, high phosphorus, and low PTH.

Anaphylactic Shock:

A severe reaction that occurs when an allergen is introduced to the bloodstream of an allergic individual. Characterized by *bronchoconstriction, labored breathing, widespread vasodilation, circulatory shock, and sometimes sudden death.*

Solitary Pulmonary Nodule

A single, well-circumscribed nodule seen on CXR with no associated mediastinal or hilar lymph node involvement

Valvular Disease: Aortic Dissection

A transverse tear in the intima of a vessel that results in blood entering the media, creating a false lumen and leading to a hematoma that propagates longitudinally. Most commonly 2° to hypertension. The most common sites of origin are above the aortic valve and distal to the left subclavian artery. Most often occurs at 40-60 years of age, with a greater frequency in males than in females.

JNC 8 hypertension Tx Algorithm stragety

A) maximize first med before adding another B) add second med before reaching max dose of first med C) start w/ 2 medication classes separately or as fixed dose combination if still not at goal... for A) and B) try medication class not previously selected for C) max out dose of initial meds if still not at goal... add thiazide-type diuretic OR ACEI OR ARB OR CCB if still not at goal... add 4th medication class (beta blocker, aldosterone antagonist, others) and/or refer to specialist AVOID using ACEI and ARB together

A. The best initial test to diagnosis bronchiectasis? B. What is the most accurate test for bronchiectasis?

A. CXR B. CT Scan In addition , sputum culture can be order to dt infection.

Risk factors for mortality due to asthma?

AA race Previous hx of intubation >2 ED visits in a year Use of 2 SABA canisters in a month Use of systemic steroids Inability to perceive SOB

Diagnostic studies and findings for CF? Which is most diagnostic?

ABG - hypoexmia, compensated resp. acidosis PFTs - mixed obstructive and restrictive CXR - hyperinflation, peribronchial cuffing, mucous plugs Thin section CT - bronchiectasis Sweat Chloride test <---- more than 60 on two different days is diagnostic DNA testing <----definitive

Diagnostic tests for OHS?

ABGs - high bicarb, hypercapnia, hypoexmia PFT Sleep study CXR EKG Echo

Systolic Dysfunction Tx

ACE inhibitors or angiotensin receptor blockers (ARBs): ACEi cause cough, then switch to ARBs Beta blockers: Antiischemic effect, ↓ HR lead to ↓O2 consumption. ↓ mortality in pt. Spironolactone: Ø effect of aldosterone; SE: gynecomastia, switch to Eplerenone Diuretics: 1st therapy of CHF w/ ↓ ejection fracture. Use in combo w/ ACEi & ARBS Digoxin: Control symptoms of Dyspnea. Do not ↓ mortality.

What two medications are always prescribed for heart failure? What med is added at NYHA Class III?

ACE-I and B blocker Spironolactone

This BP med causes vascular dilation, thereby reducing preload and afterload

ACEi

Initial tx for CHF:

ACEi (captopril) and diuretic (Lasix)

What does decrease mortality in CHF?

ACEi/ARBs, BB, spironolactone

What are the "serious 6" differential diagnoses of chest pain?

ACS PE Dissection Cardiac Tamponade Pneumothorax Esophageal rupture

Altered Mental Status

AEIOU: Alcohol, Endocrine/Electrolytes, Infection, Oxygen/Opiates, Uremia TIPS: Trauma/Temperature/Toxins, Insulin, Psychiatric/Porphyria, Stroke/Schock/SAH

Oral candidiasis caused in what pts?

AIDS Diabetes Following corticosteroid/abx therapy (makes sense --> just like a yeast infection)

What is chronic bronchitis?

AKA "blue bloater". Chronic cough that is productive on most days for 3 months of the year for 2 or more consecutive years without and otherwise defined cause

What is emphysema?

AKA "pink puffer". When the air spaces are enlarged as a consequence of destruction of alveolar septae.

What is a Carcinoid tumor?

AKA carcinoid adenoma or bronchial gland tumors. These are well differentiated neuroendocrine tumors. M=F. Pts are <60 yo. They are low grade malignant neoplasms that grow slowly and rarely metastasize. More commonly found in the GI tract.

What is a Solitary pulmonary nodule?

AKA coin lesions. It's a round or oval, sharply circumscribed pulmonary lesion up to 3cm in diameter surrounded by normal lung tissue. Most are asymptomatic and are found incidentally.

What is atypical PNA? What is the most common pathogen?

AKA walking pneumonia. Clinical presentation is different from CAP. Most common pathogen is Mycoplasma pneumoniae. Usually occurs in young, healthy adults and is self-limiting.

Therapy for Heart Failure by NYHA classifications?

ALL PATIENTS: ACE I B-blocker Stage 3/4: - Spironolactone/Eplerenone - Combined hydralazine/isosrobide dinitrate (African American patients) - Digoxin (reduces hospitalizations, NOT mortality) - Diuretics (to maintain evolemia)

Cardioselective beta blockers?

AMEBA Atenolol Metoprolol** Esmolol Bisoprolol** Acebutolol **are usually used in patients with CHF with co-morbid COPD, asthma

Chest pain DDx

AMI, Angina, Aortic Dissection, PE, Community Acquired Pneumonia, Pneumothorax,Pleurisy, Pericarditis, GERD, Esophageal Spasm, Malignancy, Musculoskeletal, EDA, Shingles

What scoring system do you use to assess someone once they've been admitted to the ICU?

APACHE It is a severity-of-disease classification system (Knaus et al., 1985),[1] one of several ICU scoring systems. It is applied within 24 hours of admission of a patient to an intensive care unit (ICU): an integer score from 0 to 71 is computed based on several measurements; higher scores correspond to more severe disease and a higher risk of death.

What do you use for anticoagulation/antiplatelet therapy in stroke?

ASA (not in hemorrhagic), heparin for thrombosis

Medications for stable angina:

ASA (or clopidogrel if ASA is contraindicated), BB (propranolol), statin if LDL is high

____ is characterized by refractory conduction of impulses from the atria to the ventricles through the AV node and/or bundles of HIS; may produce weakness, fatigue, light-headedness, syncope

AV block

wenchkebach/mobitz I heart block w/n ____

AV node

What is an example of a SVT treatable with adenosine?

AVNRT

Valvular Disease: Aortic Aneurysm Dx

Abdominal ultrasound. Risk Factor: HTN, Smoking, FH, LDL

1st line treatment for atrial flutter?

Ablation is superior to medical management

Cholelithiasis

Abnormal condition of gallstones., Gallstone formation in the gallbladder

What is Bronchiectasis?

Abnormal, permanent dilation of the bronchi and destruction of bronchial walls. Due to bronchial injury from recurrent or severe infections/inflammation

What is a pleural effusion?

Accumulation of significant volumes of pleural fluid.

What enzyme does metronidazole inhibit to cause the "disulfiram like reaction?

Acetaldehyde dehydrogenase (converts acetaldehyde to acetate) --> buildup of acetaldehyde causes reaction

The TST does not distinguish between....

Active and latent infection

Chest pain relieved by sitting up, worsened by laying down and breathing. EKG shows diffuse PR depression and diffuse ST elevation

Acute pericarditis; Tx: ASA+NSAIDS (indocin)

Tx for Asthma

Acute: O2, bronchodilating agents (short-acting inhaled β2-agonists are first-line therapy: *Albuterol*), ipratropium (never use alone for asthma), systemic corticosteroids, magnesium (for severe exacerbations). Maintain a low threshold for intubation in severe cases or acutely in patients with PCO2 > 50 mmHg or PO2 < 50 mmHg. ■ Chronic: Measure lung function (FEV1, peak fl ow, and sometimes ABGs) to guide management. Administer long-acting inhaled bronchodilators and/ or inhaled corticosteroids, systemic corticosteroids, cromolyn, or, rarely

Purpose of LABAs?

Add on therapy for management of nocturnal sx and exercise induced asthma. Never used as monotherapy!

Most common type of lung cancer?

Adenocarcinoma

Most common type of esophageal cancer

Adenocarcinoma (70%) then Squamous cell carcinoma. Adenocarcinoma is seen more in distal esophagus and GEJ.

Torsed Gonad Female

Adnexal tenderness to palpation and possibly mass Must exclude ectopic pregnancy Pain control Prompt OB/GYN consult Ultrasound Laparotomy

This refers to the resistance to left ventricular ejection and outflow, or the stress on the ventricular wall at the end of systole:

Afterload

RANSONs criteria

Age in years > 55 years White blood cell count > 16000 cells/mm3 Blood glucose > 10 mmol/L (> 200 mg/dL) Serum AST > 250 IU/L Serum LDH > 350 IU/L In 48 hours: Serum calcium < 2.0 mmol/L (< 8.0 mg/dL) Hematocrit fall > 10% Oxygen (hypoxemia PaO2 < 60 mmHg) BUN increased by 1.8 or more mmol/L (5 or more mg/dL) after IV fluid hydration Base deficit (negative base excess) > 4 mEq/L Sequestration of fluids > 6 L

What pathogen is most likely to cause PNA in a patient with alcohol abuse? COPD? Cystic fibrosis? Young adults/college settings? Air conditioning? Post-splenectomy? Leukemia/lymphoma? Children<1 year? Children >1, <2?

Alcohol abuse - Klebsiella COPD - Haemophilus Cystic fibrosis - Pseudomonas Young adults/college settings - Mycoplasma or Chlamydia Air conditioning - Legionella Post-splenectomy - Strep pneumo, Haemophilus Leukemia/lymphoma - Fungus Children<1 year - RSV Children >1, <2 - Parainfluenza

2 most common causes of cirrhosis

Alcoholic liver disease, Hepatitis C

Treatment for uric acid and cysteine stones?

Alkalinization of urine (citrate)

Cardiomyopathy can be dilated, hypertrophic, or restrictive

All present SOB, particularly worsened by exertion. Rales, edema, JVD are found in all types of cardiomyopathy. Diagnostic Test for all is *ECHO* *Tx all with diuretics*

What are common triggers for asthma attack?

Allergens Exercise URIs GERD Drugs (BB, ACEI, ASA, NSAIDs) Stress Cold air

What deficiency leads to COPD?

Alpha 1 antitrypsin

MI is usually d/t atherosclerotic dz

Also: in cocaine abuse, HF, hypotension, or excessive metabolic demands • Atrial fibrillation - you are firing off all these random pulses, and sometimes a clot gets thrown - leading to an MI

What are PE findings with CAP?

Altered breath sounds (crackles) Dullness to percussion Bronchial breath sounds over an area of consolidation

What is the pathophysiology of resistance of MRSA?

Altered penicillin binding proteins

Medication that causes ototoxicity:

Aminoglycoside ABX

Macrolides vs. Aminoglycosides?

Aminoglycosides - Streptomycin - Gentamycin - Tobramycin Macrolides - Azithromycin - Clarithromycin - Erythromycin

Treatment for complex regional pain syndrome

Amitriptyline, nortriptyline, gabapentin, pregabalin, lamotrigine; NSAIDs; Calcitonin to reduce pain as adjunctive therapy; Bisphosphonates, IVIG, regional nerve blocks, dorsal column stimulation

For a patient with heart failure and hypertension already receiving optimal antihypertensive therapy what drug should be added to further control hypertension?

Amlodipine

Augmentin Zosyn Unasyn

Amoxicillin/clavulanic acid Piperacillin/tazobactam Ampicillin/Sulbactam

Treatment of cryptococcus neoformans?

Amphotericin B + flucytosine followed by Fluconazole

Cholangitis

An acute bacterial infection of the biliary tree that commonly occurs 2° to obstruction, usually from gallstones (choledocholithiasis) or primary sclerosing cholangitis (progressive infl ammation of the biliary tree associated with ulcerative colitis). Other etiologies include bile duct stricture and malignancy (biliary or pancreatic). Gram- enterics (e.g., E. coli, Enterobacter, Pseudomonas) are commonly identifi ed pathogens.

Atopy define

An allergy to something that is inhaled such as pollen or house dust. Also called 'inhalant allergy.'

A conduction disorder due to interruption between the SA node and the AV node results in:

An elongation of the PR interval

Paroxysmal Atrial fibrillation definition?

An episode of AF that terminates spontaneously or with intervention in less than 7 days.

Irritable bowel syndrome

An idiopathic functional disorder that is characterized by changes in bowel habits that ↑ with stress as well as by abdominal pain that is relieved by bowel movements. It is most common in the second and third decades, but since the syndrome is chronic, patients may present at any age. Half of all IBS patients who seek medical care have comorbid psychiatric disorders (e.g., depression, anxiety, fibromyalgia).

Diverticular Disease: CBC may show leukocytosis

An increase in the number of leukocytes (white blood cells); usually the result of a microbiological attack on the body.

Common Cardiovascular causes of chest pain:

Angina (Stable, Prinzmetal's), Unstable angina, MI, Myocarditis, Pericarditis, Dissecting aortic aneurysm, MVP

Life expectancy in aortic stenosis w/ different symptoms Angina Syncope CHF A fib

Angina - 5 years Syncope - 3 years CHF - 2 years A fib - 6 months

Digoxin Toxicity includes

Anorexia, N/V, change in visual perception.

ECG leads corresponding to areas of MI?

Anterior wall - LAD artery - V1-V4 Lateral wall - circumflex artery - Lead I, aVL Inferior wall - RCA - Lead II, III, aVF

diagnosis of celiac's disease? Skin disease association?

Anti-tissue transglutaminase IgA endomysial antibody Dermatitis Herpetiformis

When are antibiotics indicated for acute bronchitis?

Antibiotics are indicated for elderly patients, patients with cough >7-10 days + comorbidities, or immunocompromised patients.

Treatment of Dementia

Antipsychotics to manage psychosis

Chest Pain (Substernal Tightness) Differentials: *Psychiatric*

Anxiety

Cause of Torsade de Pointes

Anything that prolongs the QT interval; MC cause: hypomagnesemia

Crescendo-decrescendo murmur in systole with a paradoxical S2 split

Aortic Stenosis

Aortic Aneurysm vs Aortic dissection

Aortic aneurysm is most often associated with atherosclerosis, while aortic dissection is commonly linked to hypertension.

Valvular Disease: Aortic Aneurysm

Aortic aneurysms are most commonly associated with atherosclerosis. Most are abdominal, and > 90% originate below the renal arteries.

Caused by a tear in the intimal layer of the aorta

Aortic dissection (#1 risk factor: HTN)

Diastolic murmur w expanded pulse pressure, Quincke's pulse, Austin Flint murmur

Aortic regurgitation

Quincke's pulse, Watson's waterhammer pulse (bounding), de Musset's sign, Duroziez's sign; Austin Flint murmur are all associated w:

Aortic regurgitation

Systolic murmur w distinct hx/sx of angina, syncope, pulsus tardus et parvus, carotid thrill, paradoxical S2 split

Aortic stenosis

Why do people die of rhabdomyolysis What are the CMP changes in rhabdomyolysis

Arrhythmias (Hyperkalemia) Hyperkalemia Hyperphosphatemia Hyperuricemia Hypocalcemia (phosphate binds calcium and excretes it)

What are four significant types of pneumoconioses?

Asbestosis Coal worker's pneumoconiosis Silicosis Berylliosis

Lung infection in a PRE EXISTING LUNG CAVITY?

Aspergillomas form in pre-existing lung cavities (ie sarcoidosis causes granulomatous inflammation)

How long to continue Dual Antiplatelet therapy (DAPT) after Coronary Artery stenting to prevent restenosis?

Aspirin and Plavix for 12 months, then continue with Aspirin alone. 3 months is the new guideline...1 month is the MOST CRUCIAL

What medications are given for ACS?

Aspirin, Clopidogrel, Metoprolol, Heparin, Nitrates, Morphine

Anticoagulaiton after CABG?

Aspirin/Plavix Atorvostatin

Treatment for acute pericarditis:

Aspirin; can add corticosteroids [dx'd on EKG and will show diffuse PR depression and/or ST elevation]

Non-Convulsive (absence) seizures

Associated wtih only minor motor activity such as blinking or facial twitching. Begin in childhood typically subsiding before adulthood.

Signs of hepatic encephalopathy?

Asterixis, Palmar erythema

Clinical features of effusions

Asymptomatic if small Large or bilateral effusions - dyspnea, dullness to percussion, reduced or absent breath sounds Mediastinum is shifted away from the side of a large effusion

Clinical features of carcinoid tumor?

Asymptomatic usually Hemoptysis Cough Focal wheezing Recurrent PNA Carcinoid syndrome (flushing, diarrhea, wheezing, hypotension) <----occurs in 10% of patients

What is the strongest predisposing factor to asthma?

Atopic triad: Wheeze, eczema, seasonal rhinitis

Uncoordinated electrical activity of the atria, ultimately resulting in irregular activation of the AV node and irregular pulse

Atrial Fibrillation

Arrhythmia Types

Atrial Fibrillation Atrial Flutter PSVT Heart Block PVC, PAC, MAT Ventricular Fibrillation Ventricular Tachycardia WPW

Mainstay of therapy for bradyarrhythmias:

Atropine

For ANY bradycardia developing in an MI/post MI pt, the initial tx of choice is:

Atropine; if atropine is insufficient to raise BP - temporary pacing

SVT

Attempt Vagal Maneuvers Adenosine (6, 12, 12) Cardioversion

low-pitched diastolic murmur at apex

Austin-Flint murmur

mid-diastolic murmur in severe aortic regurgitation

Austin-Flint murmur

Anti smooth muscle Ab

Autoimmune hepatitis Treat with steroids (don't treat viral hepatitis with steroids)

What is Cystic Fibrosis (CF)?

Autosomal recessive disorder that results in the abnormal production of mucus by almost all exocrine glands, causing obstruction of those glands and ducts. Patients are at increased risk for malignancies of the GI tract, osteopenia, and arthropathies. Median age of survival is 31.

Pneumoconiosis Tx

Avoid triggers; supportive therapy and supplemental O2.

What class of medicine should you avoid with cor pulmonale?

BB

Tx for MVP

BB for pain

hypertrophic cardiomyopathy Tx

BB, CCB disopyramide good for its negative inotropic effects non/surgical ablation of hypertrophic septum dual-chamber pacing implantable defibrillators mitral valve replacement

Outpatient management of CHF:

BB, diuretic, ACEi, if systolic failure (esp with A-Fib) - digoxin; low sodium diet with fluid restriction (<2L/d), management of comorbid conditions

Tx for atrial fibrillation w/o accessory pathway or CHF

BB/CCB

Mnemonic used for Chronic kidney disease management?

BEANS BP control (less than 140/90) Erythropoietin (for pts. with Hgb <10) Access for long term dialysis Nutrition Specialist referral

Right sided HF hampers venous return from the systemic circulation leading to systemic venous engorgement.

Back pressure on systemic veins leads to conditions such as peripheral edema, hepatic congestion, and gastrointestinal complaints.

Bacterial and Viral meningitis glucose in CSF from lumbar puncture

Bacterial: <40 Viral: >40

Bacterial and Viral meningitis WBC in CSF from lumbar puncture

Bacterial: >1000 Viral: <1000

Bacterial and Viral meningitis protein in CSF from lumbar puncture

Bacterial: >200 Viral: <200

Tx of PCP pneumonia?

Bactrim is DOC

Causative organism for bacillary angiomatosis? Treatment for bacillary angiomatosis?

Bartonella Heneslae Erythromycin or Doxycycline

Paget's disease Dx

Based on clinical history, characteristic radiographic changes (see Figure 2.3- 3), and lab findings.

How is asthma classified?

Based on frequency of symptoms and PFTs: Intermittent Mild Persistent Moderate persistent Severe persistent

What are the two major categories of lung cancer?

Based on staging and treatment options: Small cell lung cancer (SCLC) Non-small cell lung cancer (NSCLC)

What is sinus arrhythmia?

Basically everything is within normal limits on EKG except the R-R interval is variable

what trio of PE findings might be present in pt w/ pericardial tamponade

Beck's Triad 1. muffled heart sounds 2. JVD 3. narrowing pulse pressure/hypotension

Cardiac Tamponade signs/symptoms?

Beck's triad: Hypotension JVD Muffled heart sounds Pulsus paradoxus (decrease in systolic BP > 10 mmHg during inspiration) ECG shows low voltage QRS and electrical alterans due to "swinging" movement of heart in large effusion

What are characteristics of a benign solitary pulmonary nodule? Malignant?

Benign - has not enlarged in >2 yrs, asymptomatic Malignant - symptomatic, occur in patients >45, >2cm in diameter, indistinct margins, rapid progression in size, rarely calcified

Pterygium?

Benign growth of the conjuctiva, from the nasal side of the sclera

Cause of propylene glycol toxicity?

Benzodiazapenes

1st line treatment for active seizures in ED

Benzodiazepines (midazolam, lorazepam, diazepam), Short acting barbiturates (phenobarbital, thiopental)

Diagnostic Test for asthma

Best initial test : ABG Others: CBC (↑ eosinophil count) , CXR (hyperinflation), ↑ IgE (suggest an allergic etiology)

Best initial test for pt whom you suspect valvular heart disease? What about most accurate?

Best initial: echocardiogram Most accurate: cardiac catheterization

Purpose of Beta blocker and ACE I after MI?

Beta Blocker reduces infarct size ACE I reduces ventricular remodeling

Diastolic Dysfunction

Beta Blockers have clear benefits and Diuretics.

Treatment of aortic dissections distal to the subclavian artery?

Beta blockers --> control blood pressure

Management of solitary pulmonary nodule that has an intermediate probability of malignancy?

Biopsy - transthoracic needle biopsy or bronchoscopy if lesion is peripheral High resolution PET scan High resolution CT - to determine where the mass is and detect adenopathy or presence of multiple nodules

Hypertension

Blood Pressure >140/90 Common in the ED, 33% of HTN in ED is situational, Treatment in asymptomatic patients is controversial

what is the organism that causes lyme disease

Borellia burgdoferi, a spirochete

Subdural Hematoma

Bridging Veins *Cresent* Venous bleed

the ___ test is used to differentiate saphenofemoral valve incompetence from perforator vein incompetence

Brodie-Trendelenburg

What are characteristics of SCC?

Bronchial in origin and a centrally located mass. More likely to present with hemoptysis (more likely to be diagnosed via sputum cytology)

Diagnostic studies and findings for carcinoid tumors?

Bronchoscopy - pink or purple central lesion that is well vascularized. Can be pedunculated or sessile. CT and octreotide scintigraphy - localizes disease

Protocol for Stress Test:

Bruce Protocol

What are two buzz words that go with Mycoplasma pneumoniae?

Bullous myringitis and cold agglutinins

Diagnostic studies and findings for sarcoidosis?

CBC - leukopenia, eosinophilia Elevated ESR BMP - hypercalcemia, hypercalciuria ACE levels - elevated 40-80% CXR - symmetrical bilateral hilar and right paratracheal adenopathy and bilateral diffuse reticular infiltrates Transbronchial biopsy of lung or FNA biopsy confirms diagnosis

HIV prophylaxis meds? HIV patient with Diarrhea? HIV patient with neurological signs? HIV meningitis causative organism?

CD4 count < 200 --> TMP SMX (Pneumocystis Jiroveci) CD4 count < 50 --> Azithromycin (Mycobacterium avium) CD4 count < 100 --> TMP SMX (Toxoplasmosis) CMV - gangcicylovir (neutropenia) or foscarnet (renal tox) MAC - Azithromycin Cryptosporidium - watery diarrhea, oocysts are acid fast Multiple ring enhancing lesions - Toxoplasmosis Single ring enhancing lesion - CNS lymphoma Cryptococcus (+ India ink) Treate with amphotericin B

long term anticoagulation for Afib is based on ___ score

CHADSVAS

CHADS2 for stroke

CHF, HTN, Age>75, DM, Stroke Hx

Cardiac enzyme (CK-MB):

CK-MB (will be abnormal first 4-12h) and is also most specific. Compared to troponin, it will also correct first after MI

which disease do HIV pts get at cd4 counts less than 50 and can cause necrotizing adrenalitis, which causes clinical adrenal insufficiencY?

CMV

4 Causes of low voltage EKG?

COPD Tamponade Obesity Hypothyroidism

Diagnostic studies for solitary pulmonary nodule?

CT Biopsy

Diagnosis of stroke (gold standard)

CT angiography

Management of solitary pulmonary nodule that has a low probability of malignancy?

CT every 3 months for a year. If stable, CT every 6 months for the next 2 years.

Diagnosis of acute pancreatitis

CT scan, Elevated Lipase, Ranson's Criteria

Imaging modality used to diagnose Subarachnoid Hemorrhage?

CT without contrast. If CT negative but still suspect SAH, do LP to look for RBC or xanthochromia (will not develop until 12hrs after onset)

What is the best initial test for diagnosing COPD?

CXR

aortic dissection Dx

CXR -*widened mediastinum -deviation of trachea, mainstem bronchi, or esophagus -apical capping -pleural effusion -"calcium sign" EKG -ischemia TEE-test of choice spiral CT-most sensitive MRI/MRA

hypertrophic cardiomyopathy Dx

CXR -not remarkable EKG -nonspecific ST and T wave changes -exaggerated septal Q waves -LVH **ECHO -LVH -asymmetric septal hypertrophy -small left ventricle -diastolic dysfunction myocardial perfusion studies cardiac MRI cardiac cath

heart failure Dx

CXR -pulmonary edema -pleural effusions -cardiomegaly (left-sided failure shows enlarged cardiac silhouette) -cephalization = localization of important organs near the head -kerley b-lines BNP -levels increase w/ CHF (secreted from ventricles in response to elevated ventricular filling pressures) -levels increase w/ age -levels decrease w/ obesity -levels decrease w/ renal function -levels decreased w/ AF evidence of decreased organ perfusion -increased Cr -decreased Na -abnl LFTs echo -decreased ejection fraction -increased chamber size PA catheterization -increased PCWP -decreased cardiac output -increased SVR (in low-output failure) EKG coronary angio

What are diagnostic studies for lung cancer?

CXR CT Cytologic exam of sputum Bronchoscopy, exam of pleural fluid, biopsy PET scan

Diagnostic studies for pulmonary fibrosis?

CXR CT - diffuse, patchy fibrosis with pleural-based honeycombing PFTs- will show restrictive pattern (decreased lung volume with normal to increased FEV1/FVC ratio Bronchoalveolar lavage Transbronchial biopsy Surgical lung biopsy

Diagnostic studies for TB

CXR Tuberculin skin test (TST) Cultures/DNA or RNA amplification Biopsy

Diagnostic studies for COPD?

CXR PFTs CBC - may show polycythemia due to compensation

Diagnostic studies and findings for pleural effusion? What is the gold standard?

CXR - blunting of the costophrenic angle, loss of sharp demarcation of the diaphragm and heart, mediastinal shift to uninvolved side Lateral decubitus radiography - shows small effusions; differentiates free-flowing vs loculated CT - separates parenchymal and pleural densities Thoracentesis <----GOLD STANDARD!

Diagnostic studies and findings of PCP pneumonia?

CXR - diffuse or perihilar infiltrates in a butterfly or "ground glass" pattern. No infiltrates. CBC - low WBC CD4 count - <200 Sputum staining or bronchoalveolar lavage - will establish diagnosis in >90% of patients

Diagnostic studies for pulmonary HTN?

CXR - enlarged pulmonary arteries EKG - RVH, atrial hypertrophy, RV strain Echo - estimates pulmonary arterial pressure Cath - most precise way to measure pressure

First best step when suspecting CHF:

CXR; next: Echo to confirm dx and to determine LVEF, Electrocardiogram in systolic HF,

What are Ghon complexes that represent healed infection?

Calcified primary focus in the lungs

First line antihypertensive medication in Aftican Americans?

Calcium channel blockers

3 main causes of HIV+ esophagitis?

Candida (plaques) HSV-1 (punched out ulcers) CMV (linear ulcers)

Treatment of generalized convulsive, simple partial and complex partial seizures is what?

Carbamazepine, Phenytoin, Valproic Acid

Beware of this in penetrative or blunt force trauma to the chest (can also be anything that causes pericarditis)

Cardiac Tamponade

PSVT w/risk factors for MI; first labs:

Cardiac enzymes

Condition to r/o with electrical alternans

Cardiac tamponade; may also be caused by any pericardial effusion

What is the historical landmark of TB?

Caseating granuloma that is AKA necrotizing granuloma

What is a histologic hallmark of TB?

Caseating granulomas (necrotizing granulomas) on biopsy

Empiric treatment for bacterial meningitis

Ceftriaxone 2g IV and Vancomycin =/- Acyclovifr, Dexamethasone 10 mg IV; spupportive care

Initial therapy for Community acquired pneumonia?

Ceftriaxone/Tetracycline

CVA

Cerebral ischemia/infaction Occlusive vs. Hemorrhagic HTN, atherosclerosis, DM, lipids, smoking, family hx, estrogen, trauma, Arteriovenous Malformations, aneurysms, tumors TIA(symptoms resolve in 24 hours)- 33% have CVA within 5 years

What changes in blood pressure suggest subclavian steal syndrome?

Change in BP>20 mm Hg between upper and lower extremities

Chronic obstructive pulmonary disease (COPD)

Characterized by ↓ lung function with airflow obstruction. There are two classic types of COPD: chronic bronchitis and emphysema.

Classic finding with a central retinal artery occlusion?

Cherry red spot

Pulmonary hypertension Diagnostic Test

Chest X-Ray and CT (both are best initial test) ■ ECG demonstrates RVH. ■ Echocardiogram and *right heart catheterization* may show signs of right ventricular overload and may aid in the diagnosis of the underlying cause.

Best 1st test for pneumonia? Most common overall causative organism Most common atypical pneumonia (walking pneumonia) Most cmmon in alcoholics

Chest X-ray Strep. pneumonia Mycoplasma (associated with cold agglutinins) Klebsiella (aspiration pneumonia)

MI Hx/ Clin Sx:

Chest pain/pressure, like "an elephant sitting on their chest", radiating to jaw, shoulder, etc; SOB; nausea/vomiting; feeling of impending doom; syncope

How do you remove the fluid or air from Pneumothorax?

Chest tube

Most common cause of female infertility?

Chlamydia infection (PID, salpingitis, Tuboovarian abscess, ectopic pregnancy, Fitz hugh curtis syndrome)

What is the main presentation of an aspirated foreign body?

Choking, coughing or unexplained wheezing or hemoptysis

Cholelithiasis Tx

Cholecystectomy is curative and can be performed electively for symptomatic gallstones. It is generally performed laparoscopically. Asymptomatic gallstones do not require any intervention. ■ Patients may require preoperative endoscopic retrograde cholangiopancreatography (ERCP) for common bile duct stones. ■ Treat nonsurgical candidates with dietary modifi cation (avoid triggers such as fatty foods).

Pain is RUQ, "the 4 F's": fat, female, fertile, and forty; positive Murphy's sign

Cholecystitis

"rice water" stools are characteristic of what bacterial gastroenteritis?

Cholera (vibrio); stools becomes colorless and mucus flecked

What is Pneumoconiosis?

Chronic fibrotic lung diseases caused by inhalation of coal dust or various inert, inorganic, or silicate dusts

Ulcerative colitis

Chronic inflammatory disease that causes ulceration of the colonic mucosa

Management of primary pulmonary HTN?

Chronic oral coagulants CCBs to lower systemic arterial pressure Prostacyclin - a potent pulmonary vasodilator Heart-lung transplant is usually needed

What are clinical features of bronchiectasis?

Chronic purulent sputum (foul smelling) production Hemoptysis Chronic cough Recurrent PNA

Treatment for UTIs?

Ciprofloxacin Ceftriaxone Bactrim

What are classic clinical features of asthma?

Classic: wheezing, cough, dyspnea, sputum production Pts are asymptomatic between episodes

When to consider spironolactone in a heart failure pt?

Classically in a NYHA stage III/IV heart failure EF<35% New guidelines allow for consideration of spironolactone in NYHA class II heart faiulre

Celiac disease Presentation

Clinical presentation a. Diarrhea, steatorrhea, flatulence, weight loss, weakness, and abdominal distension are common. b. Infants and children may present with failure to thrive. c. Older patients may present with iron deficiency, coagulopathy, and hypocalcemia.

Valvular Disease: Deep Venous Thrombosis (DVT)

Clot formation in the large veins of the extremities or pelvis. The classic Virchow's triad of risk factors includes venous stasis (e.g., from plane flights, bed rest, or incompetent venous valves in the lower extremities), endothelial trauma (injury to the lower extremities), and hypercoagulable states (e.g., malignancy, pregnancy, OCP use).

What causes pneumoconoises?

Coal dust, silicate or other inert dusts

Cholelithiasis Info

Colic results from transient cystic duct blockage from impacted stones. Although risk factors include the 4 F's—Female, Fat, Fertile, and Forty—the disorder is common and can occur in any patient. Flatulence can be thought of as a "5th F." Other risk factors include OCP use, rapid weight loss, a family history, chronic hemolysis (pigment stones in sickle cell disease), small bowel resection, and TPN.

Hiatal hernia Dx

Commonly an incidental finding on CXR; also frequently diagnosed by barium swallow or EGD.

What is the name for the disease that beings following an operation, a fracture, or vascular event such as stroke or heart attack?

Complex regional pain syndrome (CRPS)

Focal (partial) seizures are Complex and Simple. Whats the difference between the two?

Complex: characterize by aura, followed by impaired consciousness lasting seconds to minutes. N/V, focal impairments may occur. Simple: consciousness maintained. May be tonic-clonic activity.

Management of CF?

Comprehensive multidisciplinary therapy is key Focus on clearance of airway secretions, reversal of constriction, tx of infections, replacement of pancreatic enzymes, nutritional and psychosocial support

Head Injuries

Concussion: Repetitive questioning, brief LOC Skull Fracture: Battle sign (bruising of mastoid process), Raccoon's eyes, Cerebrospinal fluid leak, blood in EAC EDH, SDH, ICH: Seizure, Altered Mental Status, Pain, Focal Deficits In the elderly, clinical acumen cannot reliably exclude significant injury

1° (Essential) Hypertension Dx

Conduct cardiovascular, neurologic, ophthalmologic, and abdominal exams. ■ Obtain a UA, BUN/creatinine, CBC, and electrolytes to assess the extent of end-organ damage.

What are causes of bronchiectasis?

Congenital - CF (50% of cases) Acquired from infections - TB, fungal infx, lung abscess Obstruction - tumor

The inability of the heart to pump blood in proportion to the metabolic needs of the body

Congestive Heart Failure

Dark colored urine caused by what?

Conjugated hyperbilirubinemia (hepatitis)

Head injuries Treatments

Consider transfer to traumacenter Neurosurgical consult Hyperventilation controversial, Airway protection for GCS <8 or rapid decline Mannitol/Hypertonic saline for ICP Burr hole Correct platelets, coagulopathy Avoid hypotension/hypoxia

What is the pneumovax?

Contains 23 strains. For patients >65, patients with chronic illnesses (sickle cell disease, tobacco abuse, splenectomy, liver disease), immunocompromised.

Management of pulmonary fibrosis

Controversial because no treatment has been able to improve the outcome

rapid, quick arterial pulse

Corrigan's pulse

Chest pain worse w/ palpitation

Costochondriasis

Pt has chest pain that is reproducible to palpation

Costochondritis

What is the most common symptom of TB? What are the classic symptoms?

Cough (dry then productive, with or without hemoptysis) for three weeks or longer - MC symptom Fever, night sweats, anorexia, weight loss, posttussive rales <---classic Patient will appear ill and malnourished

What are clinical features of acute bronchitis?

Cough (wet or dry) - note sputum is not predictive of bacterial involvement Dyspnea Fever Sore throat HA Myalgias Substernal discomfort Expiratory rhonchi or wheezes

What is the Clinical features of acute bronchitis?

Cough (with or without sputum) is the predominant symptom-it lasts 1 to 2 weeks. In a significant number of patients, the cough may last for 1 month or longer.

Management of bronchiectasis?

Cough should be treated with antibiotics (PCNs, bactrim, Cipro), bronchodilators, chest physiotherapy Lung transplant is definitive tx

What are the main symptoms of cystic fibrosis?

Cough, excessive sputum, sinusitis, steatorrhea and ABD pain

Tx for HAP?

Cover for Pseudomonas (ceftriaxone, respiratory fluoroquinolone, imipenem, cefepime) Aggressive supportive measures

Which one shows "fat wrapping" of intestines, Ulcerative Colitis or Crohn's?

Crohn's

How do you treat GI bleed?

Crystalloid fluid of choice, PPI or histamine blockers, vasopressin and octreotide to constrict dilated esophageal vessels

Is surgery curative for Crohn's and Ulcerative Colitis?

Curative for Ulcerative Colitis. Only used in Crohn's if patient has complications.

What is the most common cause of bronchiectasis?

Cystic fibrosis

Appendicitis Work-up and Treatment

DDx: PID, gonadal torsion, acute gastroenteritis, diverticulitis, UTI, kidney stone, ectopic pregnancy IV, pain control, NPO, fluids CBC, UA, BMPm GC/Chlamydia, hCG CT or ultrasound Surgical Consult +/- Antibiotics

Most common causes of CKD

DM, HTN, glomerulonephritis, polycystic kidney dz

Coronary vascular disease Risk factors

DM, a family history of premature CAD, smoking, dyslipidemia, abdominal obesity, hypertension, male gender, symptomatic carotid artery disease, peripheral arterial disease, and AAA.

endocarditis Dx ____ criteria

DUKE criteria note: 2 major 1 major + 3 minor 5 minor

dilated cardiomyopathy Sx

DYSPNEA S3 gallop pulmonary crackles (rales) increased jugular venous pressure

Peptic Ulcer Disease (PUD)

Damage to the gastric or duodenal mucosa caused by impaired mucosal defense and/or ↑ acidic gastric contents. *H. pylori* plays a causative role in > 90% of duodenal ulcers and 70% of gastric ulcers. Other risk factors include *corticosteroid, NSAID, alcohol, and tobacco* use. Males are affected more often than females.

What serum lab markers can be clinically useful for diagnosing SIADH in a patient with hyponatremia?

Decreased BUN < 4.0 mmol/L Serum uric acid < 240 uml/L

CXR findings with emphysema? What is the pathognomonic finding?

Decreased lung markings at apices Flattened diaphragms Hyperinflation Small-thin appearing heart Parenchymal bullae and subpleural blebs <--- pathognomonic

Vfib

Defibrillation is most important ACLS pulseless and unconscious

Coronary vascular disease

Defined as a clinical syndrome caused by the inability of the heart to pump enough blood to maintain fluid and metabolic homeostasis.

Congestive heart failure

Defined as a clinical syndrome caused by the inability of the heart to pump enough blood to maintain fluid and metabolic homeostasis. Risk factors include coronary artery disease (CAD), hypertension, cardiomyopathy, valvular heart disease, and diabetes

What is Hypertension?

Defined as a systolic BP > 140 mmHg and/or a diastolic BP > 90 based on three measurements separated in time.

Cirrhosis define

Defined as fibrosis and nodular regeneration resulting from hepatocellular injury. Etiologies include causes of chronic hepatitis, biliary tract disease (e.g., primary biliary cirrhosis, primary sclerosing cholangitis), right-sided heart failure, constrictive pericarditis, and Budd-Chiari syndrome (hepatic vein thrombosis 2° to hypercoagulability).

Hypertrophic Cardiomyopathy

Defined as impaired left ventricular relaxation and filling (nonsystolic dysfunction) due to thickened ventricular walls. Is a rxn to stressors on a heart such as increased blood pressure.

Restrictive Cardiomyopathy

Defined as ↓ elasticity of myocardium leading to impaired diastolic filling without significant systolic dysfunction

Mallory-Weiss tear Dx

Diagnose with endoscopy.

Diverticular Disease Dx

Diagnosis is based on AXR (to rule out free air, ileus, or obstruction), colonoscopy, or barium enema. Sigmoidoscopy/colonoscopy must be avoided in those with early diverticulitis due to the risk of perforation.

Valvular Disease: Lymphedema Dx

Diagnosis is clinical. Rule out other causes of edema, such as cardiac and metabolic disorders.

Heart that can "contract, but can't relax"

Diastolic CHF

Hypertrophy is concentric (inward not outward) and there are not sx of LVH in this disease:

Diastolic CHF

Stereotypical pres cardiogenic pulm edema: Pt presents w severe dyspnea and SOB, has a longstanding hx of uncontrolled HTN

Diastolic CHF

What is Light's criteria?

Differentiates transudates vs exudates:

EKG of pericarditis?

Diffuse ST elevation + *PR elevation in lead aVR*

Pericarditis EKG

Diffuse ST segment elevation-upward curves (smile) p-r depression

Medication that is positive inotrope and antiarrhythmic that has a severe toxicity profile

Digoxin

Tx for atrial fibrillation w/o accessory pathway w CHF

Digoxin/amiodarone

Esophageal varices

Dilation of the veins of the esophagus, generally at the distal end. Underlying cause is portal HTN most commonly caused by cirrhosis either from alcohol abuse or from chronic viral hepatitis.

What is DOT?

Direct observation therapy for pts. taking TB meds

Valvular Disease: Lymphedema Tx

Directed at symptom management, as no curative treatment exists. Exercise, massage therapy, and pressure garments to mobilize and limit fluid accumulation may be of help.

What is asthma?

Disease of chronic inflammation leading to airway narrowing and increased mucus production. Triggered by allergens, irritants, cold air, exercise.

Valvular Disease: Lymphedema

Disruption of the lymphatic circulation that results in peripheral edema and chronic infection of the extremities. Often a complication of surgery involving lymph node dissection. In underdeveloped countries, parasitic infection can lead to lymphatic obstruction, resulting in edema. Congenital malformations of the lymphatic system, such as Milroy's disease, can present with lymphedema in childhood.

Clue features: substernal pain that radiates to the back, "tearing pain"

Dissecting aortic aneurysm

What is therapy for reducing RV pressure?

Diuretics

What does not decrease mortality in CHF?

Diuretics, positive inotropes, nitrates and vasodilators, supp care: 100% o2, morphine, etc.

If I have sudden painless rectal bleeding (maroon/dark red), what do I have?

Diverticular disease (diverticulosis)

If patient is on a PPI and you suspect H. Pylori, how to confirm?

Do serology (serum antibody test)

Dobutamine, Digoxin for CHF

Dobutamine - acute CHF Digoxin - pt sent home with Positive inotropes

What are the most common irritants used for pleurodesis?

Doxycycline and talc

How often to draw cardiac enzymes?

Draw cardiac enzymes once on admission and every 8 hours until 3 samples are obtained

Occurs weeks to months after MI; sx: low grade fever, pleuritic chest pain, pericardial friction rub; labs: elevated ESR

Dressler's syndrome; usually self-limited, tx: colchicine

DVT Diagnosis

Duplex US (if negative, follow up US in 3-5 days) Impedance Plethysmography Venography MRI DDimer Well's Criteria

"singsong" murmur over femoral artery

Duroziez murmur

Dx and Tx of Cardiac Tamponade:

Dx: Echocardiogram; and EKG Tx: Pericardiocentesis

PE Diagnosis and Treatment

Dx: Well's Criteria (risk factors), Ddimer, Doppler US, PERC Rule, CTA, VQ Scan Tx: Heparin (quick on/off) vs. LMWH (easier to give) Thrombolytics (Saddle PE, admit to ICU) Embolectomy

Clinical features of pulmonary HTN?

Dyspnea Angina-like retrosternal chest pain Weakness/Fatigue Edema Ascites Cyanosis Syncope

What are clinical features of cor pulmonale?

Dyspnea Fatigue/lethargy Tachycardia Cyanosis/Hypoxia Clubbing Edema Accentuated RV thrust along LSB Split S2 JVD with prominent a wave

Clinical features of pneumoconioses?

Dyspnea Inspiratory crackles Clubbing of fingers Cyanosis

The hallmarks of heart failure:

Dyspnea and fatigue

Valvular Disease: Aortic Dissection Dx

ECG, CXR (shows widening of the mediastinum, cardiomegaly, or new left pleural effusion). *CT angiography is the gold standard of imaging.*

Indications for ICD (implantable cardioverter device) and biventricular pacing?

EF <35% and optimum medicine failed Biventricular pacing is indicated if pt has EF <35%, medical management has been maximized, AND pt. has prolonged QRS >120 msec (Bundle branch block).

sinus sick syndrome Dx

EKG

sinus tachycardia Dx

EKG

takotsubo Dx

EKG -ST elevation is frequent -ST depression is unfrequent -prolonged QT -T wave inversion -abnormal Q waves troponins elevated BNP elevated cardiac cath (coronary angiography) -absence of obstructive coronary dz -no evidence of acute plaque rupture serial TTE -transient LV systolic dysfunction cardiovascular magnetic resonance (CMR) -aid DDx -may exclude myocarditis

STEMI Dx

EKG -ST segment elevation -early peaked T waves -Q waves Cardiac biomarkers -myoglobin: peaks 6-7 hrs, duration 24 hrs -CK: peaks 12 hrs, duration 36 hrs -CK-MB: peaks 24 hrs, duration 48-72 hrs -LDH: peaks ~ 24-48 hrs, duration is 10-14 days -*TnT: peaks ~ 12-48 hrs, duration 5-14 days

dilated cardiomyopathy Dx

EKG -nonspecific ST and T wave changes -conduction abnormalities -ventricular ectopy CXR -cardiomegaly -pulmonary congestion Echo -LV dilation and dysfunction -high diastolic pressures -low cardiac output nuclear studies cardiac cath

NSTEMI/unstable angina Dx

EKG acute ischemic changes -ST depression -T wave inversion cardiac enzymes positive in NSTEMI negative in unstable angina

prinzmetal/variant angina Dx

EKG associated w/ ST elevations (RCA most often involved)

angina pectoris Dx

EKG btw episodes... 1/3 have normal EKGs nonspecific ST-T changes evidence of prior MI (pathological Qs) conduction abnormalities (LBBB, RBBB, fascicular blocks) during angina... ST depression/possibly elevation T-wave inversion cardiac enzymes negative (CK, CKMB, troponin) elevated cholesterol elevated glucose CXR often normal evidence of CHF arterial calcifications diagnose via stress testing! > ST thus > chance of CAD systolic BP drop > 10 mm Hg is bad

heart murmurs Dx

EKG echo doppler ultrasonography MRI CXR radionuclide flow studies cardiac catherization angiography

What is a buzz word for klebisella pneumonia?

ETOH abuse and current jelly sputum

What are the metabolic changes in early and late pulmonary edema?

Early pulm edema: CMP shows respiratory alkalosis due to high RR and release of CO2 Late pulm edema: CMP will show a pt in respiratory acidosis due to respiratory fatigue

Best test of choice in the setting of muffled heart sounds in a stable patient:

Echocardiogram

Most accurate test to differentiate between systolic and diastolic HF.

Echocardiography

What diagnostic test is used to evaluate CHF?

Echocardiography. Ejection fraction can be evaluated with this.

What is the cornerstone of therapy for all asthma patients?

Education Peak flow monitoring daily! Environmental control Management of cormorbidities Consider immunotherapy

What are two buzz words with pneumocystis jiroveci?

Elevated LDH and hyper-hypoxia

What will Echo show with cor pulmonale?

Enlarged RV

What will CXR show with cor pulmonale?

Enlarged pulmonary artery Dilated RV

Tx for atypical PNA?

Erythromycin or doxy for mycoplasma or legionella Tetracycline for chlamydia Supportive care if viral

Clue features: Normal EKG, hx of dysphagia, regurgitation; pain gets worse with administration of nitrates

Esophageal Spasm

Esophageal cancer

Esophageal cancer is a fast-growing and metastasizing type of cancer. Cancers of the upper esophagus are typically squamous cell and cancers of the lower esophagus are typically adenomas. Early symptoms are often vague and the delay before medical attention is sought significantly affects the prognosis. Treatment often involves several weeks of chemotherapy and radiation therapy followed by surgery.

Sinus Bradycardia Tx:

Establish IV access, administer IV atropine until resolution. If refractory, initiate transcutaneous pacing

When is coronary angiography indicated in the setting of heart failure?

Evaluation of new onset heart failure in the setting of a disease like diabetes that can cause silent ischemia

What is the pathogenesis for chronic bronchitis?

Excess mucus production narrows the airways; patients often have a productive cough. • Inflammation and scarring in airways, enlargement in mucous glands, and smooth muscle hyperplasia lead to obstruction.

Workup for stable angina:

Exercise stress test, cholesterol panel, fasting glucose, A1C, CMP

What are clinical findings with emphysema?

Exertional dyspnea Cough is rare Quiet lungs No peripheral edema Thin; recent weight loss Barrel chest Hyperventilation Pursed-lip breathing

Pneumoconioses Presentation

Exertional dyspnea and a nonproductive cough. ■ Inspiratory crackles and/or clubbing on exam.

Pulmonary fibrosis Presentation

Exertional dyspnea and a nonproductive cough. ■ Inspiratory crackles and/or clubbing on exam.

What type of image reveals the presence of pneumothorax?

Expiratory CXR

Which type of hemorrhoid causes pain?

External

What are 4 types of effusions?

Exudates - assosciated with "leaky capillaries". Caused by infection, malignancy, trauma Transudates - "intact capillaries" - caused by increased hydrostatic/oncotic pressure (CHF, atelectasis, renal or liver disease) Empyema - infection in the pleural space Hemothorax - bleeding in to the pleural space because of trauma or malignancy

T/F? antibiotics prevent post-streptococcal glomerulonephritis?

F

T/F antibiotics before invasive procedures to prevent endocarditis in pts w/ hx of rheumatic fever

F AHA no longer recommends antibiotics before invasive procedures to prevent endocarditis in pts w/ hx of rheumatic fever

endocarditis Sx

FEVER (may be absent in elderly) non-specific complaints -HA -myalgias -cough -dyspnea -arthralgias -back or flank pain -GI complaints stable murmur (~90%) classic features (~25%) -palatal, conjunctival, or subungal petechiae -splinter hemorrhages -osler nodes (painful, violaceous, raised lesions of the fingers/toes) -janeway lesions (painless red lesions, palms/soles) -roth spots (exudative lesions in the retina) pallor splenomegaly strokes/emboli

Tx of CAP for adults >60?

FLUOROQUINOLONE (Levaquin) Newer macrolide + 2nd gen cephalosporin (i.e. Ceftriaxone)

Fibrosis

FORMATION OF EXCESS FIBROUS CONNECTIVE TISSUE IN REPAIR OR REACTION.

Loss of elastic recoil causes increases of....

FRC-Functional Residual capacity

Shock Definition and Types

Failure to adequately deliver blood, oxygen, or nutrients to tissue to meet metabolic demands 1. Hypovolemic (not enough blood) 2. Cardiogenic (heart is not working well enough) 3. Distributive (Sepsis, anaphylaxis, cord injury) 4. Obstructive (PE, Tamponade)

What are clinical features of PCP pneumonia?

Fever Tachypnea/Dyspnea Non-productive cough Profoundly hypoxemic

What are the classic symptoms of TB?

Fever, night sweats, weight loss

Primarily used to tx high triglycerides; increases lipoprotein lipase which breaks down TGs

Fibrates [do not combine w statins (rhabdomyolysis)]

What will CXR show with reactivation TB?

Fibrocavitary apical disease Nodules Infiltrates Posterior and apical segments of RUL Apical-posterior segments of LUL Superior segments of lower lobes

AV Nodal Block treatments:

First degree: none 2nd degree: IV atropine if sx, otherwise none 2nd degree type 2: internal pacing 3rd degree: internal pacing

Best therapy for pericardial effusion:

Fluid aspiration and management of underlying cause

SE of SABAs?

Flushing Tremors Tachycardia

When are anticholinergics used in asthma?

For severe exacerbations [Ipratropium (Atrovent)]

FEV₁:

Forced Expiratory Volume in 1 second. →maximal amount of air you can forcefully exhale in one second. →Converted to a percentage of normal. →Marker for the degree of pulmonary obstruction. →FEV₁ greater than 80% of predicted=normal. →FEV₁ 60%-79% of predicted=Mild obstruction. →FEV₁ less than 40% of predicted=Severe obstruction.

Important elements of history to ask in a patient with asthma?

Frequency of symptoms Presence of nocturnal sx Identification of triggers Current symptoms Previous treatments Past hospitalizations Use of steroids Previous intubations

Clue features: Pain is 1-2h postprandial, relieved by antacids, "burning" sensation, any kind of vomiting, sour taste in mouth

GERD

Common GI causes of chest pain

GERD, PUD, Cholecystitis, Esophageal Spasm

Fibromyalgia Trigger Points

General characteristics a. The fibromyalgia syndrome is a central pain disorder whose cause and pathogenesis are poorly understood. b. Fibromyalgia can occur with RA, SLE, and Sjögren's syndrome. 2. Clinical features a. Patients have nonarticular musculoskeletal aches, pains, fatigue, sleep disturbance, and multiple tender points on examination. b. Anxiety, depression, headaches, irritable bowel syndrome, dysmenorrhea, and paresthesias are associated with this condition.

Diagnostic studies for bronchitis?

Generally not needed. Can get CXR to differentiate it from PNA.

What represents healed primary infection of TB on CXR?

Ghon complexes (calcified primary focus) Ranke complexes (calcified primary focus and calcified hilar lymph node)

What disease is a systemic inflammatory condition of medium and large vessels affecting people over 50 years old, coexists with polymyalgia rheumatica and can cause blindness if not treated appropriately?

Giant cell arteritis

What do you do differently to treat hemorrhagic stroke?

Give prophylactic anticonvulsant like phenytoin because of increased seizure risk, antiplatelet therapy contraindicated

Guillain Barre syndrome vs. botulinism paralysis?

Guillan Barre - ascending paralysis Botulinism - descending paralysis

What are 3 pathogens that cause bronchitis in patients with chronic lung disease?

H. influenzae S. pneumo M. catarrhalis

What causes peptic ulcer disease (PUD)?

H. pylori, NSAID use

Syncope

HEAD, HEART, VESSLS (mnemonic) HEAD- Hypoglycemia/hypoxia, Epilepsy, Anxiety, Dysfunction of brain stem HEART- Heart attack, Embolism of pulmonary artery, Aortic obstruction, Rhythm disturbance, Tachycardia (vtach) VESSLS- Vasovagal, Ectopic pregnancy, Situational, Carotid Sinus sensitivity, Low SVR (hypotension), Subclavial steal *Check blood glucose*

What conditions cause an exaggerated response to bug bites?

HIV CLL

Systolic murmur that can lead to syncope and sudden death

HOCM

Most common STD?

HPV (genital warts)

Causes of Intracerebral hemorrhagic stroke are what?

HTN, amyloidosis, iatrogenic anticoagulation, vascular malformations, cocaine use

JNC 8

HTN: Current Guidelines

MC cause of Hypertrophic Cardiomyopathy

HTN; often idiopathic in young pts though

What bug is most likely to cause pneumonia in a patient with COPD?

Haemophilus

Rhinophyma?

Hard, misformed nose associated with Rosacea

Ventricular septal rupture leads to this murmur:

Harsh systolic murmur

S/S of Bell's Palsy

Has forehead involvement (whereas a stroke does not affect forehead movement), facial weakness, inability to keep one eye closed

Miliary TB?

Hematogenous spread of mycobacterium tuberculosis following reactivation of a latent infection

Acute Hepatitis

Hep B is usually follow with pt with unsafe sex practice.

DVT Treatment

Heparin or LMWH Thrombolysis (role unclear) Inpatient vs home care IVC filter (failed heparin or LMWH, contraindication to anticoagulation, breakthrough DVT)

What liver tumor is associated with oral contraceptive pills? Treatment?

Hepatic adenoma Biopsy is contraindicated b/c of bleeding!

What vaccine to get when you are diabetic and unvaccinated?

Hepatitis B vaccine (b/c of sharing of diabetic care equipment)

Acute Hepatitis info

Hepatitis is an infection or inflammation of the liver. Most cases of acute hepatitis are from viral hepatitis A or B. Hepatitis C, for unknown reasons, rarely presents with an acute infection, and is found as a "silent" infection on blood tests, or unfortunately, when patients present with cirrhosis. Hepatitis D exists exclusively in those who have active viral replication of hepatitis B.

Osler weber rendu syndrome?

Hereditary hemorrhagic telengiectasia *Telengiectasias* AV malformations (bleeding) Aneurysms (bleeding

Patient presents with pulmonary edema, and it disappears by itself?

Heroine induced flash pulmonary edema

Chest Pain (Substernal Tightness) Differentials: *Skin*

Herpes Zoster

What are characteristics of large cell carcinoma?

Heterogeneous group of undifferentiated types that do not fit elsewhere. Cytology shows large cells. Metastasis is early. May be central or peripheral masses.

Diagnostic studies and findings for bronchiectasis? What is the test of choice?

High resolution CT <---test of choice. Will show dilated, tortuous airways CXR - crowded broncial markings and basal cystic spaces. Tram-track lung markings, honeycombing, and atelectasis Bronchoscopy - to evaluate hemoptysis, remove secretions, r/o tumor

Aortic aneurysm workup and treatment

High suspicion(non-urgent, urgent, emergent) US vs CT, 2 large bore IV's, Type and Cross 6-10 units PRBC, BP control, don't over resuscitate Surgical Consult- 3.5-4 cm needs consult and further work up

Pulmonary fibrosis Dx

High-resolution CT: Patchy opacities at the lung bases, often with honeycombing. ■ PFTs: Restrictive pattern. ■ Surgical biopsy (usually required to confirm the diagnosis): Interstitial inflammation, fibrosis, and honeycombing.

Bird or bat droppings?

Histoplasmosis

What will CXR show with primary TB?

Homogenous infiltrates Hilar/paratracheal lymph node enlargement Segmental atelectasis Cavitations with progressive disease

Side effect of Bactrim (TMP-smx)?

Hyperkalemia

Hyperlipidemia

Hyperlipidemia is one of the most important (and modifiable) risk factors for CAD. It causes accelerated atherosclerosis.

What is Hypertension 2°?

Hypertension 2° to an identifiable organic cause

1° (Essential) Hypertension History/PE

Hypertension is asymptomatic until complications develop. ■ Patients should be evaluated for end-organ damage to the brain (stroke, dementia), eye (cotton-wool exudates, hemorrhage), heart (LVH), and kidney (proteinuria, chronic kidney disease). Renal bruits may signify renal artery stenosis as the cause of hypertension.

1° (Essential) Hypertension

Hypertension with no identifiable cause. Represents 95% of cases of hypertension.Risk factors include a family history of hypertension or heart disease, a high-sodium diet, smoking, obesity, race (blacks > whites), and advanced age.

Effect of Hyperthryoidism/Hypothyroidism on CO?

Hyperthyroidism increases CO by decreasing SVR Hypothyroidism decreases CO by increasing SVR T3 relaxes smooth muscle

COPD with clubbed fingers?

Hypertrophic Osteoarthropathy Next best step --> CXR --> might find lung malignancy

Causes of a low anion gap?

Hypoalbuminemia Mutiple myeloma Heavy metal poisoning (lithium) --> act as cations!!

Electrolyte abnormalities on EKG Hypocalcemia Hypercalcemia Hyperkalemia Hypokalemia

Hypocalcemia - Prolonged QT interval Hypercalcemia - Shortened QT interval Hyperkalemia - Peaked T wave Hypokalemia - U waves

Cause of refractory hypokalemia in alcoholics

Hypomagnesia!! Due to renal potassium wasting in the loop of Henle Magnesium is an important cofactor for K+

What electrolyte abnormality can Legionella cause?

Hyponatremia

Which of the causes of hypoxemia present with normal A-a gradient?

Hypoventilation, High Altitude V/Q mismatch, Shunt, Diffusion limitation all have increased A-a gradient

What is the most important and potent stimulus of pulmonary HTN?

Hypoxia (other causes include acidosis and veno-occlusive conditions)

Should Clinical Indications

Hyptension,Cyanosis, Altered mental status, end organ dysfunction, lactate elevation

What is the treatment for VF? What if it occurs within 48 hours of an MI?

ICD If within 48 hours, standard post-MI medical therapy

Signs and symptoms of Intracerebral hemorrhagic stroke are what?

ICP rises, vasoconstriction-sweating


Related study sets

(2) Chapter 17: Neurologic (QUIZ)

View Set

Philosophy Chapter 2 Dualism, Functionalism, Behaviorism)

View Set

Chapter 11 Committed Romantic Relationships

View Set

Early childhood science study guide ( BY: MONICA HERNANDEZ)

View Set

Ryan - Basic Principles of Accounting

View Set

Första deklinationen Grupp F - sjukliga förändringar och tillstånd

View Set

AWS Cloud Practitioner Exam - Coursera Questions

View Set

Corals and Symbiotic Relationships

View Set